OB Exam 2 Quizlets

Lakukan tugas rumah & ujian kamu dengan baik sekarang menggunakan Quizwiz!

Describe the pathophysiology of preeclampsia. What is the root cause? What is the main pathogenic factor?

*The placenta is the root cause* and, thus, the disease begins to resolve with the expelling of the placenta. Inadequate vascular remodeling in uterus = insufficient placental perfusion = endothelial cell dysfunction (release of substance toxic to endothelial cells) = This anomaly causes general vasospasm = poor tissue perfusion in all organ systems, increased peripheral resistance increased BP Increased endothelial cell permeability = loss of intravascular protein and fluid loss = less plasma volume = poor perfusion= further systemic effects Summary: *the main patho factor is not an increase in BP but poor perfusion as a result of vasospasm and reduced plasma volume*

MATCHING Because most pregnant women continue their usual activities, trauma remains a common complication during pregnancy. Approximately 1 in 12 pregnancies in the United States are complicated by trauma each year. As a result of the physiologic alterations that accompany pregnancy, special considerations for the mother and fetus are necessary when trauma occurs. Match the maternal system adaptation in pregnancy with the clinical response to trauma. a. Increased oxygen consumption b. Increased heart rate c. Decreased gastric motility d. Displacement of abdominal viscera e. Increase in clotting factors 1. Decreased placental perfusion in the supine position 2. Increased risk of thrombus formation 3. Altered pain referral 4. Increased risk of acidosis 5. Increased risk of aspiration

1. ANS: B DIF: Cognitive Level: Apply REF: p. 732 TOP: Nursing Process: Assessment MSC: Client Needs: Physiologic Integrity NOT: Immediate priorities for the stabilization of the pregnant woman after trauma should be identical to that of the nonpregnant client after trauma. Fetal survival depends on maternal survival, and stabilization of the mother improves the chance of fetal well-being. Trauma may affect a number of systems within the body, and being aware of normal system alterations in the pregnant woman is important for the nurse who is caring for this client. Care should be adapted according to the body system that has been injured. The effects of trauma on pregnancy are also influenced by the length of gestation, type and severity of the injuries, and the degree of disruption of uterine and fetal physiologic features. 2. ANS: E DIF: Cognitive Level: Apply REF: p. 732 TOP: Nursing Process: Assessment MSC: Client Needs: Physiologic Integrity NOT: Immediate priorities for the stabilization of the pregnant woman after trauma should be identical to that of the nonpregnant client after trauma. Fetal survival depends on maternal survival, and stabilization of the mother improves the chance of fetal well-being. Trauma may affect a number of systems within the body, and being aware of normal system alterations in the pregnant woman is important for the nurse who is caring for this client. Care should be adapted according to the body system that has been injured. The effects of trauma on pregnancy are also influenced by the length of gestation, type and severity of the injuries, and the degree of disruption of uterine and fetal physiologic features. 3. ANS: D DIF: Cognitive Level: Apply REF: p. 732 TOP: Nursing Process: Assessment MSC: Client Needs: Physiologic Integrity NOT: Immediate priorities for the stabilization of the pregnant woman after trauma should be identical to that of the nonpregnant client after trauma. Fetal survival depends on maternal survival, and stabilization of the mother improves the chance of fetal well-being. Trauma may affect a number of systems within the body, and being aware of normal system alterations in the pregnant woman is important for the nurse who is caring for this client. Care should be adapted according to the body system that has been injured. The effects of trauma on pregnancy are also influenced by the length of gestation, type and severity of the injuries, and the degree of disruption of uterine and fetal physiologic features. 4. ANS: A DIF: Cognitive Level: Apply REF: p. 732 TOP: Nursing Process: Assessment MSC: Client Needs: Physiologic Integrity NOT: Immediate priorities for the stabilization of the pregnant woman after trauma should be identical to that of the nonpregnant client after trauma. Fetal survival depends on maternal survival, and stabilization of the mother improves the chance of fetal well-being. Trauma may affect a number of systems within the body, and being aware of normal system alterations in the pregnant woman is important for the nurse who is caring for this client. Care should be adapted according to the body system that has been injured. The effects of trauma on pregnancy are also influenced by the length of gestation, type and severity of the injuries, and the degree of disruption of uterine and fetal physiologic features. 5. ANS: C DIF: Cognitive Level: Apply REF: p. 732 TOP: Nursing Process: Assessment MSC: Client Needs: Physiologic Integrity NOT: Immediate priorities for the stabilization of the pregnant woman after trauma should be identical to that of the nonpregnant client after trauma. Fetal survival depends on maternal survival, and stabilization of the mother improves the chance of fetal well-being. Trauma may affect a number of systems within the body, and being aware of normal system alterations in the pregnant woman is important for the nurse who is caring for this client. Care should be adapted according to the body system that has been injured. The effects of trauma on pregnancy are also influenced by the length of gestation, type and severity of the injuries, and the degree of disruption of uterine and fetal physiologic features.

Nursing interventions for mild gestational hypertension and preeclampsia without severe features

1. Activity restriction: complete or partial bed rest 2. maternal and fetal assessment 3. Diet: regular diet with adequate protien, calcium, folic acid, zinc, sodium, fluid intake

Nursing intervention for a pt with eclampsia after a siezure

1. Assess status of the woman's airway, breathing and pulse 2. Suction secretions from glottis to clear airway, 3. administer O2 4. Start IV if not in place or if previous infiltrated 5. Administer mg sulfate therapy

What assessment should be performed to assess for hypertensive disorders?

1. BP 2. Edema 3. DTRs (refelcct balance bt cerebral cortex and spinal cord) 4. assess for ankle clonus 5. Presence of proteinuria 6. Evaluate for s & s of severe preeclampsia: headache, epigastric pain, RUQ, visual disturbances

Nursing interventions for severe gestational hypertension and preeclampsia with severe features

1. Bed rest with side rails up 2. darkened enviroment 3. Mg sulfate therapy 4. anti hypertensive medications

Goals of therapy for mild gestational hypertension and preeclampsia without severe features

1. Ensure maternal safety 2. Deliver newborn close to term

Goals of therapy for severe gestational hypertension and preeclampsia with severe features

1. Ensure maternal safety 2. formulate plan for delivery

Describe the diagnostic components/ criteria for preeclampsia

1. HTN: BP >140/90 X 2 at least 4 hrs apart 2. Protienuria: >300 mg in 24-hr specimen 3. Thrombocytopenia: platelet count < 1000,000/ uL 4. Impaired liver functions: elevated liver enzyme (transaminases) 5. Renal insufficiency: new development of serum creatinine > 1.1 mg/dL 6. Pulmonary edema 7. Cerebral/ visual disturbances

Nursing intervention for a pt with eclampsia during seizure

1. Priority care: keep airway patent: turn head to one side, place pillow under one shoulder if possible 2. Call for assistance. Do not leave bedside 3. Raise side rails and pad them with a folded blanket or pillow if possible 4. Observe and record convulsion activity (note time and duration)

What are the systemic effects as a result of poor tissue perfusion and reduced plasma volume in a pt with preeclampsia?

1. Reduced kidney perfusion= decrease in glomerular filtration rate= protein (albumin) lost in urine, retaining of Na+ and H2O; necrosis and renal failure can occur 2. Decrease in plasma colloid osmotic pressure from decrease in levels of serum albumin= decrease in intravascular vol= hemoconcentration, increased blood viscosity, and tissue edema; increase in hct value = arteriolar vasospasm = endothelial damage and increased cap permeability= pulmonary edema 3. Decreased liver perfusion= impaired liver fxn and elevated liver enzyme levels; n&v, epigastric pain, RUQ pain 4. Cerebral edema and hemorrhage in increased CNS irritability (headaches, hyperflexia, clonus in ankle, seizures); arteriolar vasospasms and decreased blood flow to retina= visual disturbances

What is intrapartum nursing care directed toward for a pt diagnosed with severe gestational htn or preeclampsia with severe features?

1. early identification of fetal heart rate abnormalities 2. Prevention of maternal complications

What is HELP syndrome characterized by?

1. hemolysis 2. elevated liver enzymes 3. low platelet

Name four common risk factors for preeclampsia

1. primigravidity in woman <19 or >40 2. First pregnancy with new partner 3. Hx of preeclampsia 4. pregnancy-onset snoring

2. Which condition would require prophylaxis to prevent subacute bacterial endocarditis (SBE) both antepartum and intrapartum? a. Valvular heart disease b. Congestive heart disease c. Arrhythmias d. Postmyocardial infarction

: A Prophylaxis for intrapartum endocarditis and pulmonary infection may be provided for women who have mitral valve prolapse. Prophylaxis for intrapartum endocarditis is not indicated for a client with congestive heart disease, underlying arrhythmias, or postmyocardial infarction. DIF: Cognitive Level: Understand REF: p. 712 TOP: Nursing Process: Implementation

MULTIPLE RESPONSE 1. Which congenital anomalies can occur as a result of the use of antiepileptic drugs (AEDs) in pregnancy? (Select all that apply.) a. Cleft lip b. Congenital heart disease c. Neural tube defects d. Gastroschisis e. Diaphragmatic hernia

: A, B, C Congenital anomalies that can occur with AEDs include cleft lip or palate, congenital heart disease, urogenital defects, and neural tube defects. Carbamazepine and valproate should be avoided if all possible; they may cause neural tube defects. Congenital anomalies of gastroschisis and diaphragmatic hernia are not associated with the use of AEDs. DIF: Cognitive Level: Understand REF: p. 725 TOP: Nursing Process: Planning

3. In caring for a pregnant woman with sickle cell anemia, the nurse must be aware of the signs and symptoms of a sickle cell crisis. What do these include? (Select all that apply.) a. Fever b. Endometritis c. Abdominal pain d. Joint pain e. Urinary tract infection (UTI)

: A, C, D Women with sickle cell anemia have recurrent attacks (crises) of fever and pain, most often in the abdomen, joints, and extremities. These attacks are attributed to vascular occlusion when red blood cells (RBCs) assume the characteristic sickled shape. Crises are usually triggered by dehydration, hypoxia, or acidosis. Women with the sickle cell trait are usually at a greater risk for postpartum endometritis (uterine wall infection); however, this development is not likely to occur during the pregnancy and is not a sign for the disorder. Although women with sickle cell anemia are at an increased risk for UTIs, these infections are not an indication of a sickle cell crisis. DIF: Cognitive Level: Understand REF: p. 721 TOP: Nursing Process: Assessment

2. A lupus flare-up during pregnancy or early postpartum occurs in 15% to 60% of women with this disorder. Which conditions associated with systemic lupus erythematosus (SLE) are maternal risks? (Select all that apply.) a. Miscarriage b. Intrauterine growth restriction (IUGR) c. Nephritis d. Preeclampsia e. Cesarean birth

: A, C, D, E Maternal risks associated with SLE include miscarriage, nephritis, preeclampsia, and cesarean birth. IUGR is a fetal risk related to SLE. Other fetal risks include stillbirth and prematurity. DIF: Cognitive Level: Understand REF: p. 727 TOP: Nursing Process: Assessment

3. Which information should the nurse take into consideration when planning care for a postpartum client with cardiac disease? a. The plan of care for a postpartum client is the same as the plan for any pregnant woman. b. The plan of care includes rest, stool softeners, and monitoring of the effect of activity. c. The plan of care includes frequent ambulating, alternating with active range-of-motion exercises. d. The plan of care includes limiting visits with the infant to once per day.

: B Bed rest may be ordered, with or without bathroom privileges. Bowel movements without stress or strain for the woman are promoted with stool softeners, diet, and fluids. Care of the woman with cardiac disease in the postpartum period is tailored to the woman's functional capacity. The woman will be on bed rest to conserve energy and to reduce the strain on the heart. Although the woman may need help caring for the infant, breastfeeding and infant visits are not contraindicated. DIF: Cognitive Level: Understand REF: pp. 718-719 TOP: Nursing Process: Planning

9. Which information regarding the care of antepartum women with cardiac conditions is mostimportant for the nurse to understand? a. Stress on the heart is greatest in the first trimester and the last 2 weeks before labor. b. Women with class II cardiac disease should avoid heavy exertion and any activity that causes even minor symptoms. c. Women with class III cardiac disease should get 8 to 10 hours of sleep every day and limit housework, shopping, and exercise. d. Women with class I cardiac disease need bed rest through most of the pregnancy and face the possibility of hospitalization near term.

: B Class II cardiac disease is symptomatic with ordinary activity. Women in this category need to avoid heavy exertion and limit regular activities as symptoms dictate. Stress is greatest between weeks 28 and 32 of gestation, when hemodynamic changes reach their maximum. Class III cardiac disease is symptomatic with less-than-ordinary activity. These women need bed rest most of the day and face the possibility of hospitalization near term. Class I cardiac disease is asymptomatic at normal levels of activity. These women can perform limited normal activities with discretion, although they still need a good amount of sleep. DIF: Cognitive Level: Understand REF: p. 711 TOP: Nursing Process: Planning

12. The client makes an appointment for preconception counseling. The woman has a known heart condition and is unsure if she should become pregnant. Which is the only cardiac condition that would cause concern? a. Marfan syndrome b. Eisenmenger syndrome c. Heart transplant d. Ventricular septal defect (VSD)

: B Pregnancy is contraindicated in clients with Eisenmenger syndrome. Women who have had heart transplants are successfully having babies. However, conception should be postponed for at least 1 year after transplantation. Management of the client with Marfan syndrome during pregnancy includes bed rest, beta-blockers, and surgery before conception. VSD is usually corrected early in life and is therefore not a contraindication to pregnancy. DIF: Cognitive Level: Understand REF: p. 714 TOP: Nursing Process: Assessment

15. Bell palsy is an acute idiopathic facial paralysis, the cause for which remains unknown. Which statement regarding this condition is correct? a. Bell palsy is the sudden development of bilateral facial weakness. b. Women with Bell palsy have an increased risk for hypertension. c. Pregnant women are affected twice as often as nonpregnant women. d. Bell palsy occurs most frequently in the first trimester.

: B The clinical manifestations of Bell palsy include the development of unilateral facial weakness, pain surrounding the ears, difficulty closing the eye, and hyperacusis. The cause is unknown; however, Bell palsy may be related to a viral infection. Pregnant women are affected at a rate of three to five times that of nonpregnant women. The incidence rate peaks during the third trimester and puerperium. Women who develop Bell palsy in pregnancy have an increased risk for hypertension. DIF: Cognitive Level: Understand REF: p. 726 TOP: Nursing Process: Assessment

18. It is extremely rare for a woman to die in childbirth; however, it can happen. In the United States, the annual occurrence of maternal death is 12 per 100,000 cases of live birth. What are the leading causes of maternal death? a. Embolism and preeclampsia b. Trauma and motor vehicle accidents (MVAs) c. Hemorrhage and infection d. Underlying chronic conditions

: B Trauma is the leading cause of obstetric death in women of childbearing age. Most maternal injuries are the result of MVAs and falls. Although preeclampsia and embolism are significant contributors to perinatal morbidity, these are not the leading cause of maternal mortality. Maternal death caused by trauma may occur as the result of hemorrhagic shock or abruptio placentae. In these cases, the hemorrhage is the result of trauma, not childbirth. The wish to become a parent is not eliminated by a chronic health problem, and many women each year risk their lives to have a baby. Because of advanced pediatric care, many women are surviving childhood illnesses and reaching adulthood with chronic health problems such as cystic fibrosis, diabetes, and pulmonary disorders. DIF: Cognitive Level: Understand REF: p. 731 TOP: Nursing Process: Assessment

4. Autoimmune disorders often occur during pregnancy because a large percentage of women with an autoimmune disorder are of childbearing age. Which disorders fall into the category of collagen vascular disease? (Select all that apply.) a. Multiple sclerosis b. SLE c. Antiphospholipid syndrome d. Rheumatoid arthritis e. Myasthenia gravis

: B, C, D, E Multiple sclerosis is not an autoimmune disorder. This patchy demyelination of the spinal cord may be a viral disorder. Autoimmune disorders (collagen vascular disease) make up a large group of conditions that disrupt the function of the immune system of the body. These disorders include those listed, as well as systemic sclerosis. DIF: Cognitive Level: Comprehend REF: p. 726 TOP: Nursing Process: Assessment

6. Which important component of nutritional counseling should the nurse include in health teaching for a pregnant woman who is experiencing cholecystitis? a. Assess the woman's dietary history for adequate calories and proteins. b. Teach the woman that the bulk of calories should come from proteins. c. Instruct the woman to eat a low-fat diet and to avoid fried foods. d. Instruct the woman to eat a low-cholesterol, low-salt diet.

: C Eating a low-fat diet and avoiding fried foods is appropriate nutritional counseling for this client. Caloric and protein intake do not predispose a woman to the development of cholecystitis. The woman should be instructed to limit protein intake and choose foods that are high in carbohydrates. A low-cholesterol diet may be the result of limiting fats. However, a low-salt diet is not indicated. DIF: Cognitive Level: Apply REF: p. 728 TOP: Nursing Process: Implementation

10. A woman at 28 weeks of gestation experiences blunt abdominal trauma as the result of a fall. The nurse must closely observe the client for what? a. Alteration in maternal vital signs, especially blood pressure b. Complaints of abdominal pain c. Placental absorption d. Hemorrhage

: C Electronic fetal monitoring (EFM) tracings can help evaluate maternal status after trauma and can reflect fetal cardiac responses to hypoxia and hypoperfusion. Signs and symptoms of placental absorption include uterine irritability, contractions, vaginal bleeding, and changes in FHR characteristics. Hypoperfusion may be present in the pregnant woman before the onset of clinical signs of shock. EFM tracings show the first signs of maternal compromise, such as when the maternal heart rate, blood pressure, and color appear normal, yet the EFM printout shows signs of fetal hypoxia. Abdominal pain, in and of itself, is not the most important symptom. However, if it is accompanied by contractions, changes in the FHR, rupture of membranes, or vaginal bleeding, then the client should be evaluated for abruptio placentae. Clinical signs of hemorrhage do not appear until after a 30% loss of circulating volume occurs. Careful monitoring of fetal status significantly assists in maternal assessment, because the fetal monitor tracing works as an oximeter of internal well-being. DIF: Cognitive Level: Apply REF: p. 732 TOP: Nursing Process: Assessment

16. A pregnant woman at term is transported to the emergency department (ED) after a severe vehicular accident. The obstetric nurse responds and rushes to the ED with a fetal monitor. Cardiopulmonary arrest occurs as the obstetric nurse arrives. What is the highest priority for the trauma team? a. Obtaining IV access, and starting aggressive fluid resuscitation b. Quickly applying the fetal monitor to determine whether the fetus viability c. Starting cardiopulmonary resuscitation (CPR) d. Transferring the woman to the surgical unit for an emergency cesarean delivery in case the fetus is still alive

: C In a situation of severe maternal trauma, the systematic evaluation begins with a primary survey and the initial ABCs (airway, breathing, and circulation) of resuscitation. CPR is initiated first, followed by intravenous (IV) replacement fluid. After immediate resuscitation and successful stabilization measures, a more detailed secondary survey of the mother and fetus should be accomplished. Attempts at maternal resuscitation are made, followed by a secondary survey of the fetus. In the presence of multisystem trauma, a cesarean delivery may be indicated to increase the chance for maternal survival. DIF: Cognitive Level: Apply REF: p. 734 TOP: Nursing Process: Implementation

5. A woman with asthma is experiencing a postpartum hemorrhage. Which drug should be avoided when treating postpartum bleeding to avoid exacerbating asthma? a. Oxytocin (Pitocin) b. Nonsteroidal antiinflammatory drugs (NSAIDs) c. Hemabate d. Fentanyl

: C Prostaglandin derivatives should not be used to treat women with asthma, because they may exacerbate symptoms. Oxytocin is the drug of choice to treat this woman's bleeding; it will not exacerbate her asthma. NSAIDs are not used to treat bleeding. Fentanyl is used to treat pain, not bleeding. DIF: Cognitive Level: Analyze REF: p. 722 TOP: Nursing Process: Planning

4. A woman has experienced iron deficiency anemia during her pregnancy. She had been taking iron for 3 months before the birth. The client gave birth by cesarean 2 days earlier and has been having problems with constipation. After assisting her back to bed from the bathroom, the nurse notes that the woman's stools are dark (greenish-black). What should the nurse's initial action be? a. Perform a guaiac test, and record the results. b. Recognize the finding as abnormal, and report it to the primary health care provider. c. Recognize the finding as a normal result of iron therapy. d. Check the woman's next stool to validate the observation.

: C The nurse should recognize that dark stools are a common side effect in clients who are taking iron replacement therapy. A guaiac test would be indicated if gastrointestinal (GI) bleeding was suspected. GI irritation, including dark stools, is also a common side effect of iron therapy. Observation of stool formation is a normal nursing activity. DIF: Cognitive Level: Apply REF: p. 716 TOP: Nursing Process: Evaluation

11. Which neurologic condition would require preconception counseling, if at all possible? a. Eclampsia b. Bell palsy c. Epilepsy d. Multiple sclerosis

: C Women with epilepsy should receive preconception counseling, if at all possible. Achieving seizure control before becoming pregnant is a desirable state. Medication should also be carefully reviewed. Eclampsia may sometimes be confused with epilepsy, and Bell palsy is a form of facial paralysis; preconception counseling for either condition is not essential to care. Multiple sclerosis is a patchy demyelination of the spinal cord that does not affect the normal course of pregnancy or birth. DIF: Cognitive Level: Understand REF: p. 725 TOP: Nursing Process: Planning

7. Postoperative care of the pregnant woman who requires abdominal surgery for appendicitis includes which additional assessment? a. Intake and output (I&O) and intravenous (IV) site b. Signs and symptoms of infection c. Vital signs and incision d. Fetal heart rate (FHR) and uterine activity

: D Care of a pregnant woman undergoing surgery for appendicitis differs from that for a nonpregnant woman in one significant aspect: the presence of the fetus. Continuous fetal and uterine monitoring should take place. An assessment of I&O levels, along with an assessment of the IV site, are normal postoperative care procedures. Evaluating the client for signs and symptoms of infection is also part of routine postoperative care. Routine vital signs and evaluation of the incision site are expected components of postoperative care. DIF: Cognitive Level: Apply REF: p. 730 TOP: Nursing Process: Assessment

8. Since the gene for cystic fibrosis was identified in 1989, data can be collected for the purposes of genetic counseling for couples regarding carrier status. According to the most recent statistics, how often does cystic fibrosis occur in Caucasian live births? a. 1 in 100 b. 1 in 1000 c. 1 in 2000 d. 1 in 3200

: D Cystic fibrosis occurs in approximately 1 in 3200 Caucasian live births. 1 in 100, 1 in 1000, and 1 in 2000 occurrences of cystic fibrosis in live births are all too frequent rates. DIF: Cognitive Level: Remember REF: p. 722 TOP: Nursing Process: Assessment

13. What form of heart disease in women of childbearing years generally has a benign effect on pregnancy? a. Cardiomyopathy b. Rheumatic heart disease c. Congenital heart disease d. Mitral valve prolapse

: D Mitral valve prolapse is a benign condition that is usually asymptomatic. Cardiomyopathy produces congestive heart failure during pregnancy. Rheumatic heart disease can lead to heart failure during pregnancy. Some congenital heart diseases produce pulmonary hypertension or endocarditis during pregnancy. DIF: Cognitive Level: Remember REF: p. 713 TOP: Nursing Process: Assessment

14. A pregnant woman at 33 weeks of gestation is brought to the birthing unit after a minor automobile accident. The client is experiencing no pain and no vaginal bleeding, her vital signs are stable, and the FHR is 132 beats per minute with variability. What is the nurse's highestpriority? a. Monitoring the woman for a ruptured spleen b. Obtaining a physician's order to discharge her home c. Monitoring her for 24 hours d. Using continuous EFM for a minimum of 4 hours

: D Monitoring the external FHR and contractions is recommended after blunt trauma in a viable gestation for a minimum of 4 hours, regardless of injury severity. Fetal monitoring should be initiated as soon as the woman is stable. In this scenario, no clinical findings indicate the possibility of a ruptured spleen. If the maternal and fetal findings are normal, then EFM should continue for a minimum of 4 hours after a minor trauma or a minor automobile accident. Once the monitoring has been completed and the health care provider is reassured of fetal well-being, the client may be discharged home. Monitoring for 24 hours is unnecessary unless the ERM strip is abnormal or nonreassuring. DIF: Cognitive Level: Apply REF: p. 732 TOP: Nursing Process: Planning

17. Another common pregnancy-specific condition is pruritic urticarial papules and plaques of pregnancy (PUPPP). A client asks the nurse why she has developed this condition and what can be done. What is the nurse's bestresponse? a. PUPPP is associated with decreased maternal weight gain. b. The rate of hypertension decreases with PUPPP. c. This common pregnancy-specific condition is associated with a poor fetal outcome. d. The goal of therapy is to relieve discomfort.

: D PUPPP is associated with increased maternal weight gain, increased rate of twin gestation, and hypertension. It is not, however, associated with poor maternal or fetal outcomes. The goal of therapy is simply to relieve discomfort. Antipruritic topical medications, topical steroids, and antihistamines usually provide relief. PUPPP usually resolves before childbirth or shortly thereafter. DIF: Cognitive Level: Apply REF: p. 724 TOP: Nursing Process: Planning

1. When caring for a pregnant woman with cardiac problems, the nurse must be alert for the signs and symptoms of cardiac decompensation. Which critical findings would the nurse find on assessment of the client experiencing this condition? a. Regular heart rate and hypertension b. Increased urinary output, tachycardia, and dry cough c. Shortness of breath, bradycardia, and hypertension d. Dyspnea, crackles, and an irregular, weak pulse

: D Signs of cardiac decompensation include dyspnea; crackles; an irregular, weak, and rapid pulse; rapid respirations; a moist and frequent cough; generalized edema; increasing fatigue; and cyanosis of the lips and nailbeds. A regular heart rate and hypertension are not generally associated with cardiac decompensation. Of the symptoms of increased urinary output, tachycardia, and dry cough, only tachycardia is indicative of cardiac decompensation. Of the symptoms of shortness of breath, bradycardia, and hypertension, only dyspnea is indicative of cardiac decompensation. DIF: Cognitive Level: Understand REF: p. 716 TOP: Nursing Process: Assessment

The nurse providing care for the laboring woman should understand that amnioinfusion is used to treat: A. Variable decelerations B. Late decelerations C. Fetal bradycardia D. Fetal tachycardia

A A. Correct: Amnioinfusion is used during labor either to dilute meconium-stained amniotic fluid or to supplement the amount of amniotic fluid to reduce the severity of variable decelerations caused by cord compression. B. Incorrect: Amnioinfusion has no bearing on this alteration in FHR tracings. C. Incorrect: Amnioinfusion has no bearing on this alteration in FHR tracings. D. Incorrect: Amnioinfusion has no bearing on this alteration in FHR tracings. p. 508

A normal uterine activity (UA) pattern in labor is characterized by: A. Contractions every 2 to 5 minutes B. Contractions lasting about 2 minutes C. Contractions about 1 minute apart D. A contraction intensity of about 1000 mm Hg with relaxation at 50 mm Hg

A A. Correct: Contractions normally occur every 2 to 5 minutes and last less than 90 seconds (intensity 800 mm Hg) with about 30 seconds in between (20 mm Hg or less). B. Incorrect: Contractions normally occur every 2 to 5 minutes and last less than 90 seconds (intensity 800 mm Hg) with about 30 seconds in between (20 mm Hg or less). C. Incorrect: Contractions normally occur every 2 to 5 minutes and last less than 90 seconds (intensity 800 mm Hg) with about 30 seconds in between (20 mm Hg or less). D. Incorrect: Contractions normally occur every 2 to 5 minutes and last less than 90 seconds (intensity 800 mm Hg) with about 30 seconds in between (20 mm Hg or less). p. 498

Which deceleration of the FHR would NOT require the nurse to change the maternal position? A. Early decelerations B. Late decelerations C. Variable decelerations D. It is always a good idea to change the woman's position.

A A. Correct: Early decelerations (and accelerations) generally do not need any nursing intervention. B. Incorrect: Late decelerations suggest that the nurse should change the maternal position (lateral); variable decelerations also require a maternal position change (side to side). C. Incorrect: Late decelerations suggest that the nurse should change the maternal position (lateral); variable decelerations also require a maternal position change (side to side). D. Incorrect: Although changing positions throughout labor is recommended, it is not required in response to early decelerations. p. 505

The nurse caring for the laboring woman should understand that early decelerations are caused by: A. Altered fetal cerebral blood flow B. Umbilical cord compression C. Uteroplacental insufficiency D. Spontaneous rupture of membranes

A A. Correct: Early decelerations are the fetus's response to fetal head compression. B. Incorrect: Variable decelerations are associated with umbilical cord compression. C. Incorrect: Late decelerations are associated with uteroplacental insufficiency. D. Incorrect: Spontaneous rupture of membranes has no bearing on the FHR unless the umbilical cord prolapses, which would result in variable or prolonged bradycardia. p. 507

The nurse providing care for the laboring woman should understand that accelerations with fetal movement: A. Are reassuring B. Are caused by umbilical cord compression C. Warrant close observation D. Are caused by uteroplacental insufficiency

A A. Correct: Episodic accelerations in the FHR occur during fetal movement and are indications of fetal well-being. B. Incorrect: Umbilical cord compression results in variable decelerations in the FHR. C. Incorrect: Accelerations in the FHR are an indication of fetal well-being and do not warrant close observation. D. Incorrect: Uteroplacental insufficiency would result in late decelerations in the FHR. p. 504

Fetal tachycardia is most common during: A. Maternal fever B. Umbilical cord prolapse C. Regional anesthesia D. MgSO4 administration

A A. Correct: Fetal tachycardia can be considered an early sign of fetal hypoxemia and can also result from maternal or fetal infection. B. Incorrect: This situation most likely would result in fetal bradycardia, not tachycardia. C. Incorrect: This situation most likely would result in fetal bradycardia, not tachycardia. D. Incorrect: This situation most likely would result in fetal bradycardia, not tachycardia. p. 505

Fetal well-being during labor is assessed by: A. The response of the FHR to uterine contractions (UCs) B. Maternal pain control C. Accelerations in the FHR D. An FHR above 110 beats/min

A A. Correct: Fetal well-being during labor can be measured by the response of the FHR to UCs. In general, reassuring FHR patterns are characterized by an FHR baseline in the range of 110 to 160 beats/min with no periodic changes and a moderate baseline variability, and accelerations with fetal movement. B. Incorrect: Maternal pain control is not the measure used to determine fetal well-being in labor. C. Incorrect: Although FHR accelerations are a reassuring pattern, they are only one component of the criteria by which fetal well-being is assessed. D. Incorrect: Although an FHR above 110 beats/min may be reassuring, it is only one component of the criteria by which fetal well-being is assessed. More information would be needed to determine fetal well-being. p. 498

While evaluating an external monitor tracing of a woman in active labor, the nurse notes that the fetal heart rate (FHR) for five sequential contractions begins to decelerate late in the contraction, with the nadir of the decelerations occurring after the peak of the contraction. The nurse's first priority is to: A. Change the woman's position B. Notify the care provider C. Assist with amnioinfusion D. Insert a scalp electrode

A A. Correct: Late decelerations may be caused by maternal supine hypotension syndrome. They usually are corrected when the woman turns on her side to displace the weight of the gravid uterus from the vena cava. B. Incorrect: If the fetus does not respond to primary nursing interventions for late decelerations, the nurse would continue with subsequent intrauterine resuscitation measures, including notifying the care provider. C. Incorrect: An amnioinfusion may be used to relieve pressure on an umbilical cord that has not prolapsed. The fetal heart rate pattern associated with this situation most likely reveals variable deceleration. D. Incorrect: A fetal scalp electrode would provide accurate data for evaluating the well-being of the fetus; however, this is not a nursing intervention that would alleviate late decelerations, nor is it the nurse's first priority. p. 507

The nurse caring for a laboring woman is aware that maternal cardiac output can be increased by: A. Change in position B. Oxytocin administration C. Regional anesthesia D. Intravenous analgesic

A A. Correct: Maternal supine hypotension syndrome is caused by the weight and pressure of the gravid uterus on the ascending vena cava when the woman is in a supine position. This reduces venous return to the woman's heart, as well as cardiac output, and subsequently reduces her blood pressure. The nurse can encourage the woman to change positions and to avoid the supine position. B. Incorrect: This intervention may reduce maternal cardiac output. C. Incorrect: This intervention may reduce maternal cardiac output. D. Incorrect: This intervention may reduce maternal cardiac output. p. 503

Perinatal nurses are legally responsible for: A. Correctly interpreting FHR patterns, initiating appropriate nursing interventions, and documenting the outcomes B. Greeting the client on arrival, assessing her, and starting an IV line C. Applying the external fetal monitor and notifying the care provider D. Making sure the woman is comfortable

A A. Correct: Nurses who care for women during childbirth are legally responsible for correctly interpreting FHR patterns, initiating appropriate nursing interventions based on those patterns, and documenting the outcomes of those interventions. B. Incorrect: This may be an activity that a nurse performs, but it is not an activity for which the nurse is legally responsible. C. Incorrect: This may be an activity that a nurse performs, but it is not an activity for which the nurse is legally responsible. D. Incorrect: This is one aspect of caring for a woman in labor, but it is not an activity for which the nurse is legally responsible. p. 511

The nurse caring for the woman in labor should understand that decreased variability of the fetal heart rate would be considered benign if caused by: A. A periodic fetal sleep state B. Uterine palpation C. Uterine contractions D. Maternal activity

A A. Correct: Periodic fetal sleep states usually last 20 to 30 minutes. B. Incorrect: Uterine palpations and contractions, as well as maternal activity, might be (probably) benign signs of increased variability. C. Incorrect: Uterine palpations and contractions, as well as maternal activity, might be (probably) benign signs of increased variability. D. Incorrect: Uterine palpations and contractions, as well as maternal activity, might be (probably) benign signs of increased variability. p. 502

When using IA to assess uterine activity, nurses should be aware that: A. The examiner's hand should be placed over the fundus before, during, and after contractions. B. The frequency and duration of contractions is measured in seconds for consistency. C. Contraction intensity is given a judgment number of 1 to 7 by the nurse and client together. D. The resting tone between contractions is described as either placid or turbulent.

A A. Correct: The assessment is done by palpation; duration, frequency, intensity, and resting tone must be assessed. B. Incorrect: The duration of contractions is measured in seconds; the frequency is measured in minutes. C. Incorrect: The intensity of contractions usually is described as mild, moderate, or strong. D. Incorrect: The resting tone usually is characterized as soft or relaxed. p. 500

In assisting with the two factors that have an effect on fetal status, namely pushing and positioning, nurses should: A. Encourage the woman's cooperation in avoiding the supine position B. Advise the woman to avoid the semi-Fowler position C. Encourage the woman to hold her breath and tighten her abdominal muscles to produce a vaginal response D. Instruct the woman to open her mouth and close her glottis, letting air escape after the push

A A. Correct: The woman should maintain a side-lying position. B. Incorrect: The semi-Fowler position is the recommended side-lying position with a lateral tilt to the uterus. C. Incorrect: This is the Valsalva maneuver, which should be avoided. D. Incorrect: Both the mouth and glottis should be open, letting air escape during the push. p. 516

Fetal tachycardia is most common during: A. Maternal fever B. Umbilical cord prolapse C. Regional anesthesia D. MgSO4 administration

A A. Maternal fever Correct: Fetal tachycardia can be considered an early sign of fetal hypoxemia and can also result from maternal or fetal infection. B. Umbilical cord prolapse Incorrect: This situation most likely would result in fetal bradycardia, not tachycardia. C. Regional anesthesia Incorrect: This situation most likely would result in fetal bradycardia, not tachycardia. D. MgSO4 administration Incorrect: This situation most likely would result in fetal bradycardia, not tachycardia. p. 505

3. Nursing care measures are commonly offered to women in labor. Which nursing measure reflects the application of the gate-control theory? a. Massage the womans back. b. Change the womans position. c. Give the prescribed medication. d. Encourage the woman to rest between contractions.

A (According to the gate-control theory, pain sensations travel along sensory nerve pathways to the brain, but only a limited number of sensations, or messages, can travel through these nerve pathways at one time. Distraction techniques, such as massage or stroking, music, focal points, and imagery, reduce or completely block the capacity of the nerve pathways to transmit pain. These distractions are thought to work by closing down a hypothetic gate in the spinal cord, thus preventing pain signals from reaching the brain. The perception of pain is thereby diminished. Changing the womans position, administering pain medication, and resting between contractions do not reduce or block the capacity of the nerve pathways to transmit pain using the gate-control theory.)

16. The nurse should be cognizant of which important information regarding nerve block analgesia and anesthesia? a. Most local agents are chemically related to cocaine and end in the suffix caine. b. Local perineal infiltration anesthesia is effective when epinephrine is added, but it can be injected only once. c. Pudendal nerve block is designed to relieve the pain from uterine contractions. d. Pudendal nerve block, if performed correctly, does not significantly lessen the bearing-down reflex.

A (Common agents include lidocaine and chloroprocaine. Injections can be repeated to prolong the anesthesia. A pudendal nerve block relieves pain in the vagina, vulva, and perineum but not the pain from uterine contractions. A pudendal nerve block lessens or shuts down the bearing-down reflex.)

22. A client is experiencing back labor and complains of intense pain in her lower back. Which measure would best support this woman in labor? a. Counterpressure against the sacrum b. Pant-blow (breaths and puffs) breathing techniques c. Effleurage d. Conscious relaxation or guided imagery

A (Counterpressure is steady pressure applied by a support person to the sacral area with the fist or heel of the hand. This technique helps the woman cope with the sensations of internal pressure and pain in the lower back. The pain management techniques of pant-blow, effleurage, and conscious relaxation or guided imagery are usually helpful for contractions per the gate-control theory.)

10. The nurse should be cognizant of which physiologic effect of pain? a. Predominant pain of the first stage of labor is visceral pain that is located in the lower portion of the abdomen. b. Referred pain is the extreme discomfort experienced between contractions. c. Somatic pain of the second stage of labor is more generalized and related to fatigue. d. Pain during the third stage is a somewhat milder version of the pain experienced during the second stage.

A (Predominant pain comes from cervical changes, the distention of the lower uterine segment, and uterine ischemia. Referred pain occurs when the pain that originates in the uterus radiates to the abdominal wall, lumbosacral area of the back, iliac crests, and gluteal area. Second-stage labor pain is intense, sharp, burning, and localized. Third-stage labor pain is similar to that of the first stage.)

24. Which alterations in the perception of pain by a laboring client should the nurse understand? a. Sensory pain for nulliparous women is often greater than for multiparous women during early labor. b. Affective pain for nulliparous women is usually less than for multiparous women throughout the first stage of labor. c. Women with a history of substance abuse experience more pain during labor. d. Multiparous women have more fatigue from labor and therefore experience more pain.

A (Sensory pain is greater for nulliparous women because their reproductive tract structures are less supple. Affective pain is greater for nulliparous women during the first stage but decreases for both nulliparous and multiparous during the second stage. Women with a history of substance abuse experience the same amount of pain as those without such a history. Nulliparous women have longer labors and therefore experience more fatigue.)

4. Breathing patterns are taught to laboring women. Which breathing pattern should the nurse support for the woman and her coach during the latent phase of the first stage of labor if the couple has attended childbirth preparation classes? a. Slow-paced breathing b. Deep abdominal breathing c. Modified-paced breathing d. Patterned-paced breathing

A (Slow-paced breathing is approximately one half the womans normal breathing rate and is used during the early stages of labor when a woman can no longer walk or talk through her contractions. No such pattern called deep abdominal breathing exists in childbirth preparation. Modified-paced breathing is shallow breathing that is twice the womans normal breathing rate. It is used when labor progresses and the woman can no longer maintain relaxation through paced breathing. Patterned-pace breathing is a fast, 4:1 breathe, breathe, breathe, blow pattern that is used during the transitional phase of labor just before pushing and delivery.)

6. When assessing a woman in the first stage of labor, which clinical finding will alert the nurse that uterine contractions are effective? a. Dilation of the cervix b. Descent of the fetus to 2 station c. Rupture of the amniotic membranes d. Increase in bloody show

A (The vaginal examination reveals whether the woman is in true labor. Cervical change, especially dilation, in the presence of adequate labor, indicates that the woman is in true labor. Engagement and descent of the fetus are not synonymous and may occur before labor. ROM may occur with or without the presence of labor. Bloody show may indicate a slow, progressive cervical change (e.g., effacement) in both true and false labor.)

3. The nurse is caring for a client in early labor. Membranes ruptured approximately 2 hours earlier. This client is at increased risk for which complication? a. Intrauterine infection b. Hemorrhage c. Precipitous labor d. Supine hypotension

A (When the membranes rupture, microorganisms from the vagina can ascend into the amniotic sac, causing chorioamnionitis and placentitis. ROM is not associated with fetal or maternal bleeding. Although ROM may increase the intensity of the contractions and facilitate active labor, it does not result in precipitous labor. ROM has no correlation with supine hypotension.)

19. Conscious relaxation is associated with which method of childbirth preparation? a. Grantly Dick-Read childbirth method b. Lamaze method c. Bradley method d. Psychoprophylactic method

A (With the Grantly Dick-Read method, women are taught to consciously and progressively relax different muscle groups throughout the body until a high degree of skill at relaxation is achieved. The Lamaze method combines controlled muscular relaxation with breathing techniques. The Bradley method advocates natural labor, without any form of anesthesia or analgesia, assisted by a husband-coach and using breathing techniques for labor. The psychoprophylactic method is another name for the Lamaze method.)

2. Women who have participated in childbirth education classes often bring a birth plan with them to the hospital. Which items might this plan include? (Select all that apply.) a. Presence of companions b. Clothing to be worn c. Care and handling of the newborn d. Medical interventions e. Date of delivery

A, B, C, D (The presence of companions, clothing to be worn, care and handling of the newborn, medical interventions, and environmental modifications all might be included in the couples birth plan. Other items include the presence of nonessential medical personnel (students), labor activities such as the tub or ambulation, preferred comfort and relaxation methods, and any cultural or religious requirements. The expected date of delivery would not be part of a birth plan unless the client is scheduled for an elective cesarean birth.)

26. Intrauterine growth restriction (IUGR) is associated with numerous pregnancy-related risk factors (Select all that apply). a. Poor nutrition b. Maternal collagen disease c. Gestational hypertension d. Premature rupture of membranes e. Smoking

A, B, C, E

27. Transvaginal ultrasonography is often performed during the first trimester. While preparing your 6-week gestation patient for this procedure, she expresses concerns over the necessity for this test. The nurse should explain that this diagnostic test may be indicated for a number of situations (Select all that apply). a. Multifetal gestation b. Obesity c. Fetal abnormalities d. Amniotic fluid volume e. Ectopic pregnancy

A, B, C, E

1. Emergency conditions during labor that would require immediate nursing intervention can arise with startling speed. Which situations are examples of such an emergency? (Select all that apply.) a. Nonreassuring or abnormal FHR pattern b. Inadequate uterine relaxation c. Vaginal bleeding d. Prolonged second stage e. Prolapse of the cord

A, B, C, E (A nonreassuring or abnormal FHR pattern, inadequate uterine relaxation, vaginal bleeding, infection, and cord prolapse all constitute an emergency during labor that requires immediate nursing intervention. A prolonged second stage of labor after the upper limits for duration is reached. This is 3 hours for nulliparous women and 2 hours for multiparous women.)

5. While developing an intrapartum care plan for the client in early labor, which psychosocial factors would the nurse recognize upon the clients pain experience? (Select all that apply.) a. Culture b. Anxiety and fear c. Previous experiences with pain d. Intervention of caregivers e. Support systems

A, B, C, E (Culture: A womans sociocultural roots influence how she perceives, interprets, and responds to pain during childbirth. Some cultures encourage loud and vigorous expressions of pain, whereas others value self-control. The nurse should avoid praising some behaviors (stoicism) while belittling others (noisy expression). Anxiety and fear: Extreme anxiety and fear magnify the sensitivity to pain and impair a womans ability to tolerate it. Anxiety and fear increase muscle tension in the pelvic area, which counters the expulsive forces of uterine contractions and pushing efforts. Previous experiences with pain: Fear and withdrawal are a natural response to pain during labor. Learning about these normal sensations ahead of time helps a woman suppress her natural reactions of fear regarding the impending birth. If a woman previously had a long and difficult labor, she is likely to be anxious. She may also have learned ways to cope and may use these skills to adapt to the present labor experience. Support systems: An anxious partner is less able to provide help and support to a woman during labor. A womans family and friends can be an important source of support if they convey realistic and positive information about labor and delivery. Although the intervention of caregivers may be necessary for the well-being of the woman and her fetus, some interventions add discomfort to the natural pain of labor (i.e., fetal monitor straps, IV lines).)

2. Which alternative approaches to relaxation have proven successful when working with the client in labor? (Select all that apply.) a. Aromatherapy b. Massage c. Hypnosis d. Cesarean birth e. Biofeedback

A, B, C, E (Approaches to relaxation can include neuromuscular relaxation, aromatherapy, music, massage, imagery, hypnosis, or touch relaxation. Cesarean birth is a method of delivery, not a method of relaxation.)

3. A woman has requested an epidural block for her pain. She is 5 cm dilated and 100% effaced. The baby is in a vertex position and is engaged. The nurse increases the womans IV fluid for a preprocedural bolus. Before the initiation of the epidural, the woman should be informed regarding the disadvantages of an epidural block. Which concerns should the nurse share with this client? (Select all that apply.) a. Ability to move freely is limited. b. Orthostatic hypotension and dizziness may occur. c. Gastric emptying is not delayed. d. Higher body temperature may occur. e. Blood loss is not excessive.

A, B, D (The womans ability to move freely and to maintain control of her labor is limited, related to the use of numerous medical interventions (IV lines and electronic fetal monitoring [EFM]). Significant disadvantages of an epidural block include the occurrence of orthostatic hypotension, dizziness, sedation, and leg weakness. Women who receive an epidural block have a higher body temperature (38 C or higher), especially when labor lasts longer than 12 hours, and may result in an unnecessary neonatal workup for sepsis. An advantage of an epidural block is that blood loss is not excessive. Other advantages include the following: the woman remains alert and able to participate, good relaxation is achieved, airway reflexes remain intact, and only partial motor paralysis develops.)

4. The class of drugs known as opioid analgesics (butorphanol, nalbuphine) is not suitable for administration to women with known opioid dependence. The antagonistic activity could precipitate withdrawal symptoms (abstinence syndrome) in both mothers and newborns. Which signs would indicate opioid or narcotic withdrawal in the mother? (Select all that apply.) a. Yawning, runny nose b. Increase in appetite c. Chills or hot flashes d. Constipation e. Irritability, restlessness

A, C, E (The woman experiencing maternal opioid withdrawal syndrome will exhibit yawning, runny nose, sneezing, anorexia, chills or hot flashes, vomiting, diarrhea, abdominal pain, irritability, restlessness, muscle spasms, weakness, and drowsiness. Assessing both the mother and the newborn and planning the care accordingly are important steps for the nurse to take.)

In assessing the knowledge of a pregestational woman with type 1 diabetes concerning changing insulin needs during pregnancy, the nurse recognizes that further teaching is warranted when the patient states: A.) "I will need to increase my insulin dosage during the first 3 months of pregnancy." B.) "Insulin dosage will likely need to be increased during the second and third trimesters." C.) "Episodes of hypoglycemia are more likely to occur during the first 3 months." D.) "Insulin needs should return to normal within 7 to 10 days after birth if I am bottle-feeding

A.) "I will need to increase my insulin dosage during the first 3 months of pregnancy."

A pregnant woman's amniotic membranes rupture. Prolapsed umbilical cord is suspected. What intervention would be the top priority? A.) Placing the woman in the knee-chest position. B.) Covering the cord in sterile gauze soaked in saline. C.) Preparing the woman for a cesarean birth. D.) Starting oxygen by face mask.

A.) Placing the woman in the knee-chest position.

The priority nursing intervention when admitting a pregnant woman who has experienced a bleeding episode in late pregnancy is to: A.) assess fetal heart rate (FHR) and maternal vital signs. B.) perform a venipuncture for hemoglobin and hematocrit levels. C.) place clean disposable pads to collect any drainage. D.) monitor uterine contractions.

A.) assess fetal heart rate (FHR) and maternal vital signs.

Leopold maneuvers would be an inappropriate method of assessment to determine: A.) gender of the fetus. B.) number of fetuses. C.) fetal lie and attitude. D.) degree of the presenting part's descent into the pelvis.

A.) gender of the fetus.

In relation to primary and secondary powers, the maternity nurse comprehends that: A.) primary powers are responsible for effacement and dilation of the cervix. B.) effacement is generally slower than dilation in women giving birth for the first time. C.) scarring of the cervix caused by a previous infection or surgery may make the delivery a bit more painful, but it should not slow or inhibit dilation. D.) pushing in the second stage of labor is more effective if the woman can breathe deeply and control some of her involuntary needs to push, as the nurse directs.

A.) primary powers are responsible for effacement and dilation of the cervix.

12. When is a prophylactic cerclage for an incompetent cervix usually placed (in weeks of gestation)? a. 12 to 14 b. 6 to 8 c. 23 to 24 d. After 24

ANS: A A prophylactic cerclage is usually placed at 12 to 14 weeks of gestation. The cerclage is electively removed when the woman reaches 37 weeks of gestation or when her labor begins. Six to 8 weeks of gestation is too early to place the cerclage. Cerclage placement is offered if the cervical length falls to less than 20 to 25 mm before 23 to 24 weeks. Although no consensus has been reached, 24 weeks is used as the upper gestational age limit for cerclage placement. DIF: Cognitive Level: Apply REF: p. 674 TOP: Nursing Process: Planning MSC: Client Needs: Health Promotion and Maintenance

23. A client at 39 weeks of gestation has been admitted for an external version. Which intervention would the nurse anticipate the provider to order? a. Tocolytic drug b. Contraction stress test (CST) c. Local anesthetic d. Foley catheter

ANS: A A tocolytic drug will relax the uterus before and during the version, thus making manipulation easier. CST is used to determine the fetal response to stress. A local anesthetic is not used with external version. Although the bladder should be emptied, catheterization is not necessary. DIF: Cognitive Level: Apply REF: p. 779 TOP: Nursing Process: Planning MSC: Client Needs: Physiologic Integrity

18. What is the correct definition of a spontaneous termination of a pregnancy (abortion)? a. Pregnancy is less than 20 weeks. b. Fetus weighs less than 1000 g. c. Products of conception are passed intact. d. No evidence exists of intrauterine infection.

ANS: A An abortion is the termination of pregnancy before the age of viability (20 weeks). The weight of the fetus is not considered because some older fetuses may have a low birth weight. A spontaneous abortion may be complete or incomplete and may be caused by many problems, one being intrauterine infection. DIF: Cognitive Level: Remember REF: p. 669 TOP: Nursing Process: Assessment MSC: Client Needs: Health Promotion and Maintenance

4. A woman in preterm labor at 30 weeks of gestation receives two 12-mg intramuscular (IM) doses of betamethasone. What is the purpose of this pharmacologic intervention? a. To stimulate fetal surfactant production b. To reduce maternal and fetal tachycardia associated with ritodrine administration c. To suppress uterine contractions d. To maintain adequate maternal respiratory effort and ventilation during magnesium sulfate therapy

ANS: A Antenatal glucocorticoids administered as IM injections to the mother accelerate fetal lung maturity. Propranolol (Inderal) is given to reduce the effects of ritodrine administration. Betamethasone has no effect on uterine contractions. Calcium gluconate is given to reverse the respiratory depressive effects of magnesium sulfate therapy. DIF: Cognitive Level: Understand REF: p. 769 TOP: Nursing Process: Planning MSC: Client Needs: Physiologic Integrity

21. What is the highest priority nursing intervention when admitting a pregnant woman who has experienced a bleeding episode in late pregnancy? a. Assessing FHR and maternal vital signs b. Performing a venipuncture for hemoglobin and hematocrit levels c. Placing clean disposable pads to collect any drainage d. Monitoring uterine contractions

ANS: A Assessment of the FHR and maternal vital signs will assist the nurse in determining the degree of the blood loss and its effect on the mother and fetus. The most important assessment is to check the well-being of both the mother and the fetus. The blood levels can be obtained later. Assessing future bleeding is important; however, the top priority remains mother/fetal well-being. Monitoring uterine contractions is important but not a top priority. DIF: Cognitive Level: Apply REF: p. 681 TOP: Nursing Process: Implementation MSC: Client Needs: Health Promotion and Maintenance

15. Which description most accurately describes the augmentation of labor? a. Is part of the active management of labor that is instituted when the labor process is unsatisfactory b. Relies on more invasive methods when oxytocin and amniotomy have failed c. Is a modern management term to cover up the negative connotations of forceps-assisted birth d. Uses vacuum cups

ANS: A Augmentation is part of the active management of labor that stimulates uterine contractions after labor has started but is not progressing satisfactorily. Augmentation uses amniotomy and oxytocin infusion, as well as some more gentle, noninvasive methods. Forceps-assisted births are less common than in the past and not considered a method of augmentation. A vacuum-assisted delivery occurs during childbirth if the mother is too exhausted to push. Vacuum extraction is not considered an augmentation methodology. DIF: Cognitive Level: Understand REF: pp. 785-786 TOP: Nursing Process: Planning MSC: Client Needs: Health Promotion and Maintenance

8. The client is being induced in response to worsening preeclampsia. She is also receiving magnesium sulfate. It appears that her labor has not become active, despite several hours of oxytocin administration. She asks the nurse, "Why is this taking so long?" What is the nurse's most appropriate response? a. "The magnesium is relaxing your uterus and competing with the oxytocin. It may increase the duration of your labor." b. "I don't know why it is taking so long." c. "The length of labor varies for different women." d. "Your baby is just being stubborn."

ANS: A Because magnesium sulfate is a tocolytic agent, its use may increase the duration of labor. The amount of oxytocin needed to stimulate labor may be more than that needed for the woman who is not receiving magnesium sulfate. The nurse should explain to the client the effects of magnesium sulfate on the duration of labor. Although the length of labor varies for different women, the most likely reason this woman's labor is protracted is the tocolytic effects of magnesium sulfate. The behavior of the fetus has no bearing on the length of labor. DIF: Cognitive Level: Apply REF: p. 664 TOP: Nursing Process: Planning MSC: Client Needs: Health Promotion and Maintenance

Of these psychosocial factors, which has the least negative effect on the health of the mother and/or fetus? a. Moderate coffee consumption b. Moderate alcohol consumption c. Cigarette smoke d. Emotional distress

ANS: A Birth defects in humans have not been related to caffeine consumption. Pregnant women who consume more than 300 mg of caffeine daily may be at increased risk for miscarriage or intrauterine growth restriction (IUGR).

11. Which intervention is most important when planning care for a client with severe gestational hypertension? a. Induction of labor is likely, as near term as possible. b. If at home, the woman should be confined to her bed, even with mild gestational hypertension. c. Special diet low in protein and salt should be initiated. d. Vaginal birth is still an option, even in severe cases.

ANS: A By 34 weeks of gestation, the risk of continuing the pregnancy may be considered greater than the risks of a preterm birth. Strict bed rest is controversial for mild cases; some women in the hospital are even allowed to move around. Diet and fluid recommendations are essentially the same as for healthy pregnant women, although some authorities have suggested a diet high in protein. Women with severe gestational hypertension should expect a cesarean delivery. DIF: Cognitive Level: Apply REF: p. 660 TOP: Nursing Process: Planning MSC: Client Needs: Health Promotion and Maintenance

22. Which order should the nurse expect for a client admitted with a threatened abortion? a. Bed rest b. Administration of ritodrine IV c. Nothing by mouth (nil per os [NPO]) d. Narcotic analgesia every 3 hours, as needed

ANS: A Decreasing the woman's activity level may alleviate the bleeding and allow the pregnancy to continue. Ritodrine is not the first drug of choice for tocolytic medications. Having the woman placed on NPO is unnecessary. At times, dehydration may produce contractions; therefore, hydration is important. Narcotic analgesia will not decrease the contractions and may mask the severity of the contractions. DIF: Cognitive Level: Understand REF: pp. 671-672 TOP: Nursing Process: Planning MSC: Client Needs: Health Promotion and Maintenance

17. A woman with severe preeclampsia has been receiving magnesium sulfate by intravenous infusion for 8 hours. The nurse assesses the client and documents the following findings: temperature of 37.1° C, pulse rate of 96 beats per minute, respiratory rate of 24 breaths per minute, BP of 155/112 mm Hg, 3+ DTRs, and no ankle clonus. The nurse calls the provider with an update. The nurse should anticipate an order for which medication? a. Hydralazine b. Magnesium sulfate bolus c. Diazepam d. Calcium gluconate

ANS: A Hydralazine is an antihypertensive medication commonly used to treat hypertension in severe preeclampsia. Typically, it is administered for a systolic BP higher than 160 mm Hg or a diastolic BP higher than 110 mm Hg. An additional bolus of magnesium sulfate may be ordered for increasing signs of CNS irritability related to severe preeclampsia (e.g., clonus) or if eclampsia develops. Diazepam is sometimes used to stop or shorten eclamptic seizures. Calcium gluconate is used as the antidote for magnesium sulfate toxicity. The client is not currently displaying any signs or symptoms of magnesium toxicity. DIF: Cognitive Level: Analyze REF: p. 665 TOP: Nursing Process: Planning MSC: Client Needs: Physiologic Integrity

16. The exact cause of preterm labor is unknown but believed to be multifactorial. Infection is thought to be a major factor in many preterm labors. Which type of infection has not been linked to preterm birth? a. Viral b. Periodontal c. Cervical d. Urinary tract

ANS: A Infections that increase the risk of preterm labor and birth are bacterial and include cervical, urinary tract, periodontal, and other bacterial infections. Therefore, early, continual, and comprehensive participation by the client in her prenatal care is important. Recent evidence has shown a link between periodontal infections and preterm labor. Researchers recommend regular dental care before and during pregnancy, oral assessment as a routine part of prenatal care, and scrupulous oral hygiene to prevent periodontal infections. DIF: Cognitive Level: Remember REF: p. 760 TOP: Nursing Process: Assessment MSC: Client Needs: Physiologic Integrity

15. Which neonatal complications are associated with hypertension in the mother? a. Intrauterine growth restriction (IUGR) and prematurity b. Seizures and cerebral hemorrhage c. Hepatic or renal dysfunction d. Placental abruption and DIC

ANS: A Neonatal complications are related to placental insufficiency and include IUGR, prematurity, and necrotizing enterocolitis. Seizures and cerebral hemorrhage are maternal complications. Hepatic and renal dysfunction are maternal complications of hypertensive disorders in pregnancy. Placental abruption and DIC are conditions related to maternal morbidity and mortality. DIF: Cognitive Level: Understand REF: p. 667 TOP: Nursing Process: Assessment MSC: Client Needs: Physiologic Integrity

19. What is the most common medical complication of pregnancy? a. Hypertension b. Hyperemesis gravidarum c. Hemorrhagic complications d. Infections

ANS: A Preeclampsia and eclampsia are two noted deadly forms of hypertension. A large percentage of pregnant women will have nausea and vomiting, but a relatively few will have the severe form called hyperemesis gravidarum. Hemorrhagic complications are the second most common medical complication of pregnancy; hypertension is the most common. Infection is a risk factor for preeclampsia. DIF: Cognitive Level: Remember REF: p. 653 TOP: Nursing Process: Assessment MSC: Client Needs: Physiologic Integrity

14. In caring for the woman with DIC, which order should the nurse anticipate? a. Administration of blood b. Preparation of the client for invasive hemodynamic monitoring c. Restriction of intravascular fluids d. Administration of steroids

ANS: A Primary medical management in all cases of DIC involves a correction of the underlying cause, volume replacement, blood component therapy, optimization of oxygenation and perfusion status, and continued reassessment of laboratory parameters. Central monitoring would not be initially ordered in a client with DIC because it could contribute to more areas of bleeding. Management of DIC would include volume replacement, not volume restriction. Steroids are not indicated for the management of DIC. DIF: Cognitive Level: Apply REF: pp. 685-686 TOP: Nursing Process: Planning MSC: Client Needs: Physiologic Integrity

9. What nursing diagnosis is the most appropriate for a woman experiencing severe preeclampsia? a. Risk for injury to mother and fetus, related to central nervous system (CNS) irritability b. Risk for altered gas exchange c. Risk for deficient fluid volume, related to increased sodium retention secondary to the administration of magnesium sulfate d. Risk for increased cardiac output, related to the use of antihypertensive drugs

ANS: A Risk for injury is the most appropriate nursing diagnosis for this client scenario. Gas exchange is more likely to become impaired, attributable to pulmonary edema. A risk for excess, not deficient, fluid volume, related to increased sodium retention, is increased, and a risk for decreased, not increased, cardiac output, related to the use of antihypertensive drugs, also is increased. DIF: Cognitive Level: Apply REF: p. 660 TOP: Nursing Process: Diagnosis MSC: Client Needs: Physiologic Integrity

9. With regard to hemorrhagic complications that may occur during pregnancy, what information is most accurate? a. An incompetent cervix is usually not diagnosed until the woman has lost one or two pregnancies. b. Incidences of ectopic pregnancy are declining as a result of improved diagnostic techniques. c. One ectopic pregnancy does not affect a woman's fertility or her likelihood of having a normal pregnancy the next time. d. Gestational trophoblastic neoplasia (GTN) is one of the persistently incurable gynecologic malignancies.

ANS: A Short labors and recurring losses of pregnancy at progressively earlier gestational ages are characteristics of reduced cervical competence. Because diagnostic technology is improving, more ectopic pregnancies are being diagnosed. One ectopic pregnancy places the woman at increased risk for another one. Ectopic pregnancy is a leading cause of infertility. Once invariably fatal, GTN now is the most curable gynecologic malignancy. DIF: Cognitive Level: Understand REF: p. 675 TOP: Nursing Process: Assessment MSC: Client Needs: Health Promotion and Maintenance

13. In caring for an immediate postpartum client, the nurse notes petechiae and oozing from her intravenous (IV) site. The client would be closely monitored for which clotting disorder? a. DIC b. Amniotic fluid embolism (AFE) c. Hemorrhage d. HELLP syndrome

ANS: A The diagnosis of DIC is made according to clinical findings and laboratory markers. A physical examination reveals unusual bleeding. Petechiae may appear around a blood pressure cuff on the woman's arm. Excessive bleeding may occur from the site of slight trauma such as venipuncture sites. These symptoms are not associated with AFE, nor is AFE a bleeding disorder. Hemorrhage occurs for a variety of reasons in the postpartum client. These symptoms are associated with DIC. Hemorrhage would be a finding associated with DIC and is not a clotting disorder in and of itself. HELLP syndrome is not a clotting disorder, but it may contribute to the clotting disorder DIC. DIF: Cognitive Level: Understand REF: p. 685 TOP: Nursing Process: Planning MSC: Client Needs: Physiologic Integrity

25. Which nursing intervention should be immediately performed after the forceps-assisted birth of an infant? a. Assessing the infant for signs of trauma b. Administering prophylactic antibiotic agents to the infant c. Applying a cold pack to the infant's scalp d. Measuring the circumference of the infant's head

ANS: A The infant should be assessed for bruising or abrasions at the site of application, facial palsy, and subdural hematoma. Prophylactic antibiotics are not necessary with a forceps delivery. A cold pack would place the infant at risk for cold stress and is contraindicated. Measuring the circumference of the head is part of the initial nursing assessment. DIF: Cognitive Level: Apply REF: p. 788 TOP: Nursing Process: Implementation MSC: Client Needs: Physiologic Integrity

20. A pregnant woman's amniotic membranes have ruptured. A prolapsed umbilical cord is suspected. What intervention would be the nurse's highest priority? a. Placing the woman in the knee-chest position b. Covering the cord in sterile gauze soaked in saline c. Preparing the woman for a cesarean birth d. Starting oxygen by face mask

ANS: A The woman is assisted into a modified Sims position, Trendelenburg position, or the knee-chest position in which gravity keeps the pressure of the presenting part off the cord. Although covering the cord in sterile gauze soaked saline, preparing the woman for a cesarean, and starting oxygen by face mark are appropriate nursing interventions in the event of a prolapsed cord, the intervention of top priority would be positioning the mother to relieve cord compression. DIF: Cognitive Level: Apply REF: p. 797 TOP: Nursing Process: Implementation MSC: Client Needs: Physiologic Integrity

8. The nurse expects which maternal cardiovascular finding during labor? a. Increased cardiac output b. Decreased pulse rate c. Decreased white blood cell (WBC) count d. Decreased blood pressure

ANS: A During each contraction, 400 ml of blood is emptied from the uterus into the maternal vascular system, which increases cardiac output by approximately 10% to 15% during the first stage of labor and by approximately 30% to 50% in the second stage of labor. The heart rate increases slightly during labor. The WBC count can increase during labor. During the first stage of labor, uterine contractions cause systolic readings to increase by approximately 10 mm Hg. During the second stage, contractions may cause systolic pressures to increase by 30 mm Hg and diastolic readings to increase by 25 mm Hg. DIF: Cognitive Level: Understand REF: p. 379 TOP: Nursing Process: Diagnosis

14. Which description of the four stages of labor is correct for both the definition and the duration? a. First stage: onset of regular uterine contractions to full dilation; less than 1 hour to 20 hours b. Second stage: full effacement to 4 to 5 cm; visible presenting part; 1 to 2 hours c. Third stage: active pushing to birth; 20 minutes (multiparous woman), 50 minutes (nulliparous woman) d. Fourth stage: delivery of the placenta to recovery; 30 minutes to 1 hour

ANS: A Full dilation may occur in less than 1 hour, but in first-time pregnancies full dilation can take up to 20 hours. The second stage of labor extends from full dilation to birth and takes an average of 20 to 50 minutes, although 2 hours is still considered normal. The third stage of labor extends from birth to the expulsion of the placenta and usually takes a few minutes. The fourth stage begins after the expulsion of the placenta and lasts until homeostasis is reestablished (approximately 2 hours). DIF: Cognitive Level: Understand REF: pp. 376-377 TOP: Nursing Process: Diagnosis

10. Which presentation is accurately described in terms of both the resenting part and the frequency of occurrence? a. Cephalic: occiput, at least 96% b. Breech: sacrum, 10% to 15% c. Shoulder: scapula, 10% to 15% d. Cephalic: cranial, 80% to 85%

ANS: A In cephalic presentations (head first), the presenting part is the occiput; this presentation occurs in 96% of births. In a breech birth, the sacrum emerges first; this presentation occurs in approximately 3% of births. In shoulder presentations, the scapula emerges first; this presentation occurs in only 1% of births. In a cephalic presentation, the part of the head or cranium that emerges first is the occiput; cephalic presentations occur in 96% of births. DIF: Cognitive Level: Understand REF: p. 368 TOP: Nursing Process: Diagnosis

3. Screening at 24 weeks of gestation reveals that a pregnant woman has gestational diabetes mellitus (GDM). In planning her care, the nurse and the client mutually agree that an expected outcome is to prevent injury to the fetus as a result of GDM. This fetus is at the greatest risk for which condition? a. Macrosomia b. Congenital anomalies of the central nervous system c. Preterm birth d. Low birth weight

ANS: A Poor glycemic control later in pregnancy increases the rate of fetal macrosomia. Poor glycemic control during the preconception time frame and into the early weeks of the pregnancy is associated with congenital anomalies. Preterm labor or birth is more likely to occur with severe diabetes and is the greatest risk in women with pregestational diabetes. Increased weight, or macrosomia, is the greatest risk factor for this fetus.

9. Which statement concerning the complication of maternal diabetes is the most accurate? a. Diabetic ketoacidosis (DKA) can lead to fetal death at any time during pregnancy. b. Hydramnios occurs approximately twice as often in diabetic pregnancies than in nondiabetic pregnancies. c. Infections occur about as often and are considered about as serious in both diabetic and nondiabetic pregnancies. d. Even mild-to-moderate hypoglycemic episodes can have significant effects on fetal well-being.

ANS: A Prompt treatment of DKA is necessary to save the fetus and the mother. Hydramnios occurs 10 times more often in diabetic pregnancies. Infections are more common and more serious in pregnant women with diabetes. Mild-to-moderate hypoglycemic episodes do not appear to have significant effects on fetal well-being.

7. Which stage of labor varies the most in length? a. First b. Second c. Third d. Fourth

ANS: A The first stage of labor is considered to last from the onset of regular uterine contractions to the full dilation of the cervix. The first stage is significantly longer than the second and third stages combined. In a first-time pregnancy, the first stage of labor can take up to 20 hours. The second stage of labor lasts from the time the cervix is fully dilated to the birth of the fetus. The average length is 20 minutes for a multiparous woman and 50 minutes for a nulliparous woman. The third stage of labor lasts from the birth of the fetus until the placenta is delivered. This stage may be as short as 3 minutes or as long as 1 hour. The fourth stage of labor, recovery, lasts approximately 2 hours after the delivery of the placenta. DIF: Cognitive Level: Remember REF: p. 376 TOP: Nursing Process: Planning

14. Which major neonatal complication is carefully monitored after the birth of the infant of a diabetic mother? a. Hypoglycemia b. Hypercalcemia c. Hypobilirubinemia d. Hypoinsulinemia

ANS: A The neonate is at highest risk for hypoglycemia because fetal insulin production is accelerated during pregnancy to metabolize excessive glucose from the mother. At birth, the maternal glucose supply stops and the neonatal insulin exceeds the available glucose, thus leading to hypoglycemia. Hypocalcemia is associated with preterm birth, birth trauma, and asphyxia, all common problems of the infant of a diabetic mother. Excess erythrocytes are broken down after birth, and large amounts of bilirubin are released into the neonate's circulation, with resulting hyperbilirubinemia. Because fetal insulin production is accelerated during pregnancy, hyperinsulinemia develops in the neonate.

18. What is the nurse's understanding of the appropriate role of primary and secondary powers? a. Primary powers are responsible for the effacement and dilation of the cervix. b. Effacement is generally well ahead of dilation in women giving birth for the first time; they are closer together in subsequent pregnancies. c. Scarring of the cervix caused by a previous infection or surgery may make the delivery a bit more painful, but it should not slow or inhibit dilation. d. Pushing in the second stage of labor is more effective if the woman can breathe deeply and control some of her involuntary needs to push, as the nurse directs.

ANS: A The primary powers are responsible for dilation and effacement; secondary powers are concerned with expulsion of the fetus. Effacement is generally well ahead of dilation in first-time pregnancies; they are closer together in subsequent pregnancies. Scarring of the cervix may slow dilation. Pushing is more effective and less fatiguing when the woman begins to push only after she has the urge to do so. DIF: Cognitive Level: Understand REF: p. 372 TOP: Nursing Process: Planning

2. What are the complications and risks associated with cesarean births? (Select all that apply.) a. Pulmonary edema b. Wound dehiscence c. Hemorrhage d. Urinary tract infections e. Fetal injuries

ANS: A, B, C, D, E Pulmonary edema, wound dehiscence, hemorrhage, urinary tract infections, and fetal injuries are possible complications and risks associated with cesarean births. DIF: Cognitive Level: Understand REF: p. 790 TOP: Nursing Process: Evaluation MSC: Client Needs: Physiologic Integrity

3. Women who are obese are at risk for several complications during pregnancy and birth. Which of these would the nurse anticipate with an obese client? (Select all that apply.) a. Thromboembolism b. Cesarean birth c. Wound infection d. Breech presentation e. Hypertension

ANS: A, B, C, E A breech presentation is not a complication of pregnancy or birth for the client who is obese. Venous thromboembolism is a known risk for obese women. Therefore, the use of thromboembolism-deterrent (TED) hose and sequential compression devices may help decrease the chance for clot formation. Women should also be encouraged to ambulate as soon as possible. In addition to having an increased risk for complications with a cesarean birth, in general, obese women are also more likely to require an emergency cesarean birth. Many obese women have a pannus (i.e., large roll of abdominal fat) that overlies a lower transverse incision made just above the pubic area. The pannus causes the area to remain moist, which encourages the development of infection. Obese women are more likely to begin pregnancy with comorbidities such as hypertension and type 2 diabetes. DIF: Cognitive Level: Analyze REF: p. 778 TOP: Nursing Process: Assessment MSC: Client Needs: Health Promotion and Maintenance

27. At least five factors affect the process of labor and birth. These are easily remembered as the five Ps. Which factors are included in this process? (Select all that apply.) a. Passenger b. Passageway c. Powers d. Pressure e. Psychologic response

ANS: A, B, C, E The five Ps are passenger (fetus and placenta), passageway (birth canal), powers (contractions), position of the mother, and psychologic response. Pressure is not one of the five Ps. DIF: Cognitive Level: Understand REF: p. 367

28. Because of its size and rigidity, the fetal head has a major effect on the birth process. Which bones comprise the structure of the fetal skull? (Select all that apply.) a. Parietal b. Temporal c. Fontanel d. Occipital e. Femoral

ANS: A, B, D The fetal skull has two parietal bones, two temporal bones, an occipital bone, and a frontal bone. The fontanels are membrane-filled spaces. DIF: Cognitive Level: Remember REF: p. 367

3. The reported incidence of ectopic pregnancy has steadily risen over the past 2 decades. Causes include the increase in sexually transmitted infections (STIs) accompanied by tubal infection and damage. The popularity of contraceptive devices such as the IUD has also increased the risk for ectopic pregnancy. The nurse suspects that a client has early signs of ectopic pregnancy. The nurse should be observing the client for which signs or symptoms? (Select all that apply.) a. Pelvic pain b. Abdominal pain c. Unanticipated heavy bleeding d. Vaginal spotting or light bleeding e. Missed period

ANS: A, B, D, E A missed period or spotting can be easily mistaken by the client as an early sign of pregnancy. More subtle signs depend on exactly where the implantation occurs. The nurse must be thorough in her assessment because pain is not a normal symptom of early pregnancy. As the fallopian tube tears open and the embryo is expelled, the client often exhibits severe pain accompanied by intraabdominal hemorrhage, which may progress to hypovolemic shock with minimal or even no external bleeding. In approximately one half of women, shoulder and neck pain results from irritation of the diaphragm from the hemorrhage. DIF: Cognitive Level: Apply REF: p. 676 TOP: Nursing Process: Assessment MSC: Client Needs: Physiologic Integrity

26. Which factors influence cervical dilation? (Select all that apply.) a. Strong uterine contractions b. Force of the presenting fetal part against the cervix c. Size of the woman d. Pressure applied by the amniotic sac e. Scarring of the cervix

ANS: A, B, D, E Dilation of the cervix occurs by the drawing upward of the musculofibrous components of the cervix, which is caused by strong uterine contractions. Pressure exerted by the amniotic fluid while the membranes are intact or by the force applied by the presenting part can also promote cervical dilation. Scarring of the cervix as a result of a previous infection or surgery may slow cervical dilation. Pelvic size or the size of the woman does not affect cervical dilation. DIF: Cognitive Level: Understand REF: p. 374 TOP: Nursing Process: Planning

3. Diabetes refers to a group of metabolic diseases characterized by hyperglycemia resulting from defects in insulin action, insulin secretion, or both. Over time, diabetes causes significant changes in the microvascular and macrovascular circulations. What do these complications include? (Select all that apply.) a. Atherosclerosis b. Retinopathy c. Intrauterine fetal death (IUFD) d. Nephropathy e. Neuropathy f. Autonomic neuropathy

ANS: A, B, D, E These structural changes will most likely affect a variety of systems, including the heart, eyes, kidneys, and nerves. IUFD (stillbirth) remains a major complication of diabetes in pregnancy; however, this is a fetal complication.

2. One of the most important components of the physical assessment of the pregnant client is the determination of BP. Consistency in measurement techniques must be maintained to ensure that the nuances in the variations of the BP readings are not the result of provider error. Which techniques are important in obtaining accurate BP readings? (Select all that apply.) a. The client should be seated. b. The client's arm should be placed at the level of the heart. c. An electronic BP device should be used. d. The cuff should cover a minimum of 60% of the upper arm. e. The same arm should be used for every reading.

ANS: A, B, E BP readings are easily affected by maternal position. Ideally, the client should be seated. An alternative position is left lateral recumbent with the arm at the level of the heart. The arm should always be held in a horizontal position at approximately the level of the heart. The same arm should be used at every visit. The manual sphygmomanometer is the most accurate device. If manual and electronic devices are used in the care setting, then the nurse must use caution when interpreting the readings. A proper size cuff should cover at least 80% of the upper arm or be approximately 1.5 times the length of the upper arm. DIF: Cognitive Level: Apply REF: p. 658 TOP: Nursing Process: Assessment MSC: Client Needs: Physiologic Integrity

1. A serious but uncommon complication of undiagnosed or partially treated hyperthyroidism is a thyroid storm, which may occur in response to stress such as infection, birth, or surgery. What are the signs and symptoms of this emergency disorder? (Select all that apply.) a. Fever b. Hypothermia c. Restlessness d. Bradycardia e. Hypertension

ANS: A, C Fever, restlessness, tachycardia, vomiting, hypotension, and stupor are symptoms of a thyroid storm. Fever, not hypothermia; tachycardia, not bradycardia; and hypotension, not hypertension, are symptoms of thyroid storm.

25. Which changes take place in the woman's reproductive system, days or even weeks before the commencement of labor? (Select all that apply.) a. Lightening b. Exhaustion c. Bloody show d. Rupture of membranes e. Decreased fetal movement

ANS: A, C, D Signs that precede labor may include lightening, urinary frequency, backache, weight loss, surge of energy, bloody show, and rupture of membranes. Many women experience a burst of energy before labor. A decrease in fetal movement is an ominous sign that does not always correlate with labor. DIF: Cognitive Level: Understand REF: p. 376

2. Approximately 10% to 15% of all clinically recognized pregnancies end in miscarriage. What are possible causes of early miscarriage? (Select all that apply.) a. Chromosomal abnormalities b. Infections c. Endocrine imbalance d. Systemic disorders e. Varicella

ANS: A, C, D, E Infections are not a common cause of early miscarriage. At least 50% of pregnancy losses result from chromosomal abnormalities. Endocrine imbalances such as hypothyroidism or diabetes are also possible causes for early pregnancy loss. Other systemic disorders that may contribute to pregnancy loss include lupus and genetic conditions. Although infections are not a common cause of early miscarriage, varicella infection in the first trimester has been associated with pregnancy loss. DIF: Cognitive Level: Remember REF: p. 669 TOP: Nursing Process: Assessment MSC: Client Needs: Health Promotion and Maintenance

4. The induction of labor is considered an acceptable obstetric procedure if it is in the best interest to deliver the fetus. The charge nurse on the labor and delivery unit is often asked to schedule clients for this procedure and therefore must be cognizant of the specific conditions appropriate for labor induction. What are appropriate indications for induction? (Select all that apply?) a. Rupture of membranes at or near term b. Convenience of the woman or her physician c. Chorioamnionitis (inflammation of the amniotic sac) d. Postterm pregnancy e. Fetal death

ANS: A, C, D, E The conditions listed are all acceptable indications for induction. Other conditions include intrauterine growth restriction (IUGR), maternal-fetal blood incompatibility, hypertension, and placental abruption. Elective inductions for the convenience of the woman or her provider are not recommended; however, they have become commonplace. Factors such as rapid labors and living a long distance from a health care facility may be valid reasons in such a circumstance. Elective delivery should not occur before 39 weeks of completed gestation. DIF: Cognitive Level: Apply REF: p. 790 TOP: Nursing Process: Planning MSC: Client Needs: Physiologic Integrity

MULTIPLE RESPONSE 1. Which adverse prenatal outcomes are associated with the HELLP syndrome? (Select all that apply.) a. Placental abruption b. Placenta previa c. Renal failure d. Cirrhosis e. Maternal and fetal death

ANS: A, C, E The HELLP syndrome is associated with an increased risk for adverse perinatal outcomes, including placental abruption, acute renal failure, subcapsular hepatic hematoma, hepatic rupture, recurrent preeclampsia, preterm birth, and fetal and maternal death. The HELLP syndrome is associated with an increased risk for placental abruption, not placenta previa. It is also associated with an increased risk for hepatic hematoma, not cirrhosis. DIF: Cognitive Level: Analyze REF: p. 658 TOP: Nursing Process: Assessment MSC: Client Needs: Physiologic Integrity

MULTIPLE RESPONSE 1. A client who has undergone a D&C for early pregnancy loss is likely to be discharged the same day. The nurse must ensure that her vital signs are stable, that bleeding has been controlled, and that the woman has adequately recovered from the administration of anesthesia. To promote an optimal recovery, what information should discharge teaching include? (Select all that apply.) a. Iron supplementation b. Resumption of intercourse at 6 weeks postprocedure c. Referral to a support group, if necessary d. Expectation of heavy bleeding for at least 2 weeks e. Emphasizing the need for rest

ANS: A, C, E The woman should be advised to consume a diet high in iron and protein. For many women, iron supplementation also is necessary. The nurse should acknowledge that the client has experienced a loss, however early. She can be taught to expect mood swings and possibly depression. Referral to a support group, clergy, or professional counseling may be necessary. Discharge teaching should emphasize the need for rest. Nothing should be placed in the vagina for 2 weeks after the procedure, including tampons and vaginal intercourse. The purpose of this recommendation is to prevent infection. Should infection occur, antibiotics may be prescribed. The client should expect a scant, dark discharge for 1 to 2 weeks. Should heavy, profuse, or bright bleeding occur, she should be instructed to contact her health care provider. DIF: Cognitive Level: Apply REF: p. 672 TOP: Nursing Process: Implementation MSC: Client Needs: Physiologic Integrity

24. What is a maternal indication for the use of vacuum-assisted birth? a. Wide pelvic outlet b. Maternal exhaustion c. History of rapid deliveries d. Failure to progress past station 0

ANS: B A mother who is exhausted may be unable to assist with the expulsion of the fetus. The client with a wide pelvic outlet will likely not require vacuum extraction. With a rapid delivery, vacuum extraction is not necessary. A station of 0 is too high for a vacuum-assisted birth. DIF: Cognitive Level: Understand REF: p. 786 TOP: Nursing Process: Assessment MSC: Client Needs: Physiologic Integrity

18. The client being cared for has severe preeclampsia and is receiving a magnesium sulfate infusion. Which new finding would give the nurse cause for concern? a. Sleepy, sedated affect b. Respiratory rate of 10 breaths per minute c. DTRs of 2 d. Absent ankle clonus

ANS: B A respiratory rate of 10 breaths per minute indicates the client is experiencing respiratory depression from magnesium toxicity. Because magnesium sulfate is a CNS depressant, the client will most likely become sedated when the infusion is initiated. DTRs of 2 and absent ankle clonus are normal findings. DIF: Cognitive Level: Understand REF: p. 664 TOP: Nursing Process: Diagnosis MSC: Client Needs: Physiologic Integrity

14. A number of methods can be used for inducing labor. Which cervical ripening method falls under the category of mechanical or physical? a. Prostaglandins are used to soften and thin the cervix. b. Labor can sometimes be induced with balloon catheters or laminaria tents. c. Oxytocin is less expensive and more effective than prostaglandins but creates greater health risks. d. Amniotomy can be used to make the cervix more favorable for labor.

ANS: B Balloon catheters or laminaria tents are mechanical means of ripening the cervix. Ripening the cervix, making it softer and thinner, increases the success rate of induced labor. Prostaglandin E1 is less expensive and more effective than oxytocin but carries a greater risk. Amniotomy is the artificial rupture of membranes, which is used to induce labor only when the cervix is already ripe. DIF: Cognitive Level: Apply REF: p. 781 TOP: Nursing Process: Planning MSC: Client Needs: Health Promotion and Maintenance

10. The nurse who elects to work in the specialty of obstetric care must have the ability to distinguish between preterm birth, preterm labor, and low birth weight. Which statement regarding this terminology is correct? a. Terms preterm birth and low birth weight can be used interchangeably. b. Preterm labor is defined as cervical changes and uterine contractions occurring between 20 and 37 weeks of gestation. c. Low birth weight is a newborn who weighs below 3.7 pounds. d. Preterm birth rate in the United States continues to increase.

ANS: B Before 20 weeks of gestation, the fetus is not viable (miscarriage); after 37 weeks, the fetus can be considered term. Although these terms are used interchangeably, they have different meanings: preterm birth describes the length of gestation (before 37 weeks), regardless of the newborn's weight; low birth weight describes only the infant's weight at the time of birth (2500 g or less), whenever it occurs. Low birth weight is anything below 2500 g or approximately pounds. In 2011, the preterm birth rate in the United States was 11.7 %; it has dropped every year since 2008. DIF: Cognitive Level: Understand REF: p. 759 TOP: Nursing Process: Assessment MSC: Client Needs: Health Promotion and Maintenance

2. A perinatal nurse is giving discharge instructions to a woman, status postsuction, and curettage secondary to a hydatidiform mole. The woman asks why she must take oral contraceptives for the next 12 months. What is the bestresponse by the nurse? a. "If you get pregnant within 1 year, the chance of a successful pregnancy is very small. Therefore, if you desire a future pregnancy, it would be better for you to use the most reliable method of contraception available." b. "The major risk to you after a molar pregnancy is a type of cancer that can be diagnosed only by measuring the same hormone that your body produces during pregnancy. If you were to get pregnant, then it would make the diagnosis of this cancer more difficult." c. "If you can avoid a pregnancy for the next year, the chance of developing a second molar pregnancy is rare. Therefore, to improve your chance of a successful pregnancy, not getting pregnant at this time is best." d. "Oral contraceptives are the only form of birth control that will prevent a recurrence of a molar pregnancy."

ANS: B Beta-human chorionic gonadotropin (beta-hCG) hormone levels are drawn for 1 year to ensure that the mole is completely gone. The chance of developing choriocarcinoma after the development of a hydatidiform mole is increased. Therefore, the goal is to achieve a zero human chorionic gonadotropin (hCG) level. If the woman were to become pregnant, then it may obscure the presence of the potentially carcinogenic cells. Women should be instructed to use birth control for 1 year after treatment for a hydatidiform mole. The rationale for avoiding pregnancy for 1 year is to ensure that carcinogenic cells are not present. Any contraceptive method except an intrauterine device (IUD) is acceptable. DIF: Cognitive Level: Apply REF: p. 679 TOP: Nursing Process: Planning | Nursing Process: Implementation MSC: Client Needs: Physiologic Integrity

11. Which laboratory marker is indicative of DIC? a. Bleeding time of 10 minutes b. Presence of fibrin split products c. Thrombocytopenia d. Hypofibrinogenemia

ANS: B Degradation of fibrin leads to the accumulation of multiple fibrin clots throughout the body's vasculature. Bleeding time in DIC is normal. Low platelets may occur but are not indicative of DIC because they may be the result from other coagulopathies. Hypofibrinogenemia occurs with DIC. DIF: Cognitive Level: Remember REF: p. 684 TOP: Nursing Process: Assessment MSC: Client Needs: Physiologic Integrity

17. Which maternal condition always necessitates delivery by cesarean birth? a. Marginal placenta previa b. Complete placenta previa c. Ectopic pregnancy d. Eclampsia

ANS: B In complete placenta previa, the placenta completely covers the cervical os. A cesarean birth is the acceptable method of delivery. The risk of fetal death occurring is due to preterm birth. If the previa is marginal (i.e., 2 cm or greater away from the cervical os), then labor can be attempted. A cesarean birth is not indicated for an ectopic pregnancy. Labor can be safely induced if the eclampsia is under control. DIF: Cognitive Level: Understand REF: p. 681 TOP: Nursing Process: Assessment MSC: Client Needs: Physiologic Integrity

19. What is the correct terminology for an abortion in which the fetus dies but is retained within the uterus? a. Inevitable abortion b. Missed abortion c. Incomplete abortion d. Threatened abortion

ANS: B Missed abortion refers to the retention of a dead fetus in the uterus. An inevitable abortion means that the cervix is dilating with the contractions. An incomplete abortion means that not all of the products of conception were expelled. With a threatened abortion, the woman has cramping and bleeding but no cervical dilation. DIF: Cognitive Level: Remember REF: p. 670 TOP: Nursing Process: Assessment MSC: Client Needs: Physiologic Integrity

1. A pregnant woman is being discharged from the hospital after the placement of a cervical cerclage because of a history of recurrent pregnancy loss, secondary to an incompetent cervix. Which information regarding postprocedural care should the nurse emphasize in the discharge teaching? a. Any vaginal discharge should be immediately reported to her health care provider. b. The presence of any contractions, rupture of membranes (ROM), or severe perineal pressure should be reported. c. The client will need to make arrangements for care at home, because her activity level will be restricted. d. The client will be scheduled for a cesarean birth.

ANS: B Nursing care should stress the importance of monitoring for the signs and symptoms of preterm labor. Vaginal bleeding needs to be reported to her primary health care provider. Bed rest is an element of care. However, the woman may stand for periods of up to 90 minutes, which allows her the freedom to see her physician. Home uterine activity monitoring may be used to limit the woman's need for visits and to monitor her status safely at home. The cerclage can be removed at 37 weeks of gestation (to prepare for a vaginal birth), or a cesarean birth can be planned. DIF: Cognitive Level: Apply REF: p. 675 TOP: Nursing Process: Planning | Nursing Process: Implementation MSC: Client Needs: Health Promotion and Maintenance

16. In contrast to placenta previa, what is the most prevalent clinical manifestation of abruptio placentae? a. Bleeding b. Intense abdominal pain c. Uterine activity d. Cramping

ANS: B Pain is absent with placenta previa and may be agonizing with abruptio placentae. Bleeding may be present in varying degrees for both placental conditions. Uterine activity and cramping may be present with both placental conditions. DIF: Cognitive Level: Understand REF: p. 683 TOP: Nursing Process: Diagnosis MSC: Client Needs: Physiologic Integrity

In the United States today: a. More than 20% of pregnancies meet the definition of high risk to either the mother or the infant b. Other than biophysical criteria, sociodemographic factors place both the mother and baby at risk c. High risk pregnancy status extends from first confirmation of pregnancy to birth d. High risk pregnancy is less critical a medical concern because of the reduction in family size and the decrease in unwanted pregnancies

ANS: B Sociodemographic risks include lack of prenatal care, low income, marital status, and ethnicity. Approximately 500,000 of the 4 million births (12.5%) in the United States are categorized as high risk. The high risk status for the mother extends through 30 days after childbirth. The reduction in family size and the decrease in unwanted pregnancies, along with technologic advances that facilitate pregnancies in previously infertile couples and advance the potential of care, have enhanced emphasis on delivering babies safely.

2. Which nursing intervention is paramount when providing care to a client with preterm labor who has received terbutaline? a. Assess deep tendon reflexes (DTRs). b. Assess for dyspnea and crackles. c. Assess for bradycardia. d. Assess for hypoglycemia.

ANS: B Terbutaline is a beta2-adrenergic agonist that affects the mother's cardiopulmonary and metabolic systems. Signs of cardiopulmonary decompensation include adventitious breath sounds and dyspnea. An assessment for dyspnea and crackles is important for the nurse to perform if the woman is taking magnesium sulfate. Assessing DTRs does not address the possible respiratory side effects of using terbutaline. Since terbutaline is a beta2-adrenergic agonist, it can lead to hyperglycemia, not hypoglycemia. Beta2-adrenergic agonist drugs cause tachycardia, not bradycardia. DIF: Cognitive Level: Analyze REF: pp. 767-768 TOP: Nursing Process: Assessment MSC: Client Needs: Physiologic Integrity

4. A 26-year-old pregnant woman, gravida 2, para 1-0-0-1, is 28 weeks pregnant when she experiences bright red, painless vaginal bleeding. On her arrival at the hospital, which diagnostic procedure will the client most likely have performed? a. Amniocentesis for fetal lung maturity b. Transvaginal ultrasound for placental location c. Contraction stress test (CST) d. Internal fetal monitoring

ANS: B The presence of painless bleeding should always alert the health care team to the possibility of placenta previa, which can be confirmed through ultrasonography. Amniocentesis is not performed on a woman who is experiencing bleeding. In the event of an imminent delivery, the fetus is presumed to have immature lungs at this gestational age, and the mother is given corticosteroids to aid in fetal lung maturity. A CST is not performed at a preterm gestational age. Furthermore, bleeding is a contraindication to a CST. Internal fetal monitoring is also contraindicated in the presence of bleeding. DIF: Cognitive Level: Apply REF: p. 680 TOP: Nursing Process: Assessment MSC: Client Needs: Health Promotion and Maintenance

8. A pregnant woman at 29 weeks of gestation has been diagnosed with preterm labor. Her labor is being controlled with tocolytic medications. She asks when she might be able to go home. Which response by the nurse is mostaccurate? a. "After the baby is born." b. "When we can stabilize your preterm labor and arrange home health visits." c. "Whenever your physician says that it is okay." d. "It depends on what kind of insurance coverage you have."

ANS: B This client's preterm labor is being controlled with tocolytics. Once she is stable, home care may be a viable option for this type of client. Care of a client with preterm labor is multidisciplinary and multifactorial; the goal is to prevent delivery. In many cases, this goal may be achieved at home. Managed care may dictate an earlier hospital discharge or a shift from hospital to home care. Insurance coverage may be one factor in client care, but ultimately, client safety remains the most important factor. DIF: Cognitive Level: Apply REF: p. 765 TOP: Nursing Process: Planning MSC: Client Needs: Health Promotion and Maintenance

5. A laboring woman with no known risk factors suddenly experiences spontaneous ROM. The fluid consists of bright red blood. Her contractions are consistent with her current stage of labor. No change in uterine resting tone has occurred. The fetal heart rate (FHR) begins to decline rapidly after the ROM. The nurse should suspect the possibility of what condition? a. Placenta previa b. Vasa previa c. Severe abruptio placentae d. Disseminated intravascular coagulation (DIC)

ANS: B Vasa previa is the result of a velamentous insertion of the umbilical cord. The umbilical vessels are not surrounded by Wharton jelly and have no supportive tissue. The umbilical blood vessels thus are at risk for laceration at any time, but laceration occurs most frequently during ROM. The sudden appearance of bright red blood at the time of ROM and a sudden change in the FHR without other known risk factors should immediately alert the nurse to the possibility of vasa previa. The presence of placenta previa most likely would be ascertained before labor and is considered a risk factor for this pregnancy. In addition, if the woman had a placenta previa, it is unlikely that she would be allowed to pursue labor and a vaginal birth. With the presence of severe abruptio placentae, the uterine tonicity typically is tetanus (i.e., a boardlike uterus). DIC is a pathologic form of diffuse clotting that consumes large amounts of clotting factors, causing widespread external bleeding, internal bleeding, or both. DIC is always a secondary diagnosis, often associated with obstetric risk factors such as the hemolysis, elevated liver enzyme levels, and low platelet levels (HELLP) syndrome. This woman did not have any prior risk factors. DIF: Cognitive Level: Analyze REF: p. 684 TOP: Nursing Process: Diagnosis MSC: Client Needs: Physiologic Integrity

12. An 18-year-old client who has reached 16 weeks of gestation was recently diagnosed with pregestational diabetes. She attends her centering appointment accompanied by one of her girlfriends. This young woman appears more concerned about how her pregnancy will affect her social life than her recent diagnosis of diabetes. A number of nursing diagnoses are applicable to assist in planning adequate care. What is the most appropriate diagnosis at this time? a. Risk for injury, to the fetus related to birth trauma b. Deficient knowledge, related to diabetic pregnancy management c. Deficient knowledge, related to insulin administration d. Risk for injury, to the mother related to hypoglycemia or hyperglycemia

ANS: B Before a treatment plan is developed or goals for the outcome of care are outlined, this client must come to an understanding of diabetes and the potential effects on her pregnancy. She appears more concerned about changes to her social life than adopting a new self-care regimen. Risk for injury to the fetus related to either placental insufficiency or birth trauma may come later in the pregnancy. At this time, the client is having difficulty acknowledging the adjustments that she needs to make to her lifestyle to care for herself during pregnancy. The client may not yet be on insulin. Insulin requirements increase with gestation. The importance of glycemic control must be part of health teaching for this client. However, she has not yet acknowledged that changes to her lifestyle need to be made and may not participate in the plan of care until understanding takes place.

5. A client with maternal phenylketonuria (PKU) has come to the obstetrical clinic to begin prenatal care. Why would this preexisting condition result in the need for closer monitoring during pregnancy? a. PKU is a recognized cause of preterm labor. b. The fetus may develop neurologic problems. c. A pregnant woman is more likely to die without strict dietary control. d. Women with PKU are usually mentally handicapped and should not reproduce.

ANS: B Children born to women with untreated PKU are more likely to be born with mental retardation, microcephaly, congenital heart disease, and low birth weight. Maternal PKU has no effect on labor. Women without dietary control of PKU are more likely to miscarry or bear a child with congenital anomalies. Screening for undiagnosed maternal PKU at the first prenatal visit may be warranted, especially in individuals with a family history of the disorder, with low intelligence of an uncertain cause, or who have given birth to microcephalic infants.

22. What is the primary difference between the labor of a nullipara and that of a multipara? a. Amount of cervical dilation b. Total duration of labor c. Level of pain experienced d. Sequence of labor mechanisms

ANS: B In a first-time pregnancy, the descent is usually slow but steady; in subsequent pregnancies, the descent is more rapid, resulting in a shorter duration of labor. Cervical dilation is the same for all labors. The level of pain is individual to the woman, not to the number of labors she has experienced. The sequence of labor mechanisms is the same with all labors. DIF: Cognitive Level: Understand REF: p. 378

6. The nurse who is caring for a woman hospitalized for hyperemesis gravidarum would expect the initial treatment to involve what? a. Corticosteroids to reduce inflammation b. Intravenous (IV) therapy to correct fluid and electrolyte imbalances c. Antiemetic medication, such as pyridoxine, to control nausea and vomiting d. Enteral nutrition to correct nutritional deficits

ANS: B Initially, the woman who is unable to down clear liquids by mouth requires IV therapy to correct fluid and electrolyte imbalances. Corticosteroids have been successfully used to treat refractory hyperemesis gravidarum, but they are not the expected initial treatment for this disorder. Pyridoxine is vitamin B6, not an antiemetic medication. Promethazine, a common antiemetic, may be prescribed. In severe cases of hyperemesis gravidarum, enteral nutrition via a feeding tube may be necessary to correct maternal nutritional deprivation but is not the initial treatment for this client.

11. A new mother with a thyroid disorder has come for a lactation follow-up appointment. Which thyroid disorder is a contraindication for breastfeeding? a. Hyperthyroidism b. PKU c. Hypothyroidism d. Thyroid storm

ANS: B PKU is a cause of mental retardation in infants; mothers with PKU pass on phenylalanine and therefore should elect not to breastfeed. A woman with either hyperthyroidism or hypothyroidism would have no particular reason not to breastfeed. A thyroid storm is a complication of hyperthyroidism and is not a contraindication to breastfeeding.

1. Preconception counseling is critical in the safe management of diabetic pregnancies. Which complication is commonly associated with poor glycemic control before and during early pregnancy? a. Frequent episodes of maternal hypoglycemia b. Congenital anomalies in the fetus c. Hydramnios d. Hyperemesis gravidarum

ANS: B Preconception counseling is particularly important since strict metabolic control before conception and in the early weeks of gestation is instrumental in decreasing the risk of congenital anomalies. Frequent episodes of maternal hypoglycemia may occur during the first trimester (not before conception) as a result of hormonal changes and the effects on insulin production and use. Hydramnios occurs approximately 10 times more often in diabetic pregnancies than in nondiabetic pregnancies. Typically, it is observed in the third trimester of pregnancy. Hyperemesis gravidarum may exacerbate hypoglycemic events because the decreased food intake by the mother and glucose transfer to the fetus contribute to hypoglycemia.

24. Nurses can help their clients by keeping them informed about the distinctive stages of labor. Which description of the phases of the first stage of labor is accurate? a. Latent: Mild, regular contractions; no dilation; bloody show; duration of 2 to 4 hours b. Active: Moderate, regular contractions; 4- to 7-cm dilation; duration of 3 to 6 hours c. Lull: No contractions; dilation stable; duration of 20 to 60 minutes d. Transition: Very strong but irregular contractions; 8- to 10-cm dilation; duration of 1 to 2 hours

ANS: B The active phase is characterized by moderate, regular contractions; 4- to 7-cm dilation; and a duration of 3 to 6 hours. The latent phase is characterized by mild-to-moderate and irregular contractions; dilation up to 3 cm; brownish-to-pale pink mucus, and a duration of 6 to 8 hours. No official "lull" phase exists in the first stage. The transition phase is characterized by strong- to-very strong and regular contractions; 8- to 10-cm dilation; and a duration of 20 to 40 minutes. DIF: Cognitive Level: Understand REF: p. 376 TOP: Nursing Process: Planning

16. Which statement related to fetal positioning during labor is correct and important for the nurse to understand? a. Position is a measure of the degree of descent of the presenting part of the fetus through the birth canal. b. Birth is imminent when the presenting part is at +4 to +5 cm below the spine. c. The largest transverse diameter of the presenting part is the suboccipitobregmatic diameter. d. Engagement is the term used to describe the beginning of labor.

ANS: B The station of the presenting part should be noted at the beginning of labor to determine the rate of descent. Position is the relationship of the presenting part of the fetus to the four quadrants of the mother's pelvis; station is the measure of degree of descent. The largest diameter is usually the biparietal diameter. The suboccipitobregmatic diameter is the smallest, although one of the most critical. Engagement often occurs in the weeks just before labor in nulliparous women and before or during labor in multiparous women. DIF: Cognitive Level: Understand REF: p. 370

5. The nurse has received a report regarding a client in labor. The woman's last vaginal examination was recorded as 3 cm, 30%, and -2. What is the nurse's interpretation of this assessment? a. Cervix is effaced 3 cm and dilated 30%; the presenting part is 2 cm above the ischial spines. b. Cervix is dilated 3 cm and effaced 30%; the presenting part is 2 cm above the ischial spines. c. Cervix is effaced 3 cm and dilated 30%; the presenting part is 2 cm below the ischial spines. d. Cervix is dilated 3 cm and effaced 30%; the presenting part is 2 cm below the ischial spines.

ANS: B The sterile vaginal examination is recorded as centimeters of cervical dilation, percentage of cervical dilation, and the relationship of the presenting part to the ischial spines (either above or below). For this woman, the cervix is dilated 3 cm and effaced 30%, and the presenting part is 2 cm above the ischial spines. The first interpretation of this vaginal examination is incorrect; the cervix is dilated 3 cm and is 30% effaced. However, the presenting part is correct at 2 cm above the ischial spines. The remaining two interpretations of this vaginal examination are incorrect. Although the dilation and effacement are correct at 3 cm and 30%, the presenting part is actually 2 cm above the ischial spines. DIF: Cognitive Level: Comprehend REF: p. 370 | pp. 373-374

4. A 26-year-old primigravida has come to the clinic for her regular prenatal visit at 12 weeks. She appears thin and somewhat nervous. She reports that she eats a well-balanced diet, although her weight is 5 pounds less than it was at her last visit. The results of laboratory studies confirm that she has a hyperthyroid condition. Based on the available data, the nurse formulates a plan of care. Which nursing diagnosis is most appropriate for the client at this time? a. Deficient fluid volume b. Imbalanced nutrition: less than body requirements c. Imbalanced nutrition: more than body requirements d. Disturbed sleep pattern

ANS: B This client's clinical cues include weight loss, which supports a nursing diagnosis of "Imbalanced nutrition: less than body requirements." No clinical signs or symptoms support a nursing diagnosis of deficient fluid volume. This client reports weight loss, not weight gain. Although the client reports nervousness, the most appropriate nursing diagnosis, based on the client's other clinical symptoms, is "Imbalanced nutrition: less than body requirements."

7. In terms of the incidence and classification of diabetes, which information should the nurse keep in mind when evaluating clients during their ongoing prenatal appointments? a. Type 1 diabetes is most common. b. Type 2 diabetes often goes undiagnosed. c. GDM means that the woman will receive insulin treatment until 6 weeks after birth. d. Type 1 diabetes may become type 2 during pregnancy.

ANS: B Type 2 diabetes often goes undiagnosed because hyperglycemia gradually develops and is often not severe. Type 2, sometimes called adult-onset diabetes, is the most common type of diabetes. GDM refers to any degree of glucose intolerance first recognized during pregnancy; insulin may or may not be needed. People do not go back and forth between type 1 and type 2 diabetes.

21. Which statement is the best rationale for assessing the maternal vital signs between uterine contractions? a. During a contraction, assessing the fetal heart rate is the priority. b. Maternal circulating blood volume temporarily increases during contractions. c. Maternal blood flow to the heart is reduced during contractions. d. Vital signs taken during contractions are not accurate.

ANS: B During uterine contractions, blood flow to the placenta temporarily stops, causing a relative increase in the mother's blood volume, which, in turn, temporarily increases blood pressure and slows the pulse. Monitoring fetal responses to the contractions is important; however, this question concerns the maternal vital signs. Maternal blood flow is increased during a contraction. Vital signs are altered by contractions but are considered accurate for that period. DIF: Cognitive Level: Apply REF: p. 379

9. What is the correct term describing the slight overlapping of cranial bones or shaping of the fetal head during labor? a. Lightening b. Molding c. Ferguson reflex d. Valsalva maneuver

ANS: B Molding also permits adaptation to various diameters of the maternal pelvis. Lightening is the mother's sensation of decreased abdominal distention, which usually occurs the week before labor. The Ferguson reflex is the contraction urge of the uterus after the stimulation of the cervix. The Valsalva maneuver describes conscious pushing during the second stage of labor. DIF: Cognitive Level: Remember REF: p. 367

5. Indications for a primary cesarean birth are often nonrecurring. Therefore, a woman who has had a cesarean birth with a low transverse scar may be a candidate for vaginal birth after cesarean (VBAC). Which clients would beless likely to have a successful VBAC? (Select all that apply.) a. Lengthy interpregnancy interval b. African-American race c. Delivery at a rural hospital d. Estimated fetal weight <4000 g e. Maternal obesity (BMI >30)

ANS: B, C, E Indications for a low success rate for a VBAC delivery include a short interpregnancy interval, non-Caucasian race, gestational age longer than 40 weeks, maternal obesity, preeclampsia, fetal weight greater than 4000 g, and delivery at a rural or private hospital. DIF: Cognitive Level: Understand REF: p. 794 TOP: Nursing Process: Assessment MSC: Client Needs: Physiologic Integrity

MULTIPLE RESPONSE 1. The nurse recognizes that uterine hyperstimulation with oxytocin requires emergency interventions. What clinical cues alert the nurse that the woman is experiencing uterine hyperstimulation? (Select all that apply.) a. Uterine contractions lasting <90 seconds and occurring >2 minutes in frequency b. Uterine contractions lasting >90 seconds and occurring <2 minutes in frequency c. Uterine tone <20 mm Hg d. Uterine tone >20 mm Hg e. Increased uterine activity accompanied by a nonreassuring FHR and pattern

ANS: B, D, E Uterine contractions that occur less frequently than 2 minutes apart and last longer than 90 seconds, a uterine tone over 20 mm Hg, and a nonreassuring FHR and pattern are indications of uterine hyperstimulation with oxytocin administration. Uterine contractions that occur more frequently than 2 minutes apart and last less than 90 seconds are the expected goal of oxytocin induction. A uterine tone less than 20 mm Hg is normal. DIF: Cognitive Level: Analyze REF: p. 785 TOP: Nursing Process: Implementation MSC: Client Needs: Physiologic Integrity

15. A woman arrives at the emergency department with complaints of bleeding and cramping. The initial nursing history is significant for a last menstrual period 6 weeks ago. On sterile speculum examination, the primary care provider finds that the cervix is closed. The anticipated plan of care for this woman would be based on a probable diagnosis of which type of spontaneous abortion? a. Incomplete b. Inevitable c. Threatened d. Septic

ANS: C A woman with a threatened abortion has spotting, mild cramps, and no cervical dilation. A woman with an incomplete abortion would have heavy bleeding, mild-to-severe cramping, and cervical dilation. An inevitable abortion demonstrates the same symptoms as an incomplete abortion: heavy bleeding, mild-to-severe cramping, and cervical dilation. A woman with a septic abortion has malodorous bleeding and typically a dilated cervix. DIF: Cognitive Level: Understand REF: p. 670 TOP: Nursing Process: Planning MSC: Client Needs: Physiologic Integrity

6. A woman with worsening preeclampsia is admitted to the hospital's labor and birth unit. The physician explains the plan of care for severe preeclampsia, including the induction of labor, to the woman and her husband. Which statement by the husband leads the nurse to believe that the couple needs further information? a. "I will help my wife use the breathing techniques that we learned in our childbirth classes." b. "I will give my wife ice chips to eat during labor." c. "Since we will be here for a while, I will call my mother so she can bring the two boys—2 years and 4 years of age—to visit their mother." d. "I will stay with my wife during her labor, just as we planned."

ANS: C Arranging a visit with their two children indicates that the husband does not understand the importance of the quiet, subdued environment that is needed to prevent his wife's condition from worsening. Implementing breathing techniques is indicative of adequate knowledge related to pain management during labor. Administering ice chips indicates an understanding of nutritional needs during labor. Staying with his wife during labor demonstrates the husband's support for his wife and is appropriate. DIF: Cognitive Level: Apply REF: p. 662 TOP: Nursing Process: Evaluation MSC: Client Needs: Psychosocial Integrity

21. What is the primary purpose for the use of tocolytic therapy to suppress uterine activity? a. Drugs can be efficaciously administered up to the designated beginning of term at 37 weeks gestation. b. Tocolytic therapy has no important maternal (as opposed to fetal) contraindications. c. The most important function of tocolytic therapy is to provide the opportunity to administer antenatal glucocorticoids. d. If the client develops pulmonary edema while receiving tocolytic therapy, then intravenous (IV) fluids should be given.

ANS: C Buying time for antenatal glucocorticoids to accelerate fetal lung development may be the best reason to use tocolytic therapy. Once the pregnancy has reached 34 weeks, however, the risks of tocolytic therapy outweigh the benefits. Important maternal contraindications to tocolytic therapy exist. Tocolytic-induced edema can be caused by IV fluids. DIF: Cognitive Level: Comprehend REF: p. 766 TOP: Nursing Process: Planning MSC: Client Needs: Physiologic Integrity

20. What condition indicates concealed hemorrhage when the client experiences abruptio placentae? a. Decrease in abdominal pain b. Bradycardia c. Hard, boardlike abdomen d. Decrease in fundal height

ANS: C Concealed hemorrhage occurs when the edges of the placenta do not separate. The formation of a hematoma behind the placenta and subsequent infiltration of the blood into the uterine muscle results in a very firm, boardlike abdomen. Abdominal pain may increase. The client will have shock symptoms that include tachycardia. As bleeding occurs, the fundal height increases. DIF: Cognitive Level: Analyze REF: p. 683 TOP: Nursing Process: Assessment MSC: Client Needs: Physiologic Integrity

6. A woman arrives for evaluation of signs and symptoms that include a missed period, adnexal fullness, tenderness, and dark red vaginal bleeding. On examination, the nurse notices an ecchymotic blueness around the woman's umbilicus. What does this finding indicate? a. Normal integumentary changes associated with pregnancy b. Turner sign associated with appendicitis c. Cullen sign associated with a ruptured ectopic pregnancy d. Chadwick sign associated with early pregnancy

ANS: C Cullen sign, the blue ecchymosis observed in the umbilical area, indicates hematoperitoneum associated with an undiagnosed ruptured intraabdominal ectopic pregnancy. Linea nigra on the abdomen is the normal integumentary change associated with pregnancy and exhibits a brown pigmented, vertical line on the lower abdomen. Turner sign is ecchymosis in the flank area, often associated with pancreatitis. A Chadwick sign is a blue-purple cervix that may be seen during or around the eighth week of pregnancy. DIF: Cognitive Level: Analyze REF: p. 676 TOP: Nursing Process: Assessment MSC: Client Needs: Physiologic Integrity

2. The labor of a pregnant woman with preeclampsia is going to be induced. Before initiating the oxytocin (Pitocin) infusion, the nurse reviews the woman's latest laboratory test findings, which reveal a platelet count of 90,000 mm3, an elevated aspartate aminotransaminase (AST) level, and a falling hematocrit. The laboratory results are indicative of which condition? a. Eclampsia b. Disseminated intravascular coagulation (DIC) syndrome c. Hemolysis, elevated liver enzyme levels, and low platelet levels (HELLP) syndrome d. Idiopathic thrombocytopenia

ANS: C HELLP syndrome is a laboratory diagnosis for a variant of severe preeclampsia that involves hepatic dysfunction characterized by hemolysis (H), elevated liver (EL) enzymes, and low platelets (LP). Eclampsia is determined by the presence of seizures. DIC is a potential complication associated with HELLP syndrome. Idiopathic thrombocytopenia is the presence of low platelets of unknown cause and is not associated with preeclampsia. DIF: Cognitive Level: Understand REF: p. 657 TOP: Nursing Process: Diagnosis MSC: Client Needs: Physiologic Integrity

12. What is the primary purpose for magnesium sulfate administration for clients with preeclampsia and eclampsia? a. To improve patellar reflexes and increase respiratory efficiency b. To shorten the duration of labor c. To prevent convulsions d. To prevent a boggy uterus and lessen lochial flow

ANS: C Magnesium sulfate is the drug of choice used to prevent convulsions, although it can generate other problems. Loss of patellar reflexes and respiratory depression are signs of magnesium toxicity. Magnesium sulfate can also increase the duration of labor. Women are at risk for a boggy uterus and heavy lochial flow as a result of magnesium sulfate therapy. DIF: Cognitive Level: Understand REF: p. 664 TOP: Nursing Process: Implementation MSC: Client Needs: Physiologic Integrity

3. The nurse is preparing to administer methotrexate to the client. This hazardous drug is most often used for which obstetric complication? a. Complete hydatidiform mole b. Missed abortion c. Unruptured ectopic pregnancy d. Abruptio placentae

ANS: C Methotrexate is an effective nonsurgical treatment option for a hemodynamically stable woman whose ectopic pregnancy is unruptured and measures less than 4 cm in diameter. Methotrexate is not indicated or recommended as a treatment option for a complete hydatidiform mole, for a missed abortion, or for abruptio placentae. DIF: Cognitive Level: Apply REF: p. 677 TOP: Nursing Process: Planning MSC: Client Needs: Physiologic Integrity

14. Women with mild gestational hypertension and mild preeclampsia can be safely managed at home with frequent maternal and fetal evaluation. Complete or partial bed rest is still frequently ordered by some providers. Which complication is rarely the result of prolonged bed rest? a. Thrombophlebitis b. Psychologic stress c. Fluid retention d. Cardiovascular deconditioning

ANS: C No evidence has been found that supports the practice of bed rest to improve pregnancy outcome. Fluid retention is not an adverse outcome of prolonged bed rest. The woman is more likely to experience diuresis with accompanying fluid and electrolyte imbalance and weight loss. Prolonged bed rest is known to increase the risk for thrombophlebitis. Psychologic stress is known to begin on the first day of bed rest and continue for the duration of the therapy. Therefore, restricted activity, rather than complete bed rest, is recommended. Cardiovascular deconditioning is a known complication of bed rest. DIF: Cognitive Level: Understand REF: p. 661 TOP: Nursing Process: Diagnosis MSC: Client Needs: Physiologic Integrity

10. The management of the pregnant client who has experienced a pregnancy loss depends on the type of miscarriage and the signs and symptoms. While planning care for a client who desires outpatient management after a first-trimester loss, what would the nurse expect the plan to include? a. Dilation and curettage (D&C) b. Dilation and evacuation (D&E) c. Misoprostol d. Ergot products

ANS: C Outpatient management of a first-trimester loss is safely accomplished by the intravaginal use of misoprostol for up to 2 days. If the bleeding is uncontrollable, vital signs are unstable, or signs of infection are present, then a surgical evacuation should be performed. D&C is a surgical procedure that requires dilation of the cervix and scraping of the uterine walls to remove the contents of pregnancy. This procedure is commonly performed to treat inevitable or incomplete abortion and should be performed in a hospital. D&E is usually performed after 16 weeks of pregnancy. The cervix is widely dilated, followed by removal of the contents of the uterus. Ergot products such as Methergine or Hemabate may be administered for excessive bleeding after miscarriage. DIF: Cognitive Level: Apply REF: p. 672 TOP: Nursing Process: Planning MSC: Client Needs: Physiologic Integrity

7. Prostaglandin gel has been ordered for a pregnant woman at 43 weeks of gestation. What is the primary purpose of prostaglandin administration? a. To enhance uteroplacental perfusion in an aging placenta b. To increase amniotic fluid volume c. To ripen the cervix in preparation for labor induction d. To stimulate the amniotic membranes to rupture

ANS: C Preparations of prostaglandin E1 and E2 are effective when used before labor induction to ripen (i.e., soften and thin) the cervix. Uteroplacental perfusion is not altered by the use of prostaglandins. The insertion of prostaglandin gel has no effect on the level of amniotic fluid. In some cases, women will spontaneously begin laboring after the administration of prostaglandins, thereby eliminating the need for oxytocin. It is not common for a woman's membranes to rupture as a result of prostaglandin use. DIF: Cognitive Level: Apply REF: p. 779 TOP: Nursing Process: Planning MSC: Client Needs: Physiologic Integrity

1. In planning for home care of a woman with preterm labor, which concern should the nurse need to address? a. Nursing assessments are different from those performed in the hospital setting. b. Restricted activity and medications are necessary to prevent a recurrence of preterm labor. c. Prolonged bed rest may cause negative physiologic effects. d. Home health care providers are necessary.

ANS: C Prolonged bed rest may cause adverse effects such as weight loss, loss of appetite, muscle wasting, weakness, bone demineralization, decreased cardiac output, risk for thrombophlebitis, alteration in bowel functions, sleep disturbance, and prolonged postpartum recovery. Nursing assessments differ somewhat from those performed in the acute care setting, but this concern does not need to be addressed. Restricted activity and medications may prevent preterm labor but not in all women. In addition, the plan of care is individualized to meet the needs of each client. Many women receive home health nurse visits, but care is individualized for each woman. DIF: Cognitive Level: Analyze REF: p. 777 TOP: Nursing Process: Planning MSC: Client Needs: Health Promotion and Maintenance

1. A primigravida is being monitored at the prenatal clinic for preeclampsia. Which finding is of greatest concern to the nurse? a. Blood pressure (BP) increase to 138/86 mm Hg b. Weight gain of 0.5 kg during the past 2 weeks c. Dipstick value of 3+ for protein in her urine d. Pitting pedal edema at the end of the day

ANS: C Proteinuria is defined as a concentration of 1+ or greater via dipstick measurement. A dipstick value of 3+ alerts the nurse that additional testing or assessment should be performed. A 24-hour urine collection is preferred over dipstick testing attributable to accuracy. Generally, hypertension is defined as a BP of 140/90 mm Hg or an increase in systolic pressure of 30 mm Hg or diastolic pressure of 15 mm Hg. Preeclampsia may be demonstrated as a rapid weight gain of more than 2 kg in 1 week. Edema occurs in many normal pregnancies, as well as in women with preeclampsia. Therefore, the presence of edema is no longer considered diagnostic of preeclampsia. DIF: Cognitive Level: Analyze REF: p. 660 TOP: Nursing Process: Diagnosis MSC: Client Needs: Physiologic Integrity

4. A pregnant woman has been receiving a magnesium sulfate infusion for treatment of severe preeclampsia for 24 hours. On assessment, the nurse finds the following vital signs: temperature 37.3° C, pulse rate 88 beats per minute, respiratory rate 10 breaths per minute, BP 148/90 mm Hg, absent deep tendon reflexes (DTRs), and no ankle clonus. The client complains, "I'm so thirsty and warm." What is the nurse's immediate action? a. To call for an immediate magnesium sulfate level b. To administer oxygen c. To discontinue the magnesium sulfate infusion d. To prepare to administer hydralazine

ANS: C Regardless of the magnesium level, the client is displaying the clinical signs and symptoms of magnesium toxicity. The first action by the nurse should be to discontinue the infusion of magnesium sulfate. In addition, calcium gluconate, the antidote for magnesium, may be administered. Hydralazine is an antihypertensive drug commonly used to treat hypertension in severe preeclampsia. Typically, hydralazine is administered for a systolic BP higher than 160 mm Hg or a diastolic BP higher than 110 mm Hg. DIF: Cognitive Level: Apply REF: p. 664 TOP: Nursing Process: Implementation MSC: Client Needs: Physiologic Integrity

17. The nurse is teaching a client with preterm premature rupture of membranes (PPROM) regarding self-care activities. Which activities should the nurse include in her teaching? a. Report a temperature higher than 40° C. b. Tampons are safe to use to absorb the leaking amniotic fluid. c. Do not engage in sexual activity. d. Taking frequent tub baths is safe.

ANS: C Sexual activity should be avoided because it may induce preterm labor. A temperature higher than 38° C should be reported. To prevent the risk of infection, tub baths should be avoided and nothing should be inserted into the vagina. Further, foul-smelling vaginal fluid, which may be a sign of infection, should be reported. DIF: Cognitive Level: Apply REF: p. 762 TOP: Nursing Process: Implementation MSC: Client Needs: Health Promotion and Maintenance

8. A woman who is 30 weeks of gestation arrives at the hospital with bleeding. Which differential diagnosis would not be applicable for this client? a. Placenta previa b. Abruptio placentae c. Spontaneous abortion d. Cord insertion

ANS: C Spontaneous abortion is another name for miscarriage; it occurs, by definition, early in pregnancy. Placenta previa is a well-known reason for bleeding late in pregnancy. The premature separation of the placenta (abruptio placentae) is a bleeding disorder that can occur late in pregnancy. Cord insertion may cause a bleeding disorder that can also occur late in pregnancy. DIF: Cognitive Level: Understand REF: p. 669 TOP: Nursing Process: Assessment MSC: Client Needs: Physiologic Integrity, Physiologic Adaptation

20. Which statement most accurately describes the HELLP syndrome? a. Mild form of preeclampsia b. Diagnosed by a nurse alert to its symptoms c. Characterized by hemolysis, elevated liver enzymes, and low platelets d. Associated with preterm labor but not perinatal mortality

ANS: C The acronym HELLP stands for hemolysis (H), elevated liver (EL) enzymes, and low platelets (LP). The HELLP syndrome is a variant of severe preeclampsia and is difficult to identify because the symptoms are not often obvious. The HELLP syndrome must be diagnosed in the laboratory. Preterm labor is greatly increased; therefore, so is perinatal mortality. DIF: Cognitive Level: Understand REF: p. 657 TOP: Nursing Process: Diagnosis | Nursing Process: Planning MSC: Client Needs: Physiologic Integrity

6.A 30-year-old gravida 3, para 2-0-0-2 is at 18 weeks of gestation. What screening test should be suggested to her? a. Biophysical profile b. Chorionic villi sampling c. Maternal serum alpha-fetoprotein (MSAFP) screening d. Screening for diabetes mellitus

ANS: C The biochemical assessment MSAFP test is performed from week 15 to week 20 of gestation (weeks 16 to 18 are ideal). A biophysical profile is a method of biophysical assessment of fetal well-being in the third trimester. Chorionic villi sampling is a biochemical assessment of the fetus that should be performed from the tenth to twelfth weeks of gestation. Screening for diabetes mellitus begins with the first prenatal visit.

5. A primigravida at 40 weeks of gestation is having uterine contractions every to 2 minutes and states that they are very painful. Her cervix is dilated 2 cm and has not changed in 3 hours. The woman is crying and wants an epidural. What is the likely status of this woman's labor? a. She is exhibiting hypotonic uterine dysfunction. b. She is experiencing a normal latent stage. c. She is exhibiting hypertonic uterine dysfunction. d. She is experiencing precipitous labor.

ANS: C The contraction pattern observed in this woman signifies hypertonic uterine activity. Typically, uterine activity in this phase occurs at 4- to 5-minute intervals lasting 30 to 45 seconds. Women who experience hypertonic uterine dysfunction, or primary dysfunctional labor, are often anxious first-time mothers who are having painful and frequent contractions that are ineffective at causing cervical dilation or effacement to progress. With hypotonic uterine dysfunction, the woman initially makes normal progress into the active stage of labor; then the contractions become weak and inefficient or stop altogether. Precipitous labor is one that lasts less than 3 hours from the onset of contractions until time of birth. DIF: Cognitive Level: Apply REF: p. 773 TOP: Nursing Process: Diagnosis MSC: Client Needs: Health Promotion and Maintenance

23. Which finding on a prenatal visit at 10 weeks of gestation might suggest a hydatidiform mole? a. Complaint of frequent mild nausea b. Blood pressure of 120/80 mm Hg c. Fundal height measurement of 18 cm d. History of bright red spotting for 1 day, weeks ago

ANS: C The uterus in a hydatidiform molar pregnancy is often larger than would be expected on the basis of the duration of the pregnancy. Nausea increases in a molar pregnancy because of the increased production of hCG. A woman with a molar pregnancy may have early-onset pregnancy-induced hypertension. In the client's history, bleeding is normally described as brownish. DIF: Cognitive Level: Analyze REF: p. 678 TOP: Nursing Process: Assessment MSC: Client Needs: Health Promotion and Maintenance

13. The American College of Obstetricians and Gynecologists (ACOG) has developed a comprehensive list of risk factors associated with the development of preeclampsia. Which client exhibits the greatest number of these risk factors? a. 30-year-old obese Caucasian with her third pregnancy b. 41-year-old Caucasian primigravida c. 19-year-old African American who is pregnant with twins d. 25-year-old Asian American whose pregnancy is the result of donor insemination

ANS: C Three risk factors are present in the 19-year-old African-American client. She has African-American ethnicity, is at the young end of the age distribution, and has a multiple pregnancy. In planning care for this client, the nurse must frequently monitor her BP and teach her to recognize the early warning signs of preeclampsia. The 30-year-old obese Caucasian client has only has one known risk factor: obesity. Age distribution appears to be U-shaped, with women younger than 20 years of age and women older than 40 years of age being at greatest risk. Preeclampsia continues to be more frequently observed in primigravidas; this client is a multigravida woman. Two risk factors are present for the 41-year-old Caucasian primigravida client. Her age and status as a primigravida place her at increased risk for preeclampsia. Caucasian women are at a lower risk than are African-American women. The 25-year-old Asian-American client exhibits only one risk factor. Pregnancies that result from donor insemination, oocyte donation, and embryo donation are at an increased risk of developing preeclampsia. DIF: Cognitive Level: Analyze REF: p. 655 TOP: Nursing Process: Planning MSC: Client Needs: Physiologic Integrity

12. Which statement related to cephalopelvic disproportion (CPD) is the least accurate? a. CPD can be related to either fetal size or fetal position. b. The fetus cannot be born vaginally. c. CPD can be accurately predicted. d. Causes of CPD may have maternal or fetal origins.

ANS: C Unfortunately, accurately predicting CPD is not possible. Although CPD is often related to excessive fetal size (macrosomia), malposition of the fetal presenting part is the problem in many cases, not true CPD. When CPD is present, the fetus cannot fit through the maternal pelvis to be born vaginally. CPD may be related to either fetal origins such as macrosomia or malposition or maternal origins such as a too small or malformed pelvis. DIF: Cognitive Level: Understand REF: p. 775 TOP: Nursing Process: Planning MSC: Client Needs: Health Promotion and Maintenance

6. A woman is having her first child. She has been in labor for 15 hours. A vaginal examination performed 2 hours earlier revealed the cervix to be dilated to 5 cm and 100% effaced, and the presenting part of the fetus was at station 0; however, another vaginal examination performed 5 minutes ago indicated no changes. What abnormal labor pattern is associated with this description? a. Prolonged latent phase b. Protracted active phase c. Secondary arrest d. Protracted descent

ANS: C With a secondary arrest of the active phase, the progress of labor has stopped. This client has not had any anticipated cervical change, indicating an arrest of labor. In the nulliparous woman, a prolonged latent phase typically lasts longer than 20 hours. A protracted active phase, the first or second stage of labor, is prolonged (slow dilation). With a protracted descent, the fetus fails to descend at an anticipated rate during the deceleration phase and second stage of labor. DIF: Cognitive Level: Analyze REF: p. 774 TOP: Nursing Process: Assessment MSC: Client Needs: Health Promotion and Maintenance

15. Which preexisting factor is known to increase the risk of GDM? a. Underweight before pregnancy b. Maternal age younger than 25 years c. Previous birth of large infant d. Previous diagnosis of type 2 diabetes mellitus

ANS: C A previous birth of a large infant suggests GDM. Obesity (body mass index [BMI] of 30 or greater) creates a higher risk for gestational diabetes. A woman younger than 25 years is not generally at risk for GDM. The person with type 2 diabetes mellitus already has diabetes and thus will continue to have it after pregnancy. Insulin may be required during pregnancy because oral hypoglycemia drugs are contraindicated during pregnancy.

2. The nurse is performing an initial assessment of a client in labor. What is the appropriate terminology for the relationship of the fetal body parts to one another? a. Lie b. Presentation c. Attitude d. Position

ANS: C Attitude is the relationship of the fetal body parts to one another. Lie is the relationship of the long axis (spine) of the fetus to the long axis (spine) of the mother. Presentation refers to the part of the fetus that enters the pelvic inlet first and leads through the birth canal during labor at term.Position is the relationship of the presenting part of the fetus to the four quadrants of the mother's pelvis. DIF: Cognitive Level: Remember REF: p. 369

2. During a prenatal visit, the nurse is explaining dietary management to a woman with pregestational diabetes. Which statement by the client reassures the nurse that teaching has been effective? a. "I will need to eat 600 more calories per day because I am pregnant." b. "I can continue with the same diet as before pregnancy as long as it is well balanced." c. "Diet and insulin needs change during pregnancy." d. "I will plan my diet based on the results of urine glucose testing."

ANS: C Diet and insulin needs change during the pregnancy in direct correlation to hormonal changes and energy needs. In the third trimester, insulin needs may double or even quadruple. The diet is individualized to allow for increased fetal and metabolic requirements, with consideration of such factors as prepregnancy weight and dietary habits, overall health, ethnic background, lifestyle, stage of pregnancy, knowledge of nutrition, and insulin therapy. Energy needs are usually calculated on the basis of 30 to 35 calories per kilogram of ideal body weight. Dietary management during a diabetic pregnancy must be based on blood, not urine, glucose changes.

23. Which nursing assessment indicates that a woman who is in second-stage labor is almost ready to give birth? a. Fetal head is felt at 0 station during the vaginal examination. b. Bloody mucous discharge increases. c. Vulva bulges and encircles the fetal head. d. Membranes rupture during a contraction.

ANS: C During the active pushing (descent) phase, the woman has strong urges to bear down as the presenting part of the fetus descends and presses on the stretch receptors of the pelvic floor. The vulva stretches and begins to bulge, encircling the fetal head. Birth of the head occurs when the station is +4. A 0 station indicates engagement. Bloody show occurs throughout the labor process and is not an indication of an imminent birth. Rupture of membranes can occur at any time during the labor process and does not indicate an imminent birth. DIF: Cognitive Level: Analyze REF: pp. 376-377

3. When assessing the fetus using Leopold's maneuvers, the nurse feels a round, firm, and movable fetal part in the fundal portion of the uterus and a long, smooth surface in the mother's right side close to midline. What is the position of the fetus? a. ROA b. LSP c. RSA d. LOA

ANS: C Fetal position is denoted with a three-letter abbreviation. The first letter indicates the presenting part in either the right or the left side of the maternal pelvis. The second letter indicates the anatomic presenting part of the fetus. The third letter stands for the location of the presenting part in relationship to the anterior, posterior, or transverse portion of the maternal pelvis. Palpation of a round, firm fetal part in the fundal portion of the uterus would be the fetal head, indicating that the fetus is in a breech position with the sacrum as the presenting part in the maternal pelvis. Palpation of the fetal spine along the mother's right side denotes the location of the presenting part in the mother's pelvis. The ability to palpate the fetal spine indicates that the fetus is anteriorly positioned in the maternal pelvis. This fetus is anteriorly positioned in the right side of the maternal pelvis with the sacrum as the presenting part. RSA is the correct three-letter abbreviation to indicate this fetal position. ROA denotes a fetus that is anteriorly positioned in the right side of the maternal pelvis with the occiput as the presenting part. LSP describes a fetus that is posteriorly positioned in the left side of the pelvis with the sacrum as the presenting part. A fetus that is LOA would be anteriorly positioned in the left side of the pelvis with the occiput as the presenting part. DIF: Cognitive Level: Apply REF: p. 370

12. A woman's position is an important component of the labor progress. Which guidance is important for the nurse to provide to the laboring client? a. The supine position, which is commonly used in the United States, increases blood flow. b. The laboring client positioned on her hands and knees ("all fours" position) is hard on the woman's back. c. Frequent changes in position help relieve fatigue and increase the comfort of the laboring client. d. In a sitting or squatting position, abdominal muscles of the laboring client will have to work harder.

ANS: C Frequent position changes relieve fatigue, increase comfort, and improve circulation. Blood flow can be compromised in the supine position; any upright position benefits cardiac output. The "all fours" position is used to relieve backache in certain situations. In a sitting or squatting position, the abdominal muscles work in greater harmony with uterine contractions. DIF: Cognitive Level: Apply REF: p. 375

17. To manage her diabetes appropriately and to ensure a good fetal outcome, how would the pregnant woman with diabetes alter her diet? a. Eat six small equal meals per day. b. Reduce the carbohydrates in her diet. c. Eat her meals and snacks on a fixed schedule. d. Increase her consumption of protein.

ANS: C Having a fixed meal schedule will provide the woman and the fetus with a steady blood sugar level, provide a good balance with insulin administration, and help prevent complications. Having a fixed meal schedule is more important than the equal division of food intake. Approximately 45% of the food eaten should be in the form of carbohydrates.

10. Which statement regarding the laboratory test for glycosylated hemoglobin Alc is correct? a. The laboratory test for glycosylated hemoglobin Alc is performed for all pregnant women, not only those with or likely to have diabetes. b. This laboratory test is a snapshot of glucose control at the moment. c. This laboratory test measures the levels of hemoglobin Alc, which should remain at less than 7%. d. This laboratory test is performed on the woman's urine, not her blood.

ANS: C Hemoglobin Alc levels greater than 7% indicate an elevated glucose level during the previous 4 to 6 weeks. This extra laboratory test is for diabetic women and defines glycemic control over the previous 4 to 6 weeks. Glycosylated hemoglobin level tests are performed on the blood.

8. A number of metabolic changes occur throughout pregnancy. Which physiologic adaptation of pregnancy will influence the nurse's plan of care? a. Insulin crosses the placenta to the fetus only in the first trimester, after which the fetus secretes its own. b. Women with insulin-dependent diabetes are prone to hyperglycemia during the first trimester because they are consuming more sugar. c. During the second and third trimesters, pregnancy exerts a diabetogenic effect that ensures an abundant supply of glucose for the fetus. d. Maternal insulin requirements steadily decline during pregnancy.

ANS: C Pregnant women develop increased insulin resistance during the second and third trimesters. Insulin never crosses the placenta; the fetus starts making its own around the 10th week. As a result of normal metabolic changes during pregnancy, insulin-dependent women are prone to hypoglycemia (low levels). Maternal insulin requirements may double or quadruple by the end of pregnancy.

4. Which statement by the client would lead the nurse to believe that labor has been established? a. "I passed some thick, pink mucus when I urinated this morning." b. "My bag of waters just broke." c. "The contractions in my uterus are getting stronger and closer together." d. "My baby dropped, and I have to urinate more frequently now."

ANS: C Regular, strong contractions with the presence of cervical change indicate that the woman is experiencing true labor. Although the loss of the mucous plug (operculum) often occurs during the first stage of labor or before the onset of labor, it is not the indicator of true labor. Spontaneous rupture of membranes often occurs during the first stage of labor; however, it is not an indicator of true labor. The presenting part of the fetus typically becomes engaged in the pelvis at the onset of labor but is not the indicator of true labor. DIF: Cognitive Level: Understand REF: p. 376

11. A labor and delivery nurse should be cognizant of which information regarding how the fetus moves through the birth canal? a. Fetal attitude describes the angle at which the fetus exits the uterus. b. Of the two primary fetal lies, the horizontal lie is that in which the long axis of the fetus is parallel to the long axis of the mother. c. Normal attitude of the fetus is called general flexion. d. Transverse lie is preferred for vaginal birth.

ANS: C The normal attitude of the fetus is called general flexion. The fetal attitude is the relationship of the fetal body parts to each one another. The horizontal lie is perpendicular to the mother; in the longitudinal (or vertical) lie, the long axes of the fetus and the mother are parallel. Vaginal birth cannot occur if the fetus stays in a transverse lie.

15. Nurses should be cognizant of what regarding the mechanism of labor? a. Seven critical movements must progress in a more or less orderly sequence. b. Asynclitism is sometimes achieved by means of the Leopold's maneuver. c. Effects of the forces determining descent are modified by the shape of the woman's pelvis and the size of the fetal head. d. At birth, the baby is said to achieve "restitution"; that is, a return to the C-shape of the womb.

ANS: C The size of the maternal pelvis and the ability of the fetal head to mold also affect the process. The seven identifiable movements of the mechanism of labor simultaneously occur in combinations, not in precise sequences. Asynclitism is the deflection of the baby's head; the Leopold's maneuver is a means of judging descent by palpating the mother's abdomen. Restitution is the rotation of the baby's head after the infant is born. DIF: Cognitive Level: Understand REF: p. 377

20. Which adaptation of the maternal-fetal exchange of oxygen occurs in response to uterine contraction? a. The maternal-fetal exchange of oxygen and waste products continues except when placental functions are reduced. b. This maternal-fetal exchange increases as the blood pressure decreases. c. It diminishes as the spiral arteries are compressed. d. This exchange of oxygen and waste products is not significantly affected by contractions.

ANS: C Uterine contractions during labor tend to decrease circulation through the spiral electrodes and subsequent perfusion through the intervillous space. The maternal blood supply to the placenta gradually stops with contractions. The exchange of oxygen and waste products decreases. The exchange of oxygen and waste products is affected by contractions. DIF: Cognitive Level: Understand REF: p. 379

2. Hypothyroidism occurs in 2 to 3 pregnancies per 1000. Because severe hypothyroidism is associated with infertility and miscarriage, it is not often seen in pregnancy. Regardless of this fact, the nurse should be aware of the characteristic symptoms of hypothyroidism. Which do they include? (Select all that apply.) a. Hot flashes b. Weight loss c. Lethargy d. Decrease in exercise capacity e. Cold intolerance

ANS: C, D, E Symptoms include weight gain, lethargy, decrease in exercise capacity, and intolerance to cold. Other presentations might include constipation, hoarseness, hair loss, and dry skin. Thyroid supplements are used to treat hyperthyroidism in pregnancy.

7. The nurse who elects to practice in the area of women's health must have a thorough understanding of miscarriage. Which statement regarding this condition is most accurate? a. A miscarriage is a natural pregnancy loss before labor begins. b. It occurs in fewer than 5% of all clinically recognized pregnancies. c. Careless maternal behavior, such as poor nutrition or excessive exercise, can be a factor in causing a miscarriage. d. If a miscarriage occurs before the 12th week of pregnancy, then it may be observed only as moderate discomfort and blood loss.

ANS: D Before the sixth week, the only evidence might be a heavy menstrual flow. After the 12th week, more severe pain, similar to that of labor, is likely. Miscarriage is a natural pregnancy loss, but it occurs, by definition, before 20 weeks of gestation, before the fetus is viable. Miscarriages occur in approximately 10% to 15% of all clinically recognized pregnancies. Miscarriages can be caused by a number of disorders or illnesses outside the mother's control or knowledge. DIF: Cognitive Level: Understand REF: p. 670 TOP: Nursing Process: Assessment MSC: Client Needs: Physiologic Integrity

18. A woman at 26 weeks of gestation is being assessed to determine whether she is experiencing preterm labor. Which finding indicates that preterm labor is occurring? a. Estriol is not found in maternal saliva. b. Irregular, mild uterine contractions are occurring every 12 to 15 minutes. c. Fetal fibronectin is present in vaginal secretions. d. The cervix is effacing and dilated to 2 cm.

ANS: D Cervical changes such as shortened endocervical length, effacement, and dilation are predictors of imminent preterm labor. Changes in the cervix accompanied by regular contractions indicate labor at any gestation. Estriol is a form of estrogen produced by the fetus that is present in plasma at 9 weeks of gestation. Levels of salivary estriol have been shown to increase before preterm birth. Irregular, mild contractions that do not cause cervical change are not considered a threat. The presence of fetal fibronectin in vaginal secretions between 24 and 36 weeks of gestation could predict preterm labor, but it has only a 20% to 40% positive predictive value. Of more importance are other physiologic clues of preterm labor such as cervical changes. DIF: Cognitive Level: Apply REF: p. 759 TOP: Nursing Process: Assessment | Nursing Process: Planning MSC: Client Needs: Health Promotion and Maintenance

16. The nurse has evaluated a client with preeclampsia by assessing DTRs. The result is a grade of 3+. Which DTR response most accurately describes this score? a. Sluggish or diminished b. Brisk, hyperactive, with intermittent or transient clonus c. Active or expected response d. More brisk than expected, slightly hyperactive

ANS: D DTRs reflect the balance between the cerebral cortex and the spinal cord. They are evaluated at baseline and to detect changes. A slightly hyperactive and brisk response indicates a grade 3+ response. DIF: Cognitive Level: Apply REF: p. 660 TOP: Nursing Process: Assessment MSC: Client Needs: Physiologic Integrity

11. The nurse is performing an assessment on a client who thinks she may be experiencing preterm labor. Which information is the most important for the nurse to understand and share with the client? a. Because all women must be considered at risk for preterm labor and prediction is so variable, teaching pregnant women the symptoms of preterm labor probably causes more harm through false alarms. b. Braxton Hicks contractions often signal the onset of preterm labor. c. Because preterm labor is likely to be the start of an extended labor, a woman with symptoms can wait several hours before contacting the primary caregiver. d. Diagnosis of preterm labor is based on gestational age, uterine activity, and progressive cervical change.

ANS: D Gestational age of 20 to 37 weeks, uterine contractions, and a cervix that is 80% effaced or dilated 2 cm indicates preterm labor. It is essential that nurses teach women how to detect the early symptoms of preterm labor. Braxton Hicks contractions resemble preterm labor contractions, but they are not true labor. Waiting too long to see a health care provider could result in essential medications failing to be administered. Preterm labor is not necessarily long-term labor. DIF: Cognitive Level: Understand REF: p. 759 TOP: Nursing Process: Planning MSC: Client Needs: Safe and Effective Care Environment

7. The client has been on magnesium sulfate for 20 hours for the treatment of preeclampsia. She just delivered a viable infant girl 30 minutes ago. What uterine findings does the nurse expect to observe or assess in this client? a. Absence of uterine bleeding in the postpartum period b. Fundus firm below the level of the umbilicus c. Scant lochia flow d. Boggy uterus with heavy lochia flow

ANS: D High serum levels of magnesium can cause a relaxation of smooth muscle such as the uterus. Because of this tocolytic effect, the client will most likely have a boggy uterus with increased amounts of bleeding. All women experience uterine bleeding in the postpartum period, especially those who have received magnesium therapy. Rather than scant lochial flow, however, this client will most likely have a heavy flow attributable to the relaxation of the uterine wall caused by magnesium administration. DIF: Cognitive Level: Analyze REF: p. 664 TOP: Nursing Process: Assessment MSC: Client Needs: Physiologic Integrity

3. A woman with preeclampsia has a seizure. What is the nurse's highest priority during a seizure? a. To insert an oral airway b. To suction the mouth to prevent aspiration c. To administer oxygen by mask d. To stay with the client and call for help

ANS: D If a client becomes eclamptic, then the nurse should stay with the client and call for help. Nursing actions during a convulsion are directed toward ensuring a patent airway and client safety. Insertion of an oral airway during seizure activity is no longer the standard of care. The nurse should attempt to keep the airway patent by turning the client's head to the side to prevent aspiration. Once the seizure has ended, it may be necessary to suction the client's mouth. Oxygen is administered after the convulsion has ended. DIF: Cognitive Level: Apply REF: p. 666 TOP: Nursing Process: Implementation MSC: Client Needs: Physiologic Integrity

13. Which statement related to the induction of labor is most accurate? a. Can be achieved by external and internal version techniques b. Is also known as a trial of labor (TOL) c. Is almost always performed for medical reasons d. Is rated for viability by a Bishop score

ANS: D Induction of labor is likely to be more successful with a Bishop score of 9 or higher for first-time mothers or 5 or higher for veterans. Version is the turning of the fetus to a better position by a physician for an easier or safer birth. A TOL is the observance of a woman and her fetus for several hours of active labor to assess the safety of vaginal birth. Two thirds of cases of induced labor are elective and not done for medical reasons. DIF: Cognitive Level: Understand REF: p. 780 TOP: Nursing Process: Diagnosis MSC: Client Needs: Safe and Effective Care Environment

22. When would an internal version be indicated to manipulate the fetus into a vertex position? a. Fetus from a breech to a cephalic presentation before labor begins b. Fetus from a transverse lie to a longitudinal lie before a cesarean birth c. Second twin from an oblique lie to a transverse lie before labor begins d. Second twin from a transverse lie to a breech presentation during a vaginal birth

ANS: D Internal version is used only during a vaginal birth to manipulate the second twin into a presentation that allows it to be vaginally born. For internal version to occur, the cervix needs to be completely dilated. DIF: Cognitive Level: Remember REF: p. 779 TOP: Nursing Process: Assessment MSC: Client Needs: Physiologic Integrity

With regard to maternal, fetal, and neonatal health problems, nurses should be aware that: a. Infection has replaced pulmonary embolism as one of the three top causes of maternal death attributable to pregnancy b. The leading cause of death in the neonatal period is disorders related to short gestation and low birth weight c. Factors related to the maternal death rate include age and marital status but not race d. Antepartum fetal deaths can best be prevented by better recognizing and responding to abnormalities of pregnancy and labor

ANS: D Medical teams need to be alert to signs of trouble. Race is a factor. African-American maternal mortality rates are more than three times higher than those for Caucasian women. Infection used to be an important cause of maternal death; it has been replaced by pulmonary embolism. The leading cause of death in the neonatal period is congenital anomalies. Race is a factor. African-American maternal mortality rates are more than three times higher than those for Caucasian women.

19. Which assessment is least likely to be associated with a breech presentation? a. Meconium-stained amniotic fluid b. Fetal heart tones heard at or above the maternal umbilicus c. Preterm labor and birth d. Postterm gestation

ANS: D Postterm gestation is not likely to occur with a breech presentation. The presence of meconium in a breech presentation may be a result of pressure on the fetal wall as it traverses the birth canal. Fetal heart tones heard at the level of the umbilical level of the mother are a typical finding in a breech presentation because the fetal back would be located in the upper abdominal area. Breech presentations often occur in preterm births. DIF: Cognitive Level: Analyze REF: pp. 775-776 TOP: Nursing Process: Assessment MSC: Client Needs: Health Promotion and Maintenance

24. A 32-year-old primigravida is admitted with a diagnosis of ectopic pregnancy. Which information assists the nurse in developing the plan of care? a. Bed rest and analgesics are the recommended treatment. b. She will be unable to conceive in the future. c. A D&C will be performed to remove the products of conception. d. Hemorrhage is the primary concern.

ANS: D Severe bleeding occurs if the fallopian tube ruptures. The recommended treatment is to remove the pregnancy before rupture to prevent hemorrhaging. If the tube must be removed, then the woman's fertility will decrease; however, she will not be infertile. A D&C is performed on the inside of the uterine cavity. The ectopic pregnancy is located within the tubes. DIF: Cognitive Level: Apply REF: p. 676 TOP: Nursing Process: Planning MSC: Client Needs: Physiologic Integrity

9. The obstetric provider has informed the nurse that she will be performing an amniotomy on the client to induce labor. What is the nurse's highest priority intervention after the amniotomy is performed? a. Applying clean linens under the woman b. Taking the client's vital signs c. Performing a vaginal examination d. Assessing the fetal heart rate (FHR)

ANS: D The FHR is assessed before and immediately after the amniotomy to detect any changes that might indicate cord compression or prolapse. Providing comfort measures, such as clean linens, for the client is important but not the priority immediately after an amniotomy. The woman's temperature should be checked every 2 hours after the rupture of membranes but not the priority immediately after an amniotomy. The woman would have had a vaginal examination during the procedure. Unless cord prolapse is suspected, another vaginal examination is not warranted. Additionally, FHR assessment provides clinical cues to a prolapsed cord. DIF: Cognitive Level: Analyze REF: p. 783 TOP: Nursing Process: Implementation MSC: Client Needs: Physiologic Integrity

3. In evaluating the effectiveness of magnesium sulfate for the treatment of preterm labor, which finding alerts the nurse to possible side effects? a. Urine output of 160 ml in 4 hours b. DTRs 2+ and no clonus c. Respiratory rate (RR) of 16 breaths per minute d. Serum magnesium level of 10 mg/dl

ANS: D The therapeutic range for magnesium sulfate management is 4 to 7.5 mg/dl. A serum magnesium level of 10 mg/dl could lead to signs and symptoms of magnesium toxicity, including oliguria and respiratory distress. Urine output of 160 ml in 4 hours, DTRs of 2+, and a RR of 16 breaths per minute are all normal findings. DIF: Cognitive Level: Apply REF: p. 767 TOP: Nursing Process: Evaluation MSC: Client Needs: Physiologic Integrity

5. A woman at 39 weeks of gestation with a history of preeclampsia is admitted to the labor and birth unit. She suddenly experiences increased contraction frequency of every 1 to 2 minutes, dark red vaginal bleeding, and a tense, painful abdomen. Which clinical change does the nurse anticipate? a. Eclamptic seizure b. Rupture of the uterus c. Placenta previa d. Abruptio placentae

ANS: D Uterine tenderness in the presence of increasing tone may be the earliest sign of abruptio placentae. Women with preeclampsia are at increased risk for an abruption attributable to decreased placental perfusion. Eclamptic seizures are evidenced by the presence of generalized tonic-clonic convulsions. Uterine rupture exhibits hypotonic uterine activity, signs of hypovolemia, and, in many cases, the absence of pain. Placenta previa exhibits bright red, painless vaginal bleeding. DIF: Cognitive Level: Understand REF: p. 662 TOP: Nursing Process: Diagnosis MSC: Client Needs: Physiologic Integrity

10. Which statement best describes chronic hypertension? a. Chronic hypertension is defined as hypertension that begins during pregnancy and lasts for the duration of the pregnancy. b. Chronic hypertension is considered severe when the systolic BP is higher than 140 mm Hg or the diastolic BP is higher than 90 mm Hg. c. Chronic hypertension is general hypertension plus proteinuria. d. Chronic hypertension can occur independently of or simultaneously with preeclampsia.

ANS: D Women with chronic hypertension may develop superimposed preeclampsia, which increases the morbidity for both the mother and the fetus. Chronic hypertension is present before pregnancy or diagnosed before the 20 weeks of gestation and persists longer than 6 weeks postpartum. Chronic hypertension becomes severe with a diastolic BP of 110 mm Hg or higher. Proteinuria is an excessive concentration of protein in the urine and is a complication of hypertension, not a defining characteristic. DIF: Cognitive Level: Understand REF: p. 667 TOP: Nursing Process: Diagnosis | Nursing Process: Planning MSC: Client Needs: Physiologic Integrity

13. A woman with gestational diabetes has had little or no experience reading and interpreting glucose levels. The client shows the nurse her readings for the past few days. Which reading signals the nurse that the client may require an adjustment of insulin or carbohydrates? a. 75 mg/dl before lunch. This is low; better eat now. b. 115 mg/dl 1 hour after lunch. This is a little high; maybe eat a little less next time. c. 115 mg/dl 2 hours after lunch. This is too high; it is time for insulin. d. 50 mg/dl just after waking up from a nap. This is too low; maybe eat a snack before going to sleep.

ANS: D 50 mg/dl after waking from a nap is too low. During hours of sleep, glucose levels should not be less than 60 mg/dl. Snacks before sleeping can be helpful. The premeal acceptable range is 60 to 99 mg/dl. The readings 1 hour after a meal should be less than 129 mg/dl. Two hours after eating, the readings should be less than 120 mg/dl.

13. Certain changes stimulate chemoreceptors in the aorta and carotid bodies to prepare the fetus for initiating respirations immediately after birth. Which change in fetal physiologic activity isnot part of this process? a. Fetal lung fluid is cleared from the air passages during labor and vaginal birth. b. Fetal partial pressure of oxygen (PO2) decreases. c. Fetal partial pressure of carbon dioxide in arterial blood (PaCO2) increases. d. Fetal respiratory movements increase during labor.

ANS: D Fetal respiratory movements actually decrease during labor. Fetal lung fluid is cleared from the air passages during labor and vaginal birth. Fetal PO2 decreases, and fetal PaCO2 increases. DIF: Cognitive Level: Understand REF: p. 379

16. Which physiologic alteration of pregnancy most significantly affects glucose metabolism? a. Pancreatic function in the islets of Langerhans is affected by pregnancy. b. Pregnant women use glucose at a more rapid rate than nonpregnant women. c. Pregnant women significantly increase their dietary intake. d. Placental hormones are antagonistic to insulin, thus resulting in insulin resistance.

ANS: D Placental hormones, estrogen, progesterone, and human placental lactogen (HPL) create insulin resistance. Insulin is also broken down more quickly by the enzyme placental insulinase. Pancreatic functioning is not affected by pregnancy. The glucose requirements differ because of the growing fetus. The pregnant woman should increase her intake by 200 calories a day.

19. Which statement regarding the care of a client in labor is correct and important to the nurse as he or she formulates the plan of care? a. The woman's blood pressure will increase during contractions and fall back to prelabor normal levels between contractions. b. The use of the Valsalva maneuver is encouraged during the second stage of labor to relieve fetal hypoxia. c. Having the woman point her toes will reduce leg cramps. d. Endogenous endorphins released during labor will raise the woman's pain threshold and produce sedation.

ANS: D The endogenous endorphins released during labor will raise the woman's pain threshold and produce sedation. In addition, physiologic anesthesia of the perineal tissues, caused by the pressure of the presenting part, decreases the mother's perception of pain. Blood pressure levels increase during contractions but remain somewhat elevated between them. The use of the Valsalva maneuver is discouraged during the second stage labor because of a number of unhealthy outcomes, including fetal hypoxia. Pointing the toes can cause leg cramps, as can the process of labor itself. DIF: Cognitive Level: Understand REF: p. 380

1. A new mother asks the nurse when the "soft spot" on her son's head will go away. What is the nurse's best response, based upon her understanding of when the anterior frontal closes? a. 2 months b. 8 months c. 12 months d. 18 months

ANS: D The larger of the two fontanels, the anterior fontanel, closes by 18 months after birth. The posterior fontanel closes at 6 to 8 weeks. The remaining three options are too early for the anterior fontanel to close. DIF: Cognitive Level: Understand REF: p. 367

6. A pregnant woman is at 38 weeks of gestation. She wants to know whether there are any signs that "labor is getting close to starting." Which finding is an indication that labor may begin soon? a. Weight gain of 1.5 to 2 kg (3 to 4 lb) b. Increase in fundal height c. Urinary retention d. Surge of energy

ANS: D Women speak of having a burst of energy before labor. The woman may lose 0.5 to 1.5 kg, as a result of water loss caused by electrolyte shifts that, in turn, are caused by changes in the estrogen and progesterone levels. When the fetus descends into the true pelvis (called lightening), the fundal height may decrease. Urinary frequency may return before labor. DIF: Cognitive Level: Understand REF: p. 376 TOP: Nursing Process: Planning

17. Which basic type of pelvis includes the correct description and percentage of occurrence in women? a. Gynecoid: classic female pelvis; heart shaped; 75% b. Android: resembling the male pelvis; wide oval; 15% c. Anthropoid: resembling the pelvis of the ape; narrow; 10% d. Platypelloid: flattened, wide, and shallow pelvis; 3%

ANS: D A platypelloid pelvis is flattened, wide, and shallow; approximately 3% of women have this shape. The gynecoid pelvis is the classic female shape, slightly ovoid and rounded; approximately 50% of women have this shape. An android or malelike pelvis is heart shaped; approximately 23% of women have this shape. An anthropoid or apelike pelvis is oval and wide; approximately 24% of women have this shape. DIF: Cognitive Level: Remember REF: p. 374

Which assessment is not included in the fetal biophysical profile (BPP)? a. Fetal movement b. Fetal tone c. Fetal heart rate d. Amniotic fluid index e. Placental grade

ANS: E Fetal movement, tone, heart rate, and amniotic fluid index are all assessed in a BPP. The placental grade is determined by ultrasound (as is a BPP), but it is not included in the criteria of assessment factors for a BPP.

What it essential for early detection of htn disorders in a pregnant woman?

Accurate measure of BP

The nurse providing care for the laboring woman should understand that variable FHR decelerations are caused by: A. Altered fetal cerebral blood flow B. Umbilical cord compression C. Uteroplacental insufficiency D. Fetal hypoxemia

B A. Incorrect: Altered fetal cerebral blood flow would result in early decelerations in the FHR. B. Correct: Variable decelerations can occur any time during the uterine contracting phase and are caused by compression of the umbilical cord. C. Incorrect: Uteroplacental insufficiency would result in late decelerations in the FHR. D. Incorrect: Fetal hypoxemia would result in tachycardia initially, then bradycardia if hypoxia continues. p. 507

During labor a fetus with an average heart rate of 135 beats/min over a 10-minute period would be considered to have: A. Bradycardia B. A normal baseline heart rate C. Tachycardia D. Hypoxia

B A. Incorrect: Bradycardia is an FHR below 110 beats/min for 10 minutes or longer. B. Correct: The baseline heart rate is measured over 10 minutes; a normal range is 110 to 160 beats/min. C. Incorrect: Tachycardia is an FHR over 160 beats/min for 10 minutes or longer. D. Incorrect: Hypoxia is an inadequate supply of oxygen; no indication of this condition exists with a baseline heart rate in the normal range. p. 502

What correctly matches the type of deceleration with its likely cause? A. Early deceleration—umbilical cord compression B. Late deceleration—uteroplacental inefficiency C. Variable deceleration—head compression D. Prolonged deceleration—cause unknown

B A. Incorrect: Early deceleration is caused by head compression. B. Correct: Late deceleration is caused by uteroplacental inefficiency. C. Incorrect: Variable deceleration is caused by umbilical cord compression. D. Incorrect: Prolonged deceleration has a variety of either benign or critical causes. p. 507

What three measures should the nurse implement to provide intrauterine resuscitation? Select the best response that indicates the priority of actions that should be taken, starting with the most important. A. Call the provider, reposition the mother, and perform a vaginal exam B. Reposition the mother, increase IV fluid, and provide oxygen via face mask C. Administer oxygen to the mother, increase IV fluid, and notify the care provider D. Perform a vaginal exam, reposition the mother, and provide oxygen via face mask

B A. Incorrect: The nurse should initiate intrauterine resuscitation in an ABC manner, similar to basic life support. The first priority is to open the maternal and fetal vascular systems by repositioning the mother for improved perfusion. The second priority is to increase blood volume by increasing the IV fluid. The third priority is to optimize oxygenation of the circulatory volume by providing oxygen via face mask. If these interventions do not resolve the fetal heart rate issue quickly, the primary provider should be notified immediately. B. Correct: These are the correct nursing actions for intrauterine resuscitation. C. Incorrect: The nurse should initiate intrauterine resuscitation in an ABC manner, similar to basic life support. The first priority is to open the maternal and fetal vascular systems by repositioning the mother for improved perfusion. The second priority is to increase blood volume by increasing the IV fluid. The third priority is to optimize oxygenation of the circulatory volume by providing oxygen via face mask. If these interventions do not resolve the fetal heart rate issue quickly, the primary provider should be notified immediately. D. Incorrect: The nurse should initiate intrauterine resuscitation in an ABC manner, similar to basic life support. The first priority is to open the maternal and fetal vascular systems by repositioning the mother for improved perfusion. The second priority is to increase blood volume by increasing the IV fluid. The third priority is to optimize oxygenation of the circulatory volume by providing oxygen via face mask. If these interventions do not resolve the fetal heart rate issue quickly, the primary provider should be notified immediately. p. 513

A new client and her partner arrive on the labor, delivery, recovery, and postpartum (LDRP) unit for the birth of their first child. You apply the EFM to the woman. Her partner asks you to explain what is printing on the graph, referring to the EFM strip. He wants to know what the baby's heart rate should be. Your best response is: A. "Don't worry about that machine; that's my job." B. "The top line graphs the baby's heart rate. Generally, the heart rate is between 110 and 160. The heart rate will fluctuate in response to what is happening during labor." C. "The top line graphs the baby's heart rate, and the bottom line lets me know how strong the contractions are." D. "Your doctor will explain all of that later."

B A. Incorrect: This discredits the partner's feelings and does not provide the teaching he is requesting. B. Correct: This statement educates the partner about fetal monitoring and provides support and information to alleviate his fears. C. Incorrect: This statement provides inaccurate information and does not address the partner's concerns about the fetal heart rate. The EFM graphs the frequency and duration of the contractions, not the intensity. D. Incorrect: Nurses should take every opportunity to provide client and family teaching, especially when information is requested. pp. 501-502

27. What is the correct terminology for the nerve block that provides anesthesia to the lower vagina and perineum? a. Epidural b. Pudendal c. Local d. Spinal block

B (A pudendal block anesthetizes the lower vagina and perineum to provide anesthesia for an episiotomy and the use of low forceps, if needed. An epidural provides anesthesia for the uterus, perineum, and legs. A local provides anesthesia for the perineum at the site of the episiotomy. A spinal block provides anesthesia for the uterus, perineum, and down the legs.)

13. Anxiety is commonly associated with pain during labor. Which statement regarding anxiety is correct? a. Even mild anxiety must be treated. b. Severe anxiety increases tension, increases pain, and then, in turn, increases fear and anxiety, and so on. c. Anxiety may increase the perception of pain, but it does not affect the mechanism of labor. d. Women who have had a painful labor will have learned from the experience and have less anxiety the second time because of increased familiarity.

B (Anxiety and pain reinforce each other in a negative cycle that will slow the progress of labor. Mild anxiety is normal for a woman in labor and likely needs no special treatment other than the standard reassurances. Anxiety increases muscle tension and ultimately can sufficiently build to slow the progress of labor. Unfortunately, an anxious, painful first labor is likely to carry over, through expectations and memories, into an anxious and painful experience in the second pregnancy.)

26. Developing a realistic birth plan with the pregnant woman regarding her care is important for the nurse. How would the nurse explain the major advantage of nonpharmacologic pain management? a. Greater and more complete pain relief is possible. b. No side effects or risks to the fetus are involved. c. The woman will remain fully alert at all times. d. Labor will likely be more rapid.

B (Because nonpharmacologic pain management does not include analgesics, adjunct drugs, or anesthesia, it is harmless to the mother and the fetus. However, pain relief is lessened with nonpharmacologic pain management during childbirth. Although the womans alertness is not altered by medication, the increase in pain may decrease alertness. Pain management may or may not alter the length of labor. At times when pain is decreased, the mother relaxes and labor progresses at a quicker pace.)

8. What is the role of the nurse as it applies to informed consent? a. Inform the client about the procedure, and ask her to sign the consent form. b. Act as a client advocate, and help clarify the procedure and the options. c. Call the physician to see the client. d. Witness the signing of the consent form.

B (Nurses play a part in the informed consent process by clarifying and describing procedures or by acting as the womans advocate and asking the primary health care provider for further explanations. The physician is responsible for informing the woman of her options, explaining the procedure, and advising the client about potential risk factors. The physician must be present to explain the procedure to the client. However, the nurses responsibilities go further than simply asking the physician to see the client. The nurse may witness the signing of the consent form. However, depending on the states guidelines, the womans husband or another hospital health care employee may sign as a witness.)

18. According to professional standards (the Association of Womens Health, Obstetric and Neonatal Nurses [AWHONN], 2007), which action cannot be performed by the nonanesthetist registered nurse who is caring for a woman with epidural anesthesia? a. Monitoring the status of the woman and fetus b. Initiating epidural anesthesia c. Replacing empty infusion bags with the same medication and concentrate d. Stopping the infusion, and initiating emergency measures

B (Only qualified, licensed anesthesia care providers are permitted to insert a catheter, initiate epidural anesthesia, verify catheter placement, inject medication through the catheter, or alter the medication or medications including type, amount, or rate of infusion. The nonanesthetist nurse is permitted to monitor the status of the woman, the fetus, and the progress of labor. Replacement of the empty infusion bags or syringes with the same medication and concentration is permitted. If the need arises, the nurse may stop the infusion, initiate emergency measures, and remove the catheter if properly educated to do so. Complications can require immediate interventions. Nurses must be prepared to provide safe and effective care during an emergency situation.)

6. What should the laboring client who receives an opioid antagonist be told to expect? a. Her pain will decrease. b. Her pain will return. c. She will feel less anxious. d. She will no longer feel the urge to push.

B (Opioid antagonists such as naloxone (Narcan) promptly reverse the CNS-depressant effects of opioids. In addition, the antagonist counters the effect of the stress-induced levels of endorphins. An opioid antagonist is especially valuable if the labor is more rapid than expected and the birth is anticipated when the opioid is at its peak effect. The woman should be told that the pain that was relieved by the opioid analgesic will return with the administration of the opioid antagonist. Her pain level will increase rather than decrease. Opioid antagonists have no effect on anxiety levels. They are primarily administered to reverse the excessive CNS depression in the mother, newborn, or both. An opioid antagonist (e.g., naloxone) has no effect on the mothers urge or ability to push. The practice of giving lower doses of IV opioids has reduced the incidence and severity of opioid-induced CNS depression; therefore, opioid antagonists are used less frequently.)

9. A first-time mother is concerned about the type of medications she will receive during labor. The client is in a fair amount of pain and is nauseated. In addition, she appears to be very anxious. The nurse explains that opioid analgesics are often used along with sedatives. How should the nurse phrase the rationale for this medication combination? a. The two medications, together, reduce complications. b. Sedatives enhance the effect of the pain medication. c. The two medications work better together, enabling you to sleep until you have the baby. d. This is what your physician has ordered for you.

B (Sedatives may be used to reduce the nausea and vomiting that often accompany opioid use. In addition, some ataractic drugs reduce anxiety and apprehension and potentiate the opioid analgesic affects. A potentiator may cause two drugs to work together more effectively, but it does not ensure zero maternal or fetal complications. Sedation may be a related effect of some ataractic drugs; however, sedation is not the goal. Furthermore, a woman is unlikely to be able to sleep through transitional labor and birth. Although the physician may have ordered the medication, This is what your physician has ordered for you is not an acceptable comment for the nurse to make.)

25. The nurse should be aware of what important information regarding systemic analgesics administered during labor? a. Systemic analgesics cross the maternal blood-brain barrier as easily as they do the fetal blood-brain barrier. b. Effects on the fetus and newborn can include decreased alertness and delayed sucking. c. Intramuscular (IM) administration is preferred over IV administration. d. IV patient-controlled analgesia (PCA) results in increased use of an analgesic.

B (The effects of analgesics depend on the specific drug administered, the dosage, and the timing. Systemic analgesics cross the fetal blood-brain barrier more readily than the maternal blood-brain barrier. IV administration is preferred over IM administration because the drug acts faster and more predictably. PCA results in a decrease in the use of an analgesic.)

29. What is the rationale for the use of a blood patch after spinal anesthesia? a. Hypotension b. Headache c. Neonatal respiratory depression d. Loss of movement

B (The subarachnoid block may cause a postspinal headache resulting from the loss of cerebrospinal fluid from the puncture in the dura. When blood is injected into the epidural space in the area of the dural puncture, it forms a seal over the hole to stop the leaking of cerebrospinal fluid. Hypotension is prevented by increasing fluid volume before the procedure. Neonatal respiratory depression is not an expected outcome with spinal anesthesia. Loss of movement is an expected outcome of spinal anesthesia.)

1. Maternal hypotension is a potential side effect of regional anesthesia and analgesia. What nursing interventions could the nurse use to increase the clients blood pressure? (Select all that apply.) a. Place the woman in a supine position. b. Place the woman in a lateral position. c. Increase IV fluids. d. Administer oxygen. e. Perform a vaginal examination.

B, C, D (Nursing interventions for maternal hypotension arising from analgesia or anesthesia include turning the woman to a lateral position, increasing IV fluids, administering oxygen via face mask, elevating the womans legs, notifying the physician, administering an IV vasopressor, and monitoring the maternal and fetal status at least every 5 minutes until the woman is stable. Placing the client in a supine position causes venous compression, thereby limiting blood flow to and oxygenation of the placenta and fetus. A sterile vaginal examination has no bearing on maternal blood pressure.)

The nurse providing care for a woman with preterm labor who is receiving terbutaline would include which intervention to identify side effects of the drug? A.) Assessing deep tendon reflexes (DTRs) B.) Assessing for chest discomfort and palpitations C.) Assessing for bradycardia D.) Assessing for hypoglycemia

B.) Assessing for chest discomfort and palpitations

A 41-week pregnant multigravida presents in the labor and delivery unit after a nonstress test indicated that her fetus could be experiencing some difficulties in utero. Which diagnostic tool would yield more detailed information about the fetus? A.) Ultrasound for fetal anomalies B.) Biophysical profile (BPP) C.) Maternal serum alpha-fetoprotein (MSAFP) screening D.) Percutaneous umbilical blood sampling (PUBS)

B.) Biophysical profile (BPP)

The most prevalent clinical manifestation of abruptio placentae (as opposed to placenta previa) is: A.) bleeding. B.) intense abdominal pain. C.) uterine activity. D.) cramping.

B.) intense abdominal pain.

The slight overlapping of cranial bones or shaping of the fetal head during labor is called: A.) lightening. B.) molding. C.) Ferguson reflex. D.) Valsalva maneuver.

B.) molding

The nurse expects to administer an oxytocic (e.g., Pitocin, Methergine) to a woman after expulsion of her placenta to: A.) relieve pain. B.) stimulate uterine contraction. C.) prevent infection. D.) facilitate rest and relaxation.

B.) stimulate uterine contraction.

The nurse providing care for the laboring woman should understand that late FHR decelerations are caused by: A. Altered cerebral blood flow B. Umbilical cord compression C. Uteroplacental insufficiency D. Meconium fluid

C A. Incorrect: Altered fetal cerebral blood flow would result in early decelerations in the FHR. B. Incorrect: Umbilical cord compression would result in variable decelerations in the FHR. C. Correct: Uteroplacental insufficiency would result in late decelerations in the FHR. D. Incorrect: Meconium-stained fluid may or may not produce changes in the fetal heart rate, depending on the gestational age of the fetus and whether other causative factors associated with fetal distress are present. p. 507

According to standard professional thinking, nurses should auscultate the FHR: A. Every 15 minutes in the active phase of the first stage of labor in the absence of risk factors B. Every 20 minutes in the second stage regardless of whether risk factors are present C. Before and after ambulation and rupture of membranes D. More often in a woman's first pregnancy

C A. Incorrect: In the active phase of the first stage of labor, the FHR should be auscultated every 30 minutes if no risk factors are involved; with risk factors, it should be auscultated every 15 minutes. B. Incorrect: In the second stage of labor, the FHR should be auscultated every 15 minutes if no risk factors are involved; with risk factors, it should be auscultated every 5 minutes. C. Correct: The FHR should be auscultated before and after administration of medications and induction of anesthesia. D. Incorrect: The fetus of a first-time mother is automatically at greater risk. p. 499

The nurse caring for the woman in labor should understand that maternal hypotension can result in: A. Early decelerations B. Fetal dysrhythmias C. Uteroplacental insufficiency D. Spontaneous rupture of membranes

C A. Incorrect: Maternal hypotension is not associated with this condition. B. Incorrect: Maternal hypotension is not associated with this condition. C. Correct: Low maternal blood pressure reduces placental blood flow during uterine contractions, resulting in fetal hypoxemia. D. Incorrect: Maternal hypotension is not associated with this condition. p. 503

The nurse caring for the woman in labor should understand that increased variability of the fetal heart rate might be caused by: A. Narcotics B. Barbiturates C. Methamphetamines D. Tranquilizers

C A. Incorrect: Narcotics, barbiturates, and tranquilizers might be causes of decreased variability; methamphetamines might cause increased variability. B. Incorrect: Narcotics, barbiturates, and tranquilizers might be causes of decreased variability; methamphetamines might cause increased variability. C. Correct: Narcotics, barbiturates, and tranquilizers might be causes of decreased variability; methamphetamines might cause increased variability. D. Incorrect: Narcotics, barbiturates, and tranquilizers might be causes of decreased variability; methamphetamines might cause increased variability. p. 503-504

The nurse caring for a woman in labor understands that prolonged decelerations: A. Are a continuing pattern of benign decelerations that do not require intervention B. Constitute a baseline change when they last longer than 5 minutes C. Usually are isolated events that end spontaneously D. Require the usual fetal monitoring by the nurse

C A. Incorrect: Prolonged decelerations usually are isolated events that end spontaneously. However, in certain combinations with late and/or variable decelerations, they are a danger sign that requires the nurse to notify the physician or midwife immediately. B. Incorrect: A deceleration that lasts longer than 10 minutes constitutes a baseline change. Prolonged decelerations usually are isolated events that end spontaneously. However, in certain combinations with late and/or variable decelerations, they are a danger sign that requires the nurse to notify the physician or midwife immediately. C. Correct: Prolonged decelerations usually are isolated events that end spontaneously. However, in certain combinations with late and/or variable decelerations, they are a danger sign that requires the nurse to notify the physician or midwife immediately. D. Incorrect: Prolonged decelerations usually are isolated events that end spontaneously. However, in certain combinations with late and/or variable decelerations, they are a danger sign that requires the nurse to notify the physician or midwife immediately. p. 509

A number of methods to assist in the assessment of fetal well-being have been developed for use in conjunction with electronic fetal monitoring (EFM). These various technologies assist in supporting interventions for a nonreassuring fetal heart rate pattern when necessary. The labor and delivery nurse should be aware that one of these modalities, fetal oxygen saturation monitoring, includes the use of: A. A fetal acoustic stimulator B. Fetal blood sampling C. Fetal pulse oximetry D. Umbilical cord acid-base determination

C A. Incorrect: Stimulation of the fetus is done in an effort to elicit a fetal heart rate response. The two acceptable methods of achieving this result are the use of fetal scalp stimulation or vibroacoustic stimulation. Vibroacoustic stimulation is performed by using an artificial larynx or fetal acoustic stimulation device over the fetal head for 1 or 2 seconds. B. Incorrect: Sampling of the fetal scalp blood was designed to assess fetal pH, O2, and CO2. The sample is obtained from the fetal scalp through a dilated cervix. This test is usually done in tertiary care centers, where results can be immediately available. It has recently fallen out of favor because test results vary widely. C. Correct: Continuous monitoring of the fetal O2 saturation by fetal pulse oximetry is a method that was approved for clinical use in 2000 by the FDA. This process works in a similar method to obtaining a pulse oximetry in a child or adult. A specially designed sensor is inserted into the uterus and lies against the fetus's temple or cheek. A normal result is 30% to 70%, with 30% being the cutoff for further intervention. D. Incorrect: This test is not completed until after birth. Cord blood is drawn from the umbilical artery and tested for pH, O2, and CO2. pp. 513-516

What is an advantage of external electronic fetal monitoring? A. The ultrasound transducer can accurately measure short-term variability and beat-to-beat changes in the FHR. B. The tocotransducer can measure and record the frequency, regularity, intensity, and approximate duration of UCs. C. The tocotransducer is especially valuable for measuring UA during the first stage of labor. D. Once correctly applied by the nurse, the transducer need not be repositioned even when the woman changes positions.

C A. Incorrect: These short-term changes cannot be measured with this technology. B. Incorrect: The tocotransducer cannot measure and record the intensity of UCs. C. Correct: This is especially true when the membranes are intact. D. Incorrect: The transducer must be repositioned when the woman or the fetus changes position. p. 500

Fetal bradycardia is most common during: A. Intraamniotic infection B. Fetal anemia C. Prolonged umbilical cord compression D. Tocolytic treatment using ritodrine

C A. Incorrect: This circumstance most likely would result in fetal tachycardia. B. Incorrect: This circumstance most likely would result in fetal tachycardia. C. Correct: Fetal bradycardia can be considered a later sign of fetal hypoxia and is known to occur before fetal death. Bradycardia can result from placental transfer of drugs, prolonged compression of the umbilical cord, maternal hypothermia, and maternal hypotension. D. Incorrect: This circumstance most likely would result in fetal tachycardia. p. 503

17. A woman in labor is breathing into a mouthpiece just before the start of her regular contractions. As she inhales, a valve opens and gas is released. She continues to inhale the gas slowly and deeply until the contraction starts to subside. When the inhalation stops, the valve closes. Which statement regarding this procedure is correct? a. The application of nitrous oxide gas is not often used anymore. b. An inhalation of gas is likely to be used in the second stage of labor, not during the first stage. c. An application of nitrous oxide gas is administered for pain relief. d. The application of gas is a prelude to a cesarean birth.

C (A mixture of nitrous oxide with oxygen in a low concentration can be used in combination with other nonpharmacologic and pharmacologic measures for pain relief. This procedure is still commonly used in Canada and in the United Kingdom. Nitrous oxide inhaled in a low concentration will reduce but not eliminate pain during the first and second stages of labor. Nitrous oxide inhalation is not generally used before a caesarean birth. Nitrous oxide does not appear to depress uterine contractions or cause adverse reactions in the newborn.)

14. Which statement is not an expected outcome for the client who attends a reputable childbirth preparation program? a. Childbirth preparation programs increase the womans sense of control. b. Childbirth preparation programs prepare a support person to help during labor. c. Childbirth preparation programs guarantee a pain-free childbirth. d. Childbirth preparation programs teach distraction techniques.

C (All methods try to increase a womans sense of control, prepare a support person, and train the woman in physical conditioning, which includes breathing techniques. These programs cannot, and reputable ones do not, promise a pain-free childbirth. Increasing a womans sense of control is the goal of all childbirth preparation methods. Preparing a support person to help in labor is a vitally important component of any childbirth education program. The coach may learn how to touch a womans body to detect tense and contracted muscles. The woman then learns how to relax in response to the gentle stroking by the coach. Distraction techniques are a form of care that are effective to some degree in relieving labor pain and are taught in many childbirth programs. These distractions include imagery, feedback relaxation, and attention-focusing behaviors.)

5. A laboring woman has received meperidine (Demerol) intravenously (IV), 90 minutes before giving birth. Which medication should be available to reduce the postnatal effects of meperidine on the neonate? a. Fentanyl (Sublimaze) b. Promethazine (Phenergan) c. Naloxone (Narcan) d. Nalbuphine (Nubain)

C (An opioid antagonist can be given to the newborn as one part of the treatment for neonatal narcosis, which is a state of central nervous system (CNS) depression in the newborn produced by an opioid. Opioid antagonists, such as naloxone (Narcan), can promptly reverse the CNS depressant effects, especially respiratory depression. Fentanyl (Sublimaze), promethazine (Phenergan), and nalbuphine (Nubain) do not act as opioid antagonists to reduce the postnatal effects of meperidine on the neonate.)

12. Nurses with an understanding of cultural differences regarding likely reactions to pain may be better able to help their clients. Which clients may initially appear very stoic but then become quite vocal as labor progresses until late in labor, when they become more vocal and request pain relief? a. Chinese b. Arab or Middle Eastern c. Hispanic d. African-American

C (Hispanic women may be stoic early in labor but more vocal and ready for medications later. Chinese women may not show reactions to pain. Medical interventions must be offered more than once. Arab or Middle Eastern women may be vocal in response to labor pain from the start; they may prefer pain medications. African-American women may openly express pain; the use of medications for pain is more likely to vary with the individual.)

1. Which statement by the client will assist the nurse in determining whether she is in true labor as opposed to false labor? a. I passed some thick, pink mucus when I urinated this morning. b. My bag of waters just broke. c. The contractions in my uterus are getting stronger and closer together. d. My baby dropped, and I have to urinate more frequently now.

C (Regular, strong contractions with the presence of cervical change indicate that the woman is experiencing true labor. The loss of the mucous plug (operculum) often occurs during the first stage of labor or before the onset of labor, but it is not the indicator of true labor. Spontaneous rupture of membranes (ROM) often occurs during the first stage of labor, but it is not the indicator of true labor. The presenting part of the fetus typically becomes engaged in the pelvis at the onset of labor, but this is not the indicator of true labor.)

20. A woman in labor has just received an epidural block. What is the most important nursing intervention at this time? a. Limit parenteral fluids. b. Monitor the fetus for possible tachycardia. c. Monitor the maternal blood pressure for possible hypotension. d. Monitor the maternal pulse for possible bradycardia.

C (The most important nursing intervention for a woman who has received an epidural block is for the nurse to monitor the maternal blood pressure frequently for signs of hypotension. IV fluids are increased for a woman receiving an epidural to prevent hypotension. The nurse also observes for signs of fetal bradycardia and monitors for signs of maternal tachycardia, secondary to hypotension.)

4. The uterine contractions of a woman early in the active phase of labor are assessed by an internal uterine pressure catheter (IUPC). The uterine contractions occur every 3 to 4 minutes and last an average of 55 to 60 seconds. They are becoming more regular and are moderate to strong. Based on this information, what would a prudent nurse do next? a. Immediately notify the womans primary health care provider. b. Prepare to administer an oxytocic to stimulate uterine activity. c. Document the findings because they reflect the expected contraction pattern for the active phase of labor. d. Prepare the woman for the onset of the second stage of labor.

C (The nurse is responsible for monitoring the uterine contractions to ascertain whether they are powerful and frequent enough to accomplish the work of expelling the fetus and the placenta. In addition, the nurse documents these findings in the clients medical record. This labor pattern indicates that the client is in the active phase of the first stage of labor. Nothing indicates a need to notify the primary health care provider at this time. Oxytocin augmentation is not needed for this labor pattern; this contraction pattern indicates that the woman is in active labor. Her contractions will eventually become stronger, last longer, and come closer together during the transition phase of the first stage of labor. The transition phase precedes the second stage of labor, or delivery of the fetus.)

5. Which action is correct when palpation is used to assess the characteristics and pattern of uterine contractions? a. Placing the hand on the abdomen below the umbilicus and palpating uterine tone with the fingertips b. Determining the frequency by timing from the end of one contraction to the end of the next contraction c. Evaluating the intensity by pressing the fingertips into the uterine fundus d. Assessing uterine contractions every 30 minutes throughout the first stage of labor

C (The nurse or primary health care provider may assess uterine activity by palpating the fundal section of the uterus using the fingertips. Many women may experience labor pain in the lower segment of the uterus, which may be unrelated to the firmness of the contraction detectable in the uterine fundus. The frequency of uterine contractions is determined by palpating from the beginning of one contraction to the beginning of the next contraction. Assessment of uterine activity is performed in intervals based on the stage of labor. As labor progresses, this assessment is performed more frequently.)

23. A woman has requested an epidural for her pain. She is 5 cm dilated and 100% effaced. The baby is in a vertex position and is engaged. The nurse increases the womans IV fluid for a preprocedural bolus. The nurse reviews her laboratory values and notes that the womans hemoglobin is 12 g/dl, hematocrit is 38%, platelets are 67,000, and white blood cells (WBCs) are 12,000/mm3. Which factor would contraindicate an epidural for this woman? a. She is too far dilated. b. She is anemic. c. She has thrombocytopenia. d. She is septic.

C (The platelet count indicates a coagulopathy, specifically, thrombocytopenia (low platelets), which is a contraindication to epidural analgesia and anesthesia. Typically, epidural analgesia and anesthesia are used in the laboring woman when a regular labor pattern has been achieved, as evidenced by progressive cervical change. The laboratory values show that the womans hemoglobin and hematocrit levels are in the normal range and show a slight increase in the WBC count that is not uncommon in laboring women.)

A woman in labor has just received an epidural block. The most important nursing intervention is to: A.) limit parenteral fluids. B.) monitor the fetus for possible tachycardia. C.) monitor the maternal blood pressure for possible hypotension. D.) monitor the maternal pulse for possible bradycardia.

C.) monitor the maternal blood pressure for possible hypotension.

Which FHR finding would concern the nurse during labor? A. Accelerations with fetal movement B. Early decelerations C. An average FHR of 126 beats/min D. Late decelerations

D A. Incorrect: Accelerations in the FHR are an indication of fetal well-being. B. Incorrect: Early decelerations in the FHR are associated with head compression as the fetus descends into the maternal pelvic outlet; they generally are not a concern during normal labor. C. Incorrect: This FHR finding is normal and not a concern. D. Correct: Late decelerations are caused by uteroplacental insufficiency and are associated with fetal hypoxemia. They are considered ominous if persistent and uncorrected. p. 507

The most common cause of decreased variability in the FHR that lasts 30 minutes or less is: A. Altered cerebral blood flow B. Fetal hypoxemia C. Umbilical cord compression D. Fetal sleep cycles

D A. Incorrect: Altered fetal cerebral blood flow would result in early decelerations in the FHR. B. Incorrect: Fetal hypoxemia would be evidenced by tachycardia initially and then bradycardia. A persistent decrease or loss of FHR variability may be seen. C. Incorrect: Umbilical cord compression would result in variable decelerations in the FHR. D. Correct: A temporary decrease in variability can occur when the fetus is in a sleep state. These sleep states do not usually last longer than 30 minutes. p. 502

As a perinatal nurse, you realize that an FHR that is tachycardic, bradycardic, has late decelerations, or loss of variability is nonreassuring and is associated with: A. Hypotension B. Cord compression C. Maternal drug use D. Hypoxemia

D A. Incorrect: Fetal bradycardia may be associated with maternal hypotension. B. Incorrect: Fetal variable decelerations are associated with cord compression. C. Incorrect: Maternal drug use is associated with fetal tachycardia. D. Correct: Nonreassuring heart rate patterns are associated with fetal hypoxemia. pp. 502-503

A nurse might be called on to stimulate the fetal scalp: A. As part of fetal scalp blood sampling B. In response to tocolysis C. In preparation for fetal oxygen saturation monitoring D. To elicit an acceleration in the FHR

D A. Incorrect: Fetal scalp blood sampling involves swabbing the scalp with disinfectant before a sample is collected. The nurse would stimulate the fetal scalp to elicit an acceleration of the FHR. B. Incorrect: Tocolysis is relaxation of the uterus. The nurse would stimulate the fetal scalp to elicit an acceleration of the FHR. C. Incorrect: Fetal oxygen saturation monitoring involves the insertion of a sensor. The nurse would stimulate the fetal scalp to elicit an acceleration of the FHR. D. Correct: The scalp can be stimulated using digital pressure during a vaginal examination. p. 513

When assessing the relative advantages and disadvantages of internal and external electronic fetal monitoring, nurses should be aware that both: A. Can be used when membranes are intact B. Measure the frequency, duration, and intensity of UCs C. May need to rely on the woman to indicate when UA is occurring D. Can be used during the antepartum and intrapartum periods

D A. Incorrect: For internal monitoring, the membranes must have ruptured and the cervix must be sufficiently dilated. B. Incorrect: Internal monitoring measures the intensity of contractions; external monitoring cannot do this. C. Incorrect: With external monitoring, the woman may need to alert the nurse that UA is occurring; internal monitoring does not require this. D. Correct: External monitoring can be used in both periods; internal monitoring can be used only in the intrapartum period. p. 500

Nurses should be aware that accelerations in the fetal heart rate: A. Are indications of fetal well-being when they are periodic B. Are greater and longer in preterm gestations C. Are usually seen with breech presentations when they are episodic D. May visibly resemble the shape of the uterine contraction

D A. Incorrect: Periodic accelerations occur with UC and usually are seen with breech presentations. Episodic accelerations occur during fetal movement and are indications of fetal well-being. B. Incorrect: Preterm accelerations peak at 10 beats/min above the baseline and last for at least 10 seconds, not 15 seconds. C. Incorrect: Periodic accelerations occur with UC and usually are seen with breech presentations. Episodic accelerations occur during fetal movement and are indications of fetal well-being. D. Correct: They may resemble the shape of the uterine contraction or may be spikelike. p. 507

While evaluating an external monitor tracing of a woman in active labor whose labor is being induced, the nurse notes that the FHR begins to decelerate at the onset of several contractions and returns to baseline before each contraction ends. The nurse should: A. Change the woman's position B. Discontinue the oxytocin infusion C. Insert an internal monitor D. Document the finding in the client's record

D A. Incorrect: The FHR indicates early decelerations, which are not an ominous sign and do not require any intervention. The nurse should simply document these findings. B. Incorrect: The presence of early decelerations is not an ominous sign and does not require any intervention. C. Incorrect: The presence of early decelerations is not an ominous sign and does not require any intervention. D. Correct: The FHR indicates early decelerations, which are not an ominous sign and do not require any intervention. The nurse should simply document these findings. p. 509

You are evaluating the fetal monitor tracing of your client, who is in active labor. Suddenly, you see the FHR drop from its baseline of 125 down to 80. You reposition the mother, provide oxygen, increase intravenous (IV) fluid, and perform a vaginal exam. The cervix has not changed. Five minutes have passed, and the fetal heart rates remain in the 80s. What additional nursing measures should you take? A. Scream for help B. Insert a Foley catheter C. Start pitocin D. Notify the care provider immediately

D A. Incorrect: This is an inappropriate nursing action. B. Incorrect: If the FHR were to continue in a nonreassuring pattern, a cesarean section may be warranted, which would require a Foley catheter. However, the physician must make that determination. C. Incorrect: Pitocin may put additional stress on the fetus. D. Correct: To relieve an FHR deceleration, the nurse can reposition the mother, increase IV fluid, and provide oxygen. Also, if oxytocin is infusing, it should be discontinued. If the FHR does not resolve, the primary care provider should be notified immediately. p. 511

When using IA for FHR, nurses should be aware that: A. They can be expected to cover only two or three clients when IA is the primary method of fetal assessment. B. The best course is to use the descriptive terms associated with EFM when documenting results. C. If the heartbeat cannot be found immediately, a shift must be made to electronic monitoring. D. Ultrasound can be used to find the fetal heartbeat and reassure the mother if initial difficulty was a factor.

D A. Incorrect: When used as the primary method of fetal assessment, auscultation requires a nurse-to-client ratio of one to one. B. Incorrect: Documentation should use only terms that can be numerically defined; the usual visual descriptions of EFM are inappropriate. C. Incorrect: Locating fetal heartbeats often takes time. Mothers can be reassured verbally and by the ultrasound pictures if that device is used to help locate the heartbeat. D. Correct: Locating fetal heartbeats often takes time. Mothers can be reassured verbally and by the ultrasound pictures if that device is used to help locate the heartbeat. p. 500

28. The obstetric nurse is preparing the client for an emergency cesarean birth, with no time to administer spinal anesthesia. The nurse is aware of and prepared for the greatest risk of administering general anesthesia to the client. What is this risk? a. Respiratory depression b. Uterine relaxation c. Inadequate muscle relaxation d. Aspiration of stomach contents

D (Aspiration of acidic gastric contents with possible airway obstruction is a potentially fatal complication of general anesthesia. Respirations can be altered during general anesthesia, and the anesthesiologist will take precautions to maintain proper oxygenation. Uterine relaxation can occur with some anesthesia but can be monitored and prevented. Inadequate muscle relaxation can be improved with medication.)

2. When a nulliparous woman telephones the hospital to report that she is in labor, what guidance should the nurse provide or information should the nurse obtain? a. Tell the woman to stay home until her membranes rupture. b. Emphasize that food and fluid intake should stop. c. Arrange for the woman to come to the hospital for labor evaluation. d. Ask the woman to describe why she believes she is in labor.

D (Assessment begins at the first contact with the woman, whether by telephone or in person. By asking the woman to describe her signs and symptoms, the nurse can begin her assessment and gather data. The initial nursing activity should be to gather data about the womans status. The amniotic membranes may or may not spontaneously rupture during labor. The client may be instructed to stay home until the uterine contractions become strong and regular. Before instructing the woman to come to the hospital, the nurse should initiate her assessment during the telephone interview. After this assessment has been made, the nurse may want to discuss the appropriate oral intake for early labor, such as light foods or clear liquids, depending on the preference of the client or her primary health care provider.)

2. A woman who is pregnant for the first time is dilated 3 cm and having contractions every 5 minutes. She is groaning and perspiring excessively; she states that she did not attend childbirth classes. What is the optimal intervention for the nurse to provide at this time? a. Notify the womans health care provider. b. Administer the prescribed narcotic analgesic. c. Assure her that her labor will be over soon. d. Assist her with simple breathing and relaxation instructions.

D (By reducing tension and stress, both focusing and relaxation techniques will allow the woman in labor to rest and conserve energy for the task of giving birth. For those who have had no preparation, instruction in simple breathing and relaxation can be given in early labor and is often successful. The nurse can independently perform many functions in labor and birth, such as teaching and support. Pain medication may be an option for this client. However, the initial response of the nurse should include teaching the client about her options. The length of labor varies among individuals, but the first stage of labor is the longest. At 3 cm of dilation with contractions every 5 minutes, this woman has a significant amount of labor yet to experience.)

1. An 18-year-old pregnant woman, gravida 1, para 0, is admitted to the labor and birth unit with moderate contractions every 5 minutes that last 40 seconds. The client states, My contractions are so strong, I dont know what to do. Before making a plan of care, what should the nurses first action be? a. Assess for fetal well-being. b. Encourage the woman to lie on her side. c. Disturb the woman as little as possible. d. Recognize that pain is personalized for each individual.

D (Each womans pain during childbirth is unique and is influenced by a variety of physiologic, psychosocial, and environmental factors. A critical issue for the nurse is how support can make a difference in the pain of the woman during labor and birth. This scenario includes no information that would indicate fetal distress or a logical reason to be overly concerned about the well-being of the fetus. The left lateral position is used to alleviate fetal distress, not maternal stress. The nurse has an obligation to provide physical, emotional, and psychosocial care and support to the laboring woman. This client clearly needs support.)

11. Which statement correctly describes the effects of various pain factors? a. Higher prostaglandin levels arising from dysmenorrhea can blunt the pain of childbirth. b. Upright positions in labor increase the pain factor because they cause greater fatigue. c. Women who move around trying different positions experience more pain. d. Levels of pain-mitigating beta-endorphins are higher during a spontaneous, natural childbirth.

D (Higher endorphin levels help women tolerate pain and reduce anxiety and irritability. Higher prostaglandin levels correspond to more severe labor pains. Upright positions in labor usually result in improved comfort and less pain. Moving freely to find more comfortable positions is important for reducing pain and muscle tension.)

21. A woman in the active phase of the first stage of labor is using a shallow pattern of breathing, which is approximately twice the normal adult breathing rate. She starts to complain about feeling lightheaded and dizzy and states that her fingers are tingling. Which intervention should the nurse immediately initiate? a. Contact the womans physician. b. Tell the woman to slow her pace of her breathing. c. Administer oxygen via a mask or nasal cannula. d. Help her breathe into a paper bag.

D (This woman is experiencing the side effects of hyperventilation, which include the symptoms of lightheadedness, dizziness, tingling of the fingers, or circumoral numbness. Having the woman breathe into a paper bag held tightly around her mouth and nose may eliminate respiratory alkalosis and enable her to rebreathe carbon dioxide and replace the bicarbonate ion.)

7. A client is in early labor, and her nurse is discussing the pain relief options she is considering. The client states that she wants an epidural no matter what! What is the nurses best response? a. Ill make sure you get your epidural. b. You may only have an epidural if your physician allows it. c. You may only have an epidural if you are going to deliver vaginally. d. The type of analgesia or anesthesia used is determined, in part, by the stage of your labor and the method of birth.

D (To avoid suppressing the progress of labor, pharmacologic measures for pain relief are generally not implemented until labor has advanced to the active phase of the first stage and the cervix is dilated approximately 4 to 5 cm. A plan of care is developed for each woman that addresses her particular clinical and nursing problems. The nurse collaborates with the primary health care provider and the laboring woman in selecting features of care relevant to the woman and her family. The decision whether to use an epidural to relieve labor pain is multifactorial. The nurse should not make a blanket statement guaranteeing the client one pharmacologic option over another until a complete history and physical examination has been obtained. A physicians order is required for pharmacologic options for pain management. However, expressing this requirement is not the nurses best response. An epidural is an effective pharmacologic pain management option for many laboring women. It can also be used for anesthesia control if the woman undergoes an operative delivery.)

15. Maternity nurses often have to answer questions about the many, sometimes unusual, ways people have tried to make the birthing experience more comfortable. Which information regarding nonpharmacologic pain relief is accurate? a. Music supplied by the support person has to be discouraged because it could disturb others or upset the hospital routine. b. Women in labor can benefit from sitting in a bathtub, but they must limit immersion to no longer than 15 minutes at a time. c. Effleurage is permissible, but counterpressure is almost always counterproductive. d. Electrodes attached to either side of the spine to provide high-intensity electrical impulses facilitate the release of endorphins.

D (Transcutaneous electrical nerve stimulation (TENS) may help and is most useful for lower back pain that occurs during the first stage of labor. Music may be very helpful for reducing tension and certainly can be accommodated by the hospital. Women can stay in a bath as long as they want, although repeated baths with breaks might be more effective than one long bath. Counterpressure can help the woman cope with lower back pain.)

In evaluating the effectiveness of magnesium sulfate for the treatment of preterm labor, what finding would alert the nurse to possible side effects? A.) Urine output of 160 mL in 4 hours B.) Deep tendon reflexes 2+ and no clonus C.) Respiratory rate of 16 breaths/min D.) Serum magnesium level of 10 mg/dL

D.) Serum magnesium level of 10 mg/dL

A woman at 26 weeks of gestation is being assessed to determine whether she is experiencing preterm labor. What finding indicates that preterm labor is occurring? A.) Estriol is not found in maternal saliva. B.) Irregular, mild uterine contractions are occurring every 12 to 15 minutes. C.) Fetal fibronectin is present in vaginal secretions. D.) The cervix is effacing and dilated to 2 cm.

D.) The cervix is effacing and dilated to 2 cm.

The nurse practicing in a labor setting knows that the woman most at risk for uterine rupture is: A.) a gravida 3 who has had two low-segment transverse cesarean births. B.) a gravida 2 who had a low-segment vertical incision for delivery of a 10-lb infant. C.) a gravida 5 who had two vaginal births and two cesarean births. D.) a gravida 4 who has had all cesarean births.

D.) a gravida 4 who has had all cesarean births.

When caring for a pregnant woman with cardiac problems, the nurse must be alert for signs and symptoms of cardiac decompensation, which include: A.) a regular heart rate and hypertension. B.) an increased urinary output, tachycardia, and dry cough. C.) shortness of breath, bradycardia, and hypertension. D.) dyspnea; crackles; and an irregular, weak pulse.

D.) dyspnea; crackles; and an irregular, weak pulse.

What defines gestational hypertension as opposed to preeclampsia?

It is the onset of htn after week 20 but absent of protienuria and other diagnostics for preeclampsia > 140/90

What is preecampsia?

Pregnancy-specific condition in which hypertension and protienuria develop after week 20 in a normotensive pt

16. Which information regarding the procedures and criteria for admitting a woman to the hospital labor unit is important for the nurse to understand? a. Client is considered to be in active labor when she arrives at the facility with contractions. b. Client can have only her male partner or predesignated doula with her at assessment. c. Children are not allowed on the labor unit. d. NonEnglish speaking client must bring someone to translate.

a (According to the Emergency Medical Treatment and Active Labor Act (EMTALA), a woman is entitled to active labor care and is presumed to be in true labor until a qualified health care provider certifies otherwise. A woman may have anyone she wishes present for her support. An interpreter must be provided by the hospital, either in person or by a telephonic service. Siblings of the new infant may be allowed at the delivery, depending on hospital policy and adequate preparation and supervision.)

29. When managing the care of a woman in the second stage of labor, the nurse uses various measures to enhance the progress of fetal descent. Which instruction best describes these measures? a. Encouraging the woman to try various upright positions, including squatting and standing b. Telling the woman to start pushing as soon as her cervix is fully dilated c. Continuing an epidural anesthetic so pain is reduced and the woman can relax d. Coaching the woman to use sustained, 10- to 15-second, closed-glottis bearing-down efforts with each contraction

a (Both upright and squatting positions may enhance the progress of fetal descent. Many factors dictate when a woman should begin pushing. Complete cervical dilation is necessary, but complete dilation is only one factor. If the fetal head is still in a higher pelvic station, then the physician or midwife may allow the woman to labor down if the woman is able (allowing more time for fetal descent and thereby reducing the amount of pushing needed). The epidural may mask the sensations and muscle control needed for the woman to push effectively. Closed glottic breathing may trigger the Valsalva maneuver, which increases intrathoracic and cardiovascular pressures, reducing cardiac output and inhibiting perfusion of the uterus and placenta. In addition, holding her breath for longer than 5 to 7 seconds diminishes the perfusion of oxygen across the placenta and results in fetal hypoxia.)

26. As the United States and Canada continue to become more culturally diverse, recognizing a wide range of varying cultural beliefs and practices is increasingly important for the nursing staff. A client is from which country if she requests to have the babys father in attendance? a. Mexico b. China c. Iran d. India

a (Hispanic women routinely have fathers and female relatives in attendance during the second stage of labor. The father of the baby is expected to provide encouragement, support, and reassurance that all will be well. In China, fathers are usually not present. The side-lying position is preferred for labor and birth because it is believed that this will reduce trauma to the infant. In China, the client has a stoic response to pain. In Iran, the father will not be present. Female support persons and female health care providers are preferred. For many, a male caregiver is unacceptable. In India, the father is usually not present, but female relatives are usually in attendance. Natural childbirth methods are preferred.)

24. In recovery, if a woman is asked to either raise her legs (knees extended) off the bed or flex her knees, and then place her feet flat on the bed and raise her buttocks well off the bed, the purpose of this exercise is to assess what? a. Recovery from epidural or spinal anesthesia b. Hidden bleeding underneath her c. Flexibility d. Whether the woman is a candidate to go home after 6 hours

a (If the numb or prickly sensations are gone from her legs after these movements, then she has likely recovered from the epidural or spinal anesthesia. Assessing the client for bleeding beneath her buttocks before discharge from the recovery is always important; however, she should be rolled to her side for this assessment. The nurse is not required to assess the woman for flexibility. This assessment is performed to evaluate whether the client has recovered from spinal anesthesia, not to determine if she is a candidate for early discharge.)

17. Which component of the physical examination are Leopolds maneuvers unable to determine? a. Gender of the fetus b. Number of fetuses c. Fetal lie and attitude d. Degree of the presenting parts descent into the pelvis

a (Leopolds maneuvers help identify the number of fetuses, the fetal lie and attitude, and the degree of descent of the presenting part into the pelvis. The gender of the fetus cannot be determined by performing Leopolds maneuvers.)

30. When assessing a multiparous woman who has just given birth to an 8-pound boy, the nurse notes that the womans fundus is firm and has become globular in shape. A gush of dark red blood comes from her vagina. What is the nurses assessment of the situation? a. The placenta has separated. b. A cervical tear occurred during the birth. c. The woman is beginning to hemorrhage. d. Clots have formed in the upper uterine segment.

a (Placental separation is indicated by a firmly contracting uterus, a change in the uterus from a discoid to a globular ovoid shape, a sudden gush of dark red blood from the introitus, an apparent lengthening of the umbilical cord, and a finding of vaginal fullness. Cervical tears that do not extend to the vagina result in minimal blood loss. Signs of hemorrhage are a boggy uterus, bright red vaginal bleeding, alterations in vital signs, pallor, lightheadedness, restlessness, decreased urinary output, and alteration in the level of consciousness. If clots have formed in the upper uterine segment, then the nurse would expect to find the uterus boggy and displaced to the side.)

20. Which technique is an adequate means of controlling the birth of the fetal head during delivery in a vertex presentation? a. Ritgen maneuver b. Fundal pressure c. Lithotomy position d. De Lee apparatus

a (The Ritgen maneuver extends the head during the actual birth and protects the perineum. Gentle, steady pressure against the fundus of the uterus facilitates vaginal birth. The lithotomy position has been commonly used in Western cultures, partly because it is convenient for the health care provider. The De Lee apparatus is used to suction fluid from the infants mouth.)

13. What is the most critical nursing action in caring for the newborn immediately after the birth? a. Keeping the airway clear b. Fostering parent-newborn attachment c. Drying the newborn and wrapping the infant in a blanket d. Administering eye drops and vitamin K

a (The care given immediately after the birth focuses on assessing and stabilizing the newborn. Although fostering parent-newborn attachment is an important task for the nurse, it is not the most critical nursing action in caring for the newborn immediately after birth. The care given immediately after birth focuses on assessing and stabilizing the newborn. The nursing activities are (in order of importance) to maintain a patent airway, to support respiratory effort, and to prevent cold stress by drying the newborn and covering him or her with a warmed blanket or placing the newborn under a radiant warmer. After the newborn has been stabilized, the nurse assesses the newborns physical condition, weighs and measures the newborn, administers prophylactic eye ointment and a vitamin K injection, affixes an identification bracelet, wraps the newborn in warm blankets, and then gives the newborn to the partner or to the mother of the infant.)

9. Under which circumstance should the nurse assist the laboring woman into a hands-and-knees position? a. Occiput of the fetus is in a posterior position. b. Fetus is at or above the ischial spines. c. Fetus is in a vertex presentation. d. Membranes have ruptured.

a (The hands-and-knees position is effective in helping to rotate the fetus from a posterior to an anterior position. Many women experience the irresistible urge to push when the fetus is at the level of the ischial spines. In some cases, this urge is felt before the woman is fully dilated. The woman should be instructed not to push until complete cervical dilation has occurred. No one position is correct for childbirth. The two most common positions assumed by women are the sitting and side-lying positions. The woman may be encouraged into a hands-and-knees position if the umbilical cord prolapsed when the membranes ruptured.)

38. A laboring woman is reclining in the supine position. What is the most appropriate nursing action at this time? a. Ask her to turn to one side. b. Elevate her feet and legs. c. Take her blood pressure. d. Determine whether fetal tachycardia is present.

a (The womans supine position may cause the heavy uterus to compress her inferior vena cava, thus reducing blood return to her heart and reducing placental blood flow. Elevating her legs will not relieve the pressure from the inferior vena cava. If the woman is allowed to stay in the supine position and blood flow to the placental is reduced significantly, then fetal tachycardia may occur. The most appropriate nursing action is to prevent this from occurring by turning the woman to her side. Blood pressure readings may be obtained when the client is in the appropriate and safest position.)

19. The nurse should be aware of which information related to a womans intake and output during labor? a. Traditionally, restricting the laboring woman to clear liquids and ice chips is being challenged because regional anesthesia is used more often than general anesthesia. b. Intravenous (IV) fluids are usually necessary to ensure that the laboring woman stays hydrated. c. Routine use of an enema empties the rectum and is very helpful for producing a clean, clear delivery. d. When a nulliparous woman experiences the urge to defecate, it often means birth will quickly follow.

a (Women are awake with regional anesthesia and are able to protect their own airway, which reduces the worry over aspiration. Routine IV fluids during labor are unlikely to be beneficial and may be harmful. The routine use of an enema is, at best, ineffective and may be harmful. Having the urge to defecate followed by the birth of her fetus is true for a multiparous woman but not for a nulliparous woman.)

What is HELP syndrome?

a laboratory diagnosis for a variant of severe preeclampsia that involves hepatic dysfxn. It is not a separate illness

21. A pregnant woman's biophysical profile score is 8. She asks the nurse to explain the results. The nurse's best response is: a. "The test results are within normal limits." b. "Immediate delivery by cesarean birth is being considered." c. "Further testing will be performed to determine the meaning of this score." d. "An obstetric specialist will evaluate the results of this profile and, within the next week, will inform you of your options regarding delivery."

a. "The test results are within normal limits."

8. A client asks her nurse, "My doctor told me that he is concerned with the grade of my placenta because I am overdue. What does that mean?" The best response by the nurse is: a. "Your placenta changes as your pregnancy progresses, and it is given a score that indicates the amount of calcium deposits it has. The more calcium deposits, the higher the grade, or number, that is assigned to the placenta. It also means that less blood and oxygen can be delivered to your baby." b. "Your placenta isn't working properly, and your baby is in danger." c. "This means that we will need to perform an amniocentesis to detect if you have any placental damage." d. "Don't worry about it. Everything is fine."

a. "Your placenta changes as your pregnancy progresses, and it is given a score that indicates the amount of calcium deposits it has. The more calcium deposits, the higher the grade, or number, that is assigned to the placenta. It also means that less blood and oxygen can be delivered to your baby."

3. The nurse sees a woman for the first time when she is 30 weeks pregnant. The woman has smoked throughout the pregnancy, and fundal height measurements now are suggestive of growth restriction in the fetus. In addition to ultrasound to measure fetal size, what other tool would be useful in confirming the diagnosis? a. Doppler blood flow analysis b. Contraction stress test (CST) c. Amniocentesis d. Daily fetal movement counts

a. Doppler blood flow analysis

16. Compared with contraction stress test (CST), nonstress test (NST) for antepartum fetal assessment: a. Has no known contraindications. b. Has fewer false-positive results. c. Is more sensitive in detecting fetal compromise. d. Is slightly more expensive.

a. Has no known contraindications.

13. Nurses should be aware that the biophysical profile (BPP): a. Is an accurate indicator of impending fetal well being. b. Is a compilation of health risk factors of the mother during the later stages of pregnancy. c. Consists of a Doppler blood flow analysis and an amniotic fluid index. d. Involves an invasive form of ultrasound examination.

a. Is an accurate indicator of impending fetal well being.

22. Which analysis of maternal serum may predict chromosomal abnormalities in the fetus? a. Multiple-marker screening b. Lecithin/sphingomyelin (L/S) ratio c. Biophysical profile d. Type and crossmatch of maternal and fetal serum

a. Multiple-marker screening

9. A woman is undergoing a nipple-stimulated contraction stress test (CST). She is having contractions that occur every 3 minutes. The fetal heart rate (FHR) has a baseline of approximately 120 beats/min without any decelerations. The interpretation of this test is said to be: a. Negative. b. Positive. c. Satisfactory. d. Unsatisfactory.

a. Negative.

2. A 39-year-old primigravida thinks that she is about 8 weeks pregnant, although she has had irregular menstrual periods all her life. She has a history of smoking approximately one pack of cigarettes a day, but she tells you that she is trying to cut down. Her laboratory data are within normal limits. What diagnostic technique could be used with this pregnant woman at this time? a. Ultrasound examination b. Maternal serum alpha-fetoprotein (MSAFP) screening c. Amniocentesis d. Nonstress test (NST)

a. Ultrasound examination

How does HELP syndrome occur? What are the pathophysiologic changes?

arteriolar vasospasm, endothelial cell dysfxn with fibrin deposits, and adherence of platelets in blood vessels RBC's damaged as they pass through narrowed blood vessels and become hemolyzed= decreased RBC count and hyperbilirubinemia Endothelial damage and fibrin deposits in liver= impaired liver dysfxn and can lead to hemorrhagic necrosis

22. Which statement concerning the third stage of labor is correct? a. The placenta eventually detaches itself from a flaccid uterus. b. An expectant or active approach to managing this stage of labor reduces the risk of complications. c. It is important that the dark, roughened maternal surface of the placenta appears before the shiny fetal surface. d. The major risk for women during the third stage is a rapid heart rate.

b (Active management facilitates placental separation and expulsion, reducing the risk of complications. The placenta cannot detach itself from a flaccid (relaxed) uterus. Which surface of the placenta comes out first is not clinically important. The major risk for women during the third stage of labor is postpartum hemorrhaging.)

36. A woman who is gravida 3 para 2 arrives on the intrapartum unit. What is the most important nursing assessment at this time? a. Contraction pattern, amount of discomfort, and pregnancy history b. FHR, maternal vital signs, and the womans nearness to birth c. Identification of ruptured membranes, womans gravida and para, and her support person d. Last food intake, when labor began, and cultural practices the couple desires

b (All options describe relevant intrapartum nursing assessments; however, this focused assessment has a priority. If the maternal and fetal conditions are normal and birth is not imminent, then other assessments can be performed in an unhurried manner; these include: gravida, para, support person, pregnancy history, pain assessment, last food intake, and cultural practices.)

34. Under which circumstance would it be unnecessary for the nurse to perform a vaginal examination? a. Admission to the hospital at the start of labor b. When accelerations of the FHR are noted c. On maternal perception of perineal pressure or the urge to bear down d. When membranes rupture

b (An accelerated FHR is a positive sign; therefore, a vaginal examination would not be necessary. A vaginal examination should be performed when the woman is admitted to the hospital, when she perceives perineal pressure or the urge to bear down, when her membranes rupture, when a significant change in her uterine activity has occurred, or when variable decelerations of the FHR are noted.)

32. A woman who is 39 weeks pregnant expresses fear about her impending labor and how she will manage. What is the nurses ideal response? a. Dont worry about it. Youll do fine. b. Its normal to be anxious about labor. Lets discuss what makes you afraid. c. Labor is scary to think about, but the actual experience isnt. d. You can have an epidural. You wont feel anything.

b (Its normal to be anxious about labor. Lets discuss what makes you afraid is a statement that allows the woman to share her concerns with the nurse and is a therapeutic communication tool. Dont worry about it. Youll do fine negates the womans fears and is not therapeutic. Labor is scary to think about, but the actual experience isnt negates the womans fears and offers a false sense of security. To suggest that every woman can have an epidural is untrue. A number of criteria must be met before an epidural is considered. Furthermore, many women still experience the feeling of pressure with an epidural.)

14. What is the rationale for the administration of an oxytocic (e.g., Pitocin, Methergine) after expulsion of the placenta? a. To relieve pain b. To stimulate uterine contraction c. To prevent infection d. To facilitate rest and relaxation

b (Oxytocics stimulate uterine contractions, which reduce blood loss after the third stage of labor. Oxytocics are not used to treat pain, do not prevent infection, and do not facilitate rest and relaxation.)

31. After an emergency birth, the nurse encourages the woman to breastfeed her newborn. What is the primary purpose of this activity? a. To facilitate maternal-newborn interaction b. To stimulate the uterus to contract c. To prevent neonatal hypoglycemia d. To initiate the lactation cycle

b (Stimulation of the nipples through breastfeeding or manual stimulation causes the release of oxytocin and prevents maternal hemorrhage. Breastfeeding facilitates maternal-newborn interaction, but it is not the primary reason a woman is encouraged to breastfeed after an emergency birth. The primary intervention for preventing neonatal hypoglycemia is thermoregulation. Cold stress can result in hypoglycemia. The woman is encouraged to breastfeed after an emergency birth to stimulate the release of oxytocin, which prevents hemorrhaging. Breastfeeding is encouraged to initiate the lactation cycle, but it is not the primary reason for this activity after an emergency birth.)

15. Which description of the phases of the first stage of labor is most accurate? a. Latent: mild, regular contractions; no dilation; bloody show b. Active: moderate, regular contractions; 4 to 7 cm dilation c. Lull: no contractions; dilation stable d. Transition: very strong but irregular contractions; 8 to 10 cm dilation

b (The active phase is characterized by moderate and regular contractions, 4 to 7 cm dilation, and duration of 3 to 6 hours. The latent phase is characterized by mild-to-moderate and irregular contractions, dilation up to 3 cm, brownish-to-pale pink mucus, and duration of 6 to 8 hours. No official lull phase exists in the first stage. The transition phase is characterized by strong to very strong and regular contractions, 8 to 10 cm dilation, and duration of 20 to 40 minutes.)

25. A woman who has a history of sexual abuse may have a number of traumatic memories triggered during labor. She may fight the labor process and react with pain or anger. The nurse can implement a number of care measures to help her client view the childbirth experience in a positive manner. Which intervention is key for the nurse to use while providing care? a. Tell the client to relax and that it wont hurt much. b. Limit the number of procedures that invade her body. c. Reassure the client that, as the nurse, you know what is best. d. Allow unlimited care providers to be with the client.

b (The number of invasive procedures such as vaginal examinations, internal monitoring, and IV therapy should be limited as much as possible. The nurse should always avoid words and phrases that may result in the clients recalling the phrases of her abuser (i.e., Relax, this wont hurt or Just open your legs). The womans sense of control should be maintained at all times. The nurse should explain procedures at the clients pace and wait for permission to proceed. Protecting the clients environment by providing privacy and limiting the number of staff who observe the client will help to make her feel safe.)

12. Through a vaginal examination, the nurse determines that a woman is 4 cm dilated. The external fetal monitor shows uterine contractions every to 4 minutes. The nurse reports this as what stage of labor? a. First stage, latent phase b. First stage, active phase c. First stage, transition phase d. Second stage, latent phase

b (This maternal progress indicates that the woman is in the active phase of the first stage of labor. During the latent phase of the first stage of labor, the expected maternal progress is 0 to 3 cm dilation with contractions every 5 to 30 minutes. During the transition phase of the first stage of labor, the expected maternal progress is 8 to 10 cm dilation with contractions every 2 to 3 minutes. During the latent phase of the second stage of labor, the woman is completely dilated and experiences a restful period of laboring down.)

4. A 41-week pregnant multigravida presents in the labor and delivery unit after a nonstress test indicated that her fetus could be experiencing some difficulties in utero. Which diagnostic tool would yield more detailed information about the fetus? a. Ultrasound for fetal anomalies b. Biophysical profile (BPP) c. Maternal serum alpha-fetoprotein (MSAFP) screening d. Percutaneous umbilical blood sampling (PUBS)

b. Biophysical profile (BPP)

18. A woman has been diagnosed with a high risk pregnancy. She and her husband come into the office in a very anxious state. She seems to be coping by withdrawing from the discussion, showing declining interest. The nurse can best help the couple by: a. Telling her that the physician will isolate the problem with more tests. b. Encouraging her and urging her to continue with childbirth classes. c. Becoming assertive and laying out the decisions the couple needs to make. d. Downplaying her risks by citing success rate studies.

b. Encouraging her and urging her to continue with childbirth classes.

24. Which nursing intervention is necessary before a second-trimester transabdominal ultrasound? a. Place the woman NPO for 12 hours. b. Instruct the woman to drink 1 to 2 quarts of water. c. Administer an enema. d. Perform an abdominal preparation.

b. Instruct the woman to drink 1 to 2 quarts of water.

20. Risk factors tend to be interrelated and cumulative in their effect. While planning the care for a laboring client with diabetes mellitus, the nurse is aware that she is at a greater risk for: a. Oligohydramnios. b. Polyhydramnios. c. Postterm pregnancy. d. Chromosomal abnormalities.

b. Polyhydramnios.

7. Maternal serum alpha-fetoprotein (MSAFP) screening indicates an elevated level. MSAFP screening is repeated and again is reported as higher than normal. What would be the next step in the assessment sequence to determine the well-being of the fetus? a. Percutaneous umbilical blood sampling (PUBS) b. Ultrasound for fetal anomalies c. Biophysical profile (BPP) for fetal well-being d. Amniocentesis for genetic anomalies

b. Ultrasound for fetal anomalies

12. In the first trimester, ultrasonography can be used to gain information on: a. Amniotic fluid volume. b. the presence of maternal abnormalities c. Placental location and maturity. d. Cervical length.

b. the presence of maternal abnormalities

39. Which nursing assessment indicates that a woman who is in second-stage labor is almost ready to give birth? a. Fetal head is felt at 0 station during vaginal examination. b. Bloody mucous discharge increases. c. Vulva bulges and encircles the fetal head. d. Membranes rupture during a contraction.

c (A bulging vulva that encircles the fetal head describes crowning, which occurs shortly before birth. Birth of the head occurs when the station is +4. A 0 station indicates engagement. Bloody show occurs throughout the labor process and is not an indication of an imminent birth. ROM can occur at any time during the labor process and does not indicate an imminent birth.)

11. Which clinical finding indicates that the client has reached the second stage of labor? a. Amniotic membranes rupture. b. Cervix cannot be felt during a vaginal examination. c. Woman experiences a strong urge to bear down. d. Presenting part of the fetus is below the ischial spines.

c (During the descent phase of the second stage of labor, the woman may experience an increase in the urge to bear down. The ROM has no significance in determining the stage of labor. The second stage of labor begins with full cervical dilation. Many women may have an urge to bear down when the presenting fetal part is below the level of the ischial spines. This urge can occur during the first stage of labor, as early as with 5 cm dilation.)

40. What is the primary rationale for the thorough drying of the infant immediately after birth? a. Stimulates crying and lung expansion b. Removes maternal blood from the skin surface c. Reduces heat loss from evaporation d. Increases blood supply to the hands and feet

c (Infants are wet with amniotic fluid and blood at birth, and this accelerates evaporative heat loss. The primary purpose of drying the infant is to prevent heat loss. Although rubbing the infant stimulates crying, it is not the main reason for drying the infant. This process does not remove all the maternal blood.)

18. Where is the point of maximal intensity (PMI) of the FHR located? a. Usually directly over the fetal abdomen b. In a vertex position, heard above the mothers umbilicus c. Heard lower and closer to the midline of the mothers abdomen as the fetus descends and internally rotates d. In a breech position, heard below the mothers umbilicus

c (Nurses should be prepared for the shift. The PMI of the FHR is usually directly over the fetal back. In a vertex position, the PMI of the FHR is heard below the mothers umbilicus. In a breech position, it is heard above the mothers umbilicus.)

35. Which description of the phases of the second stage of labor is most accurate? a. Latent phase: Feeling sleepy; fetal station 2+ to 4+; duration of 30 to 45 minutes b. Active phase: Overwhelmingly strong contractions; Ferguson reflux activated; duration of 5 to 15 minutes c. Descent phase: Significant increase in contractions; Ferguson reflux activated; average duration varies d. Transitional phase: Woman laboring down; fetal station 0; duration of 15 minutes

c (The descent phase begins with a significant increase in contractions; the Ferguson reflex is activated, and the duration varies, depending on a number of factors. The latent phase is the lull or laboring down period at the beginning of the second stage and lasts 10 to 30 minutes on average. The second stage of labor has no active phase. The transition phase is the final phase in the second stage of labor; contractions are strong and painful.)

28. The first 1 to 2 hours after birth is sometimes referred to as what? a. Bonding period b. Third stage of labor c. Fourth stage of labor d. Early postpartum period

c (The first 2 hours of the birth are a critical time for the mother and her baby and is often called the fourth stage of labor. Maternal organs undergo their initial readjustment to a nonpregnant state. The third stage of labor lasts from the birth of the baby to the expulsion of the placenta. Bonding will occur over a much longer period, although it may be initiated during the fourth stage of labor.)

10. A nulliparous woman has just begun the latent phase of the second stage of her labor. The nurse should anticipate which behavior? a. A nulliparous woman will experience a strong urge to bear down. b. Perineal bulging will show. c. A nulliparous woman will remain quiet with her eyes closed between contractions. d. The amount of bright red bloody show will increase.

c (The woman is able to relax and close her eyes between contractions as the fetus passively descends. The woman may be very quiet during this phase. During the latent phase of the second stage of labor, the urge to bear down is often absent or only slight during the acme of the contractions. Perineal bulging occurs during the transition phase of the second stage of labor, not at the beginning of the second stage. An increase in bright red bloody show occurs during the descent phase of the second stage of labor.)

23. A 25-year-old gravida 3, para 2 client gave birth to a 9-pound, 7-ounce boy, 4 hours ago after augmentation of labor with oxytocin (Pitocin). She presses her call light, and asks for her nurse right away, stating Im bleeding a lot. What is the most likely cause of postpartum hemorrhaging in this client? a. Retained placental fragments b. Unrepaired vaginal lacerations c. Uterine atony d. Puerperal infection

c (This woman gave birth to a macrosomic infant after oxytocin augmentation. Combined with these risk factors, uterine atony is the most likely cause of bleeding 4 hours after delivery. Although retained placental fragments may cause postpartum hemorrhaging, it is typically detected within the first hour after delivery of the placenta and is not the most likely cause of the hemorrhaging in this woman. Although unrepaired vaginal lacerations may also cause bleeding, it typically occurs in the period immediately after birth. Puerperal infection can cause subinvolution and subsequent bleeding that is, however, typically detected 24 hours postpartum.)

23. While working with the pregnant woman in her first trimester, the nurse is aware that chorionic villus sampling (CVS) can be performed during pregnancy at: a. 4 weeks b. 8 weeks c. 10 weeks d. 14 weeks

c. 10 weeks

14. With regard to amniocentesis, nurses should be aware that: a. Because of new imaging techniques, amniocentesis is now possible in the first trimester. b. Despite the use of ultrasound, complications still occur in the mother or infant in 5% to 10% of cases. c. Administration RhoD immunoglobulin may be necessary. d. The presence of meconium in the amniotic fluid is always cause for concern.

c. Administration RhoD immunoglobulin may be necessary.

5. At 35 weeks of pregnancy a woman experiences preterm labor. Tocolytics are administered and she is placed on bed rest, but she continues to experience regular uterine contractions, and her cervix is beginning to dilate and efface. What would be an important test for fetal well-being at this time? a. Percutaneous umbilical blood sampling (PUBS) b. Ultrasound for fetal size c. Amniocentesis for fetal lung maturity d. Nonstress test (NST)

c. Amniocentesis for fetal lung maturity

17. The nurse providing care for the antepartum woman should understand that contraction stress test (CST): a. Sometimes uses vibroacoustic stimulation. b. Is an invasive test; however, contractions are stimulated. c. Is considered negative if no late decelerations are observed with the contractions. d. Is more effective than nonstress test (NST) if the membranes have already been ruptured.

c. Is considered negative if no late decelerations are observed with the contractions.

10. When nurses help their expectant mothers assess the daily fetal movement counts, they should be aware that: a. Alcohol or cigarette smoke can irritate the fetus into greater activity. b. "Kick counts" should be taken every half hour and averaged every 6 hours, with every other 6-hour stretch off. c. The fetal alarm signal should go off when fetal movements stop entirely for 12 hours. d. Obese mothers familiar with their bodies can assess fetal movement as well as average-size women.

c. The fetal alarm signal should go off when fetal movements stop entirely for 12 hours.

21. Which collection of risk factors will most likely result in damaging lacerations, including episiotomies? a. Dark-skinned woman who has had more than one pregnancy, who is going through prolonged second-stage labor, and who is attended by a midwife b. Reddish-haired mother of two who is going through a breech birth c. Dark-skinned first-time mother who is going through a long labor d. First-time mother with reddish hair whose rapid labor was overseen by an obstetrician

d (Reddish-haired women have tissue that is less distensible than darker-skinned women and therefore may have less efficient healing. First-time mothers are also at greater risk, especially with breech births, long second-stage labors, or rapid labors during which the time for the perineum to stretch is insufficient. The rate of episiotomies is higher when obstetricians rather than midwives attend the births. The woman in the first scenario (a) is at low risk for either damaging lacerations or an episiotomy. She is multiparous, has dark skin, and is being attended by a midwife, who is less likely to perform an episiotomy. Reddish-haired women have tissue that is less distensible than that of darker-skinned women. Consequently, the client in the second scenario (b) is at increased risk for lacerations; however, she has had two previous deliveries, which result in a lower likelihood of an episiotomy. The fact that the woman in the third scenario (c) is experiencing a prolonged labor might increase her risk for lacerations. Fortunately, she is dark skinned, which indicates that her tissue is more distensible than that of fair-skinned women and therefore less susceptible to injury.)

27. The Valsalva maneuver can be described as the process of making a forceful bearing-down attempt while holding ones breath with a closed glottis and a tightening of the abdominal muscles. When is it appropriate to instruct the client to use this maneuver? a. During the second stage to enhance the movement of the fetus b. During the third stage to help expel the placenta c. During the fourth stage to expel blood clots d. Not at all

d (The client should not be instructed to use this maneuver. This process stimulates the parasympathetic division of the autonomic nervous system and produces a vagal response (decrease in heart rate and blood pressure.) An alternative method includes instructing the client to perform open-mouth and open-glottis breathing and pushing.)

7. The nurse performs a vaginal examination to assess a clients labor progress. Which action should the nurse take next? a. Perform an examination at least once every hour during the active phase of labor. b. Perform the examination with the woman in the supine position. c. Wear two clean gloves for each examination. d. Discuss the findings with the woman and her partner.

d (The nurse should discuss the findings of the vaginal examination with the woman and her partner, as well as report the findings to the primary care provider. A vaginal examination should be performed only when indicated by the status of the woman and her fetus. The woman should be positioned so as to avoid supine hypotension. The examiner should wear a sterile glove while performing a vaginal examination for a laboring woman.)

8. A multiparous woman has been in labor for 8 hours. Her membranes have just ruptured. What is the nurses highest priority in this situation? a. Prepare the woman for imminent birth. b. Notify the womans primary health care provider. c. Document the characteristics of the fluid. d. Assess the fetal heart rate (FHR) and pattern.

d (The umbilical cord may prolapse when the membranes rupture. The FHR and pattern should be closely monitored for several minutes immediately after the ROM to ascertain fetal well-being, and the findings should be documented. The ROM may increase the intensity and frequency of the uterine contractions, but it does not indicate that birth is imminent. The nurse may notify the primary health care provider after ROM occurs and the fetal well-being and response to ROM have been assessed. The nurses priority is to assess fetal well-being. The nurse should document the characteristics of the amniotic fluid, but the initial response is to assess fetal well-being and the response to ROM.)

37. A primigravida at 39 weeks of gestation is observed for 2 hours in the intrapartum unit. The FHR has been normal. Contractions are 5 to 9 minutes apart, 20 to 30 seconds in duration, and of mild intensity. Cervical dilation is 1 to 2 cm and uneffaced (unchanged from admission). Membranes are intact. What disposition would the nurse anticipate? a. Admitted and prepared for a cesarean birth b. Admitted for extended observation c. Discharged home with a sedative d. Discharged home to await the onset of true labor

d (This situation describes a woman with normal assessments who is probably in false labor and will likely not deliver rapidly once true labor begins. No further assessments or observations are indicated; therefore, the client will be discharged along with instructions to return when contractions increase in intensity and frequency. Neither a cesarean birth nor a sedative is required at this time.)

33. Which characteristic of a uterine contraction is not routinely documented? a. Frequency: how often contractions occur b. Intensity: strength of the contraction at its peak c. Resting tone: tension in the uterine muscle d. Appearance: shape and height

d (Uterine contractions are described in terms of frequency, intensity, duration, and resting tone. Appearance is not routinely charted.)

1. A woman arrives at the clinic seeking confirmation that she is pregnant. The following information is obtained: She is 24 years old with a body mass index (BMI) of 17.5. She admits to having used cocaine "several times" during the past year and drinks alcohol occasionally. Her blood pressure (BP) is 108/70 mm Hg, her pulse rate is 72 beats/min, and her respiratory rate is 16 breaths/min. The family history is positive for diabetes mellitus and cancer. Her sister recently gave birth to an infant with a neural tube defect (NTD). Which characteristics place the woman in a high risk category? a. Blood pressure, age, BMI b. Drug/alcohol use, age, family history c. Family history, blood pressure, BMI d. Family history, BMI, drug/alcohol abuse

d. Family history, BMI, drug/alcohol abuse

15. Nurses should be aware of the strengths and limitations of various biochemical assessments during pregnancy, including that: a. Chorionic villus sampling (CVS) is becoming more popular because it provides early diagnosis. b. Maternal serum alpha-fetoprotein (MSAFP) screening is recommended only for women at risk for neural tube defects. c. Percutaneous umbilical blood sampling (PUBS) is one of the triple-marker tests for Down syndrome. d. MSAFP is a screening tool only; it identifies candidates for more definitive procedures.

d. MSAFP is a screening tool only; it identifies candidates for more definitive procedures.

25. The nurse recognizes that a nonstress test (NST) in which two or more fetal heart rate (FHR) accelerations of 15 beats/min or more occur with fetal movement in a 20-minute period is: a. Nonreactive b. Positive c. Negative d. Reactive

d. Reactive

11. In comparing the abdominal and transvaginal methods of ultrasound examination, nurses should explain to their clients that: a. Both require the woman to have a full bladder. b. The abdominal examination is more useful in the first trimester. c. Initially the transvaginal examination can be painful. d. The transvaginal examination allows pelvic anatomy to be evaluated in greater detail.

d. The transvaginal examination allows pelvic anatomy to be evaluated in greater detail.

What non-specific clinical presentation might a pt report as a result of HELP syndrome?

hx of malaise, flu-like symptoms, epigastric/ RUQ pain, symptoms worse at night

External fetal monitoring cannot detect the ____________________ of uterine contractions.

intensity p. 500

Define eclampsia

the onset of seizure activity or coma in a woman with preeclampsia who has no hx of preexisting patho resulting in seizure activity


Set pelajaran terkait

Circuits and Electricity AP Physics

View Set

BLD 204 Hallmarks of Cancer pt. 2

View Set

Fundamentals of Networking Chapter 12

View Set

Chapter 14: Care of Preoperative Patient

View Set

Sexual Assault Prevention Post-Course Exam

View Set

World War II — Multiple Choice Exercises

View Set

Chap 3. 16e siècles - Renaissance classique

View Set